Ingenious or Misleading Rationale for the "Pause"?

Guest Post by Ira Glickstein

2016IceCubeRSSgif
[Left] The temperature of a glass of ice water “Pauses” at freezing until nearly all the ice melts. [Right] Top: The ice water effect is due to the Heat of Fusion of water. Does that effect apply to the melting of polar ice caps, and explain the statistical “Pause”? Bottom: The IPCC’s climate theory produced climate models that grossly over-estimated warming and failed to predict the “Pause”.
“A glass of ice water in a hot place is certainly warming,” said the confident questioner, “despite the thermometer ‘pausing’ at freeing until most of the ice has melted. Don’t look at the thermometer to detect the warming, watch the ice cubes melt!”

“Seriously,” he continued, “we should watch the alarming melting of glaciers and polar sea ice rather than the ‘Pause’ in Global Warming according to thermometer readings.”

When I give talks about climate science to intelligent audiences, my general theme is that Global Warming is REAL, and partly due to human activities, but it is NOT a big DEAL.,

  • Yes, the Atmospheric “Greenhouse” effect is real. It is responsible for the Earth being about 33⁰C (60⁰F) warmer than it would be absent “Greenhouse” gasses in the Atmosphere.
  • Yes Carbon Dioxide (CO2) is a key “Greenhouse” gas, second only to Water Vapor (H2O).
  • Yes CO2 has increased by about a third during the past century (from 300 to 400 parts per million), mostly due to unprecedented burning of large quantities of coal, oil, and natural gas.
  • Yes, temperatures have gone up by about 0.8⁰C (1.5F) over the past century.
  • HOWEVER, warming is mostly natural and due to Earth’s recovery from the depths of the last ice age, some 18,000 years ago.
  • No matter what we do, the Earth will warm for hundreds or thousands of years, then plunge into the next ice age. Of course this will not happen monotonically. There will be multi-decade periods of warming and of cooling, just as the Medieval Warm Period (1000-1200s) was considerably warmer than today, and the Little Ice Age (1600-1700s) was colder.
  • IPCC climate theory and computer models have failed to match actual satellite temperature data. Alarming predictions have not come to pass. They totally missed the statistical warming “Pause” of the early 2000s. [The IPCC is the Intergovernmental Panel on Climate Change]
  • [See the lower right section of the figure] For several periods, even the lowest edge of the Yellow error band is warmer than the highest edge of the Blue band! [These error bands are 5%-95% statistical confidence limits, which means there is less than 1 chance in 20 any point outside a band is due to random error. Thus, there is less than 1 chance in 20 x 20 = 400 that any point in the White space between the Yellow and Blue bands is due to random error. Either the NASA satellite sensor systems are badly out of order or the IPCC climate models are terribly wrong!]
  • The gross failure of the IPCC models to correctly predict warming, despite a significant increase in CO2, proves that the models, and the underlying IPCC climate theories, are wrong.
  • The most generous explanation is that the IPCC climate scientists simply over-estimated the sensitivity of climate to CO2 increase by a factor of two to three.
  • The most likely explanation is that their climate theory is either incomplete or totally wrong, so their models failed. Either that, or, for political purposes, they purposely jiggered the model parameters to create alarming projections and keep research funding coming from we taxpayers to their organizations.

Rationalizations for what happened to the excess heat due to human-made CO2:

  • The Oceans absorbed it!
  • The melting Ice Caps absorbed it!

How can the world’s leading climate theorists and modelers still be considered competent if they did not know about the heat capacity of the oceans? (Or, apparently, even the Ice Water Experiment! :^)

The Abstract for the recently published study by Michael Nature Trick – Hockey Stick” Mann, et. al admits the reality of the “Pause” “temporary slowdown”. Guess what he blames it on?:

The temporary slowdown in large-scale surface warming during the early 2000s has been attributed to both external and internal sources of climate variability. Using semiempirical estimates of the internal low-frequency variability component in Northern Hemisphere, Atlantic, and Pacific surface temperatures in concert with statistical hindcast experiments, we investigate whether the slowdown and its recent recovery were predictable. We conclude that the internal variability of the North Pacific, which played a critical role in the slowdown, does not appear to have been predictable using statistical forecast methods… [emphasis mine]

In other words, the unpredictable “internal variability of the North Pacific” ate my alarming projection! (A variation on the old “dog ate my homework” excuse :^)

Why was it not predictable by the IPCC’s leading climate scientists?

  • Because statistical forecast methods are weak?
  • Because the catastrophic warming climate theory is wrong?
  • Because they knew better but did not dare to reign in their alarming predictions for fear of losing research grants?

I find it amazing that so many of my friends (who are otherwise intelligent and reasonable) cling to their firm belief in a coming human-caused climate catastrophe. Their confidence is based on the alarming predictions rooted in IPCC climate theory and computer models.

Yet, like the confident questioner I mentioned in the first paragraph, they seem to acknowledge that the IPCC theorists did not know about the relatively simple concepts of ocean heat capacity, or even the temperature profile of ice water due to the Heat of Fusion!

If these  models could not correctly predict a near-term event, such as the “Pause”, why put any credence in their catastrophic predictions for 50 or 100 years hence?2016 IceCubeRSSbase

How Does the Ice Water Experiment Relate to Earth’s Proportion of Ice to Liquid Water?

To satisfy my own curiosity, I decided to do some research and figure out how much the melting of glaciers, sea ice, and ice sheets might have reduced Global Warming since 1979. This period includes the statistical “Pause” (or “temporary slowdown in large-scale surface warming during the early 2000s” as Mann refers to it).

The Ice Water Temperature Pause Experiment works for two reasons:

  1. It takes nearly 80 times as much energy to melt a given mass of ice as it does to raise an equivalent mass of water 1⁰C (1.8⁰F). (This is called the heat of fusion associated with the state transition of water from solid to liquid form.)
  2. The Ice Cubes make up a substantial percentage of the total mass of the ice water mixture. (When the ice cubes melt down to a small proportion of the water, the temperature does rise.)

So, what is the percentage of ice to liquid water on Earth, and has enough of it melted to account for the failure of the IPCC models since 1979, or during the “Pause”?

According to Debenedetti, Pablo G. & H. Eugene Stanley. “Supercooled and Glassy Water.”Physics Today. Vol. 56, No. 6 (June 2003): 40 (quoted by http://hypertextbook.com/facts/2000/HannaBerenblit.shtml) here is what we need to know about the Earth’s Ice and Water:

  • 1,300 x 106 km3 of water in the oceans [106 km3 = millions of cubic kilometers]
  •      33 x 106 km3 of ice in the polar ice caps
    •   3 x 106 km3 in the Greenland ice shelf and
    • 30 x 106 km3 in the Antarctic ice shelf
  •     0.2 x 106 kmof ice in glaciers
  •     0.1 x 106 km3 of water in lakes
  •     0.0012 x 106 kmof water in rivers
  •     0.22 x 106 kmof water in annual precipitation
We can see from the above that virtually all of the Earth’s liquid water is in the oceans and virtually all the ice is in the polar caps. Even if we froze all the water in lakes, rivers, along with annual precipitation, and combined that with the ice in glaciers, the total would be 0.32 x 106 km3, less than 1% of the total ice caps and less than 0.03% of the total water on Earth!
If both the Arctic/Greenland and Antarctic Ice were to melt, that would account for a reduction in warming of about 33 x 80 / 1300 = 2⁰C (3.6⁰F). Wow! That seems substantial, and there certainly would be catastrophic flooding in some low-lying places if all the Earth’s ice melted.

However, actual ice melt rates are much, much, much less, according to

https://nsidc.org/cryosphere/sotc/ice_sheets.html

… best estimates of mass balance changes per year for 1992 through 2011: Greenland: lost 142 ± 49 gigatons; East Antarctica: gained 14 ± 43 gigatons; West Antarctica: lost 65 ± 26 gigatons; Antarctic Peninsula: lost 20 ± 14 gigatons. [net annual melt loss 213 gigatons]

Conveniently, 1 gigaton is the weight of one cubic kilometer (km3) of fresh water. So, 213 gigatons is equal to 213 km3 of ice (momentarily ignoring the fact that 1 km3 of ice weighs a bit less than 1 kmof sea water). Lacking more specifics, let us assume an average annual melt rate of 213 km3 is at least roughly representative of average annual melt rates from 1979 to 2015. Thus, the total melt for 1979-2015 would be 213 x 36 = 7688 km3, which we will round up to 8000 kmto more than make up for the difference in weight of ice and sea water.

So how much does all that melting amount to in terms of delayed temperature increase? 80 x 8000 / 1,300,000,000 = 0.000492⁰C, which we may round up to 0.0005⁰C (0.0009⁰F) of the warming since 1979, and even less of the missing warming during the “Pause”.

So, total Earth ice melt accounts for less than 0.09% of the warming missing from the IPCC’s alarming projection. Not so impressive, is it?

So, if anyone hits you with the Ice Cube Temperature Pause Experiment, congratulate them on being 0.09% right (and thus 99.91% wrong :^)
Ira Glickstein
PS: In case you are having trouble reading the flashing yellow message in the upper right of the first figure, here it is:
  1. Heat of Fusion is the energy to change the state of a gram of a given substance from solid to liquid.
  2. Specific Heat is the energy to raise the temperature of a gram of a given substance 1⁰C.
  3. Water (in calories per gram)    Heat of Fusion = 79.7      Specific Heat    =   1.0
  4. So, it takes ~80 times more energy to melt 1 gram of ice than to heat 1 gram of water 1⁰C.
  5. Therefore it takes lots of heat energy to melt glaciers and polar ice caps.
  6. Does Ice melt explain the IPCC climate model failure to predict the “Pause”?
  7. How much Global Ice is there, and how much Global Ice has melted?
  8. How much Global Ice has melted compared to the volume of the oceans?
  9. Global Ice is only 1/50th of Global Water mass. Less than 1/20,000th of it has melted since 1979.
  10. So, the temperature effect of Global Ice melt during this time period is insignificant, only 0.0005⁰C (0.0009⁰F).
0 0 votes
Article Rating
254 Comments
Oldest
Newest Most Voted
Inline Feedbacks
View all comments
Dan
May 18, 2016 11:06 am
Crispin in Waterloo
Reply to  Ira Glickstein, PhD
May 19, 2016 4:44 am

Even if the ice melt was a ’cause of the pause’ it still means the models are wrong.
Now we see the ice can’t be the explanation so the models still fail and there is no explanation other than the obvious: they are programmed to be much more sensitive to CO2 than the real atmosphere.

john harmsworth
Reply to  Ira Glickstein, PhD
May 19, 2016 9:46 am

So the models were constructed with the activity of one of the world’s largest heat sinks unaccounted for and this could not be foreseen, but they just zipped over it on their way to a conclusion anyway. Are we supposed to believe that they exactly modeled all the others/ That all other significant factors were applied when they missed half the world’s largest ocean with an entropy factor several times that of the entire earth’s atmosphere? What made them think they should run (fly?) around in circles screaming that their hair was on fire and the world was ending and we should tear down the entire world’s economic system based on work that never should have passed peer review? How many models missed this? The headlines should read, ” Global Warming Doomsayers got Fundamentals Wrong”! That’s real news that can save millions of lives.

Harry Passfield
May 18, 2016 11:13 am

Rationale

JimB
May 18, 2016 11:16 am

Problem with this is that it assumes a complete mixing of the meltwater with the seawater. Meltwater will be fresh, and less dense than seawater. Thus it should principally be distributed near the surface of the ocean, magnifying the cooling effect. No?

Anne Ominous
Reply to  JimB
May 18, 2016 12:38 pm

There’s more problem than that.
Because there is a large distance between the poles and the equator, there will be a thermal gradient. Heating will cause the tropics to heat before the poles.
Granted, it will eventually be carried away by wind, currents, etc. But there will be a time delay. As a result, even if ice is ultimately absorbing the energy, there would still be a “hot spot” in the tropical troposphere, which has not been observed.

Bryan A
Reply to  JimB
May 18, 2016 12:39 pm

The other problem is that: If the melting Polar Ice is keeping the Temperatures from rising vis-a-vis “The Pause” and once melted thus goes the pause, how could temperatures increase before the pause when there was even more ice to be melting and an apparently greater albedo?
Why would Melting Ice (happening for the last 250+ years) suddenly cause a pause that it hadn’t caused before?

Gary Meyers
Reply to  Bryan A
May 18, 2016 3:23 pm

Bingo! Good point.

john harmsworth
Reply to  Bryan A
May 19, 2016 11:04 am

Another good headline “Global Warming Theory full of Holes”! What respectable science magazine will publish the truth?

AZ1971
Reply to  JimB
May 18, 2016 12:51 pm

Only in large injection quantities.

george e. smith
Reply to  JimB
May 18, 2016 1:22 pm

Well in Ira’s melting ice experiment, the water starts warming up immediately, it doesn’t wait for most of the ice to melt.
The ice starts to melt because presumably there is something else near the ice that is at a higher Temperature than zero deg. C (fresh water ice), which can conduct “heat energy” (noun) to the ice to supply the latent heat of fusion to some surface portion of the ice, and turn it into water (80 Calories per gram).
That water presumably remains in contact with whatever that hotter other thing is, and the contact has a common Temperature that is above zero.
As a result of the finite thermal conductivity of fresh water, there is a Temperature gradient between the Temperature of the hotter surface, and the ice surface, and it is that Temperature gradient, that keeps pumping heat to the ice/water interface to continue the melting.
So only the water at the interface with the ice surface is at essentially zero deg. C, and the rest is warming up.
Now you can also postulate additional energy in the form of EM radiant energy impinging on the ice, and getting converted into heat, and aiding in the ice melt.
But yes it is interesting that you can take equal masses of zero deg. C ice, and 80 deg. C hot water and mix them in an insulating container to slow heat transfer, and all the ice will melt, and you end up with all of the water at nearly zero deg. C
That’s why we put ice cubes in our Coca Cola.
G

Brad
May 18, 2016 11:24 am

“semiempirical” How can something be semiempirical? That is kind of like being a little pregnant.

Robert of Ottawa
Reply to  Brad
May 18, 2016 1:05 pm

“semiempirical” is half-baked theory with lots and lots of hand-waving.

Patrick
Reply to  Brad
May 18, 2016 1:05 pm

Semiempirical is ‘kind of like’ having sickness in the morning and getting fatter without being pregnant.

Doug in Calgary
Reply to  Brad
May 18, 2016 7:51 pm

Semiempiracal: partly empirical; especially : involving assumptions, approximations, or generalizations designed to simplify calculation or to yield a result in accord with observation.
– Miriam Webster dictionary
The “involving assumptions, approximations, or generalizations” part fully explains why the warmunistas use it.

john harmsworth
Reply to  Brad
May 19, 2016 11:08 am

In Michael Mann’s previous work, semi empirical means a hockey stick shaped lie!

Joe - the climate scientiest
May 18, 2016 11:25 am

Why was the pause not predicted – “Because they knew better but did not dare to reign in their alarming predictions for fear of losing research grants?”
A better explanation is that the Climate High priests were intentionally dishonest.
The pause/amo/pdo cycles showed up in the historical record 4 times since circa 1750.
Did the the climate high priests really believe a trace gas was more powerful than the ocean cycles?

george e. smith
Reply to  Joe - the climate scientiest
May 18, 2016 3:55 pm

Because nobody knew there was going to be a pause, so nobody thought about it until it happened, which is the best time to observe things, especially climate, because trying to predict something before it happens is simply guesswork.
G

joe - the realist climat scientists
Reply to  george e. smith
May 18, 2016 6:24 pm

BS – the pause was showing up every 60-70 years, why would they think it wasnt going to happen this time.

george e. smith
Reply to  george e. smith
May 19, 2016 4:40 pm

I don’t see pauses showing up every 60-70 years.
Yes there are sawtooth cycles of about that duration, but they are anything but pauses.
So BS to your BS.
G

Reply to  george e. smith
May 19, 2016 5:27 pm

There was another ‘Pause’ that started in the 1940’s, and lasted some thirty years.
I didn’t predict that one, either. Nor did anyone else.

Shawn Marshall
Reply to  Joe - the climate scientiest
May 19, 2016 6:14 am

rein not reign

Mark Gilbert
Reply to  Shawn Marshall
May 19, 2016 11:37 am

As much as I enjoy the snippy corrections on Grammar, is it really fair to snipe when you know that the poster cannot edit the original? (Waiting with bated breath for your correction of my post).

george e. smith
Reply to  Shawn Marshall
May 19, 2016 4:43 pm

Hey Mark,
Didja really mean to use the correct misspelling of “baited” breath; excuse me that’s baited breth ! ??
G
PS yes the grammaticians are tiresome; or izzat tiresome ??

Reply to  Shawn Marshall
May 20, 2016 6:29 am

I am also tired of my tablet “auto-magically” correcting the spelling “mistakes” it has found after I have typed in the word and moved on to typing the next. The algorithm used in some of these operating systems are as lacking as those in the climate models. Furthermore, it is near impossible to detect a misspelled word after several sentences have been typed. Finding homonymns is even harder. Some of these adaptive spelling checkers seem to select the homonymn if you have used it more frequently.

Reply to  Joe - the climate scientiest
May 19, 2016 8:45 am

WHY? Because the AGW Cult knew it was a scam from the beginning, all designed to enact Cap-and-trade on the carbon burners/polluters. Then once the economic leaches were firmily attached to the economic money-pits of the world the known cycle of cooling would show up (what we are calling the pause) and they would then proclaim and brag about how effective the carbon reduction program i.e. Cap-and Trade, is thus they will make fortunes forever.

MarkW
May 18, 2016 11:28 am

Let me get this straight, they are actually claiming that arctic/Greenland ice is keeping the US Gulf coast from heating?

MarkW
Reply to  MarkW
May 18, 2016 11:34 am

This is obviously why the Gulf coast doesn’t get hotter during the summer. All that melting Arctic ice keeps it cool.

David L. Hagen
May 18, 2016 11:29 am

Models 99.5% different from data
At Climate Audit, Steve McIntyre compares John Christy’s graph of predictions of 102 runs averaged for 32 global warming models for the tropical tropospheric temperature compared to balloon and satellite data.

In the present case, from the distribution in the right panel:
a model run will be warmer than an observed trend more than 99.5% of the time;
will be warmer than an observed trend by more than 0.1 deg C/decade approximately 88% of the time;
and will be warmer than an observed trend by more than 0.2 deg C/decade more than 41% of the time.
These values demonstrate a very substantial warm bias in models, as reported by Christy, a bias which cannot be dismissed by mere arm-waving about “uncertainties” in Schmidt style.

Schmidt’s Histogram Diagram Doesn’t Refute Christy

adrian smits
May 18, 2016 11:30 am

Ok lets magnify times four you still got zilch.

john harmsworth
Reply to  adrian smits
May 19, 2016 11:11 am

No funding for you!

MarkW
May 18, 2016 11:30 am

Mann is claiming that these unpredictable natural variations that they apparently had no idea even existed before now, are sufficiently strong to completely wipe out the warming that should of occurred.
On the other hand, these same unpredictable natural variations had absolutely no impact on the warming prior the start of the pause.
Who knew that natural variations can only cause cooling?

Joe - the climate scientiest
Reply to  MarkW
May 18, 2016 11:42 am

“Mann is claiming that these unpredictable natural variations that they apparently had no idea even existed before now, are sufficiently strong to completely wipe out the warming that should of occurred.” AS if, something showing up in the historical record 4 times since circa 1750 wasnt going to happen again
“On the other hand, these same unpredictable natural variations had absolutely no impact on the warming prior the start of the pause.” AS if, something showing up in the historical record 4 times since circa 1750 wasnt going to happen again
“Who knew that natural variations can only cause cooling?” As if a full cycle only has one side to the cycle.
I concur – nice demonstration of how dishonest Mann is.

john harmsworth
Reply to  Joe - the climate scientiest
May 19, 2016 11:18 am

Where is the evidence of a Pacific anomaly to reflect this melting? Higher temps should have preceded it and dissipated with the melting. Additionally, if Pacific temps are now cooler that would presage increased future ice cover and a model/prediction of that should prove out. If Pacific temps are not cooler as a result of melting ice then that indicates a constant source of heat input that should be trackable. That would provide usable information to improve models that consistently do not work. His next excuse will be that he left the oven on at his house!

RWturner
Reply to  MarkW
May 18, 2016 11:48 am

“natural variations can only cause cooling?”
I think that is in Chapter 3, section 2, subsection ciii of the Climate Inc., bible.

Alx
Reply to  RWturner
May 18, 2016 7:21 pm

Well yeah, it’s the tenth commandment of the climate dogma; humanity causes warming and natural variability causes cooling. Listen and believe. Or in other words don’t ask any questions.
I think at this point we can dispense with the term science in the phrase “climate science”. Climate Scientology has a nice ring.

AGW is Not Science
Reply to  MarkW
May 18, 2016 11:51 am

You didn’t know this? In “Climastrology,” (what is laughingly referred to as “Climate Science” by the alarmist camp), “natural variability” is just something rolled out as an excuse for the spectacular failure of every Eco-[extremist] prediction of climate catastrophe.
To give credit to any NATURAL variability for ANY of the “warming” undermines the CO2 Boogyman stories, thus they simply won’t allow it.
[edit by Ira]

May 18, 2016 11:30 am

Give thanks for “the pause” and clouds. A Limerick.
The cause for the Climate change pause:
The CO2 increase; because
there’s more clouds in the sky
make more snow, that is why
the climate is stable. Applause!
https://lenbilen.com/2016/02/24/4697/

chaamjamal
May 18, 2016 11:32 am

“Yes CO2 has increased by about a third during the past century (from 300 to 400 parts per million), mostly due to unprecedented burning of large quantities of coal, oil, and natural gas.”
it seems reasonable but is there any empirical evidence to support the attribution of changes in atmospheric CO2 to fossil fuel emissions?
http://papers.ssrn.com/sol3/papers.cfm?abstract_id=2781465
http://papers.ssrn.com/sol3/papers.cfm?abstract_id=2770539
http://papers.ssrn.com/sol3/papers.cfm?abstract_id=2642639
http://papers.ssrn.com/sol3/papers.cfm?abstract_id=2654191
http://papers.ssrn.com/sol3/papers.cfm?abstract_id=2725743

Loren Wilson
Reply to  chaamjamal
May 18, 2016 12:42 pm

How much has moisture, the much more prevalent IR-absorbing gas in the atmosphere, varied? How much have the much larger non-human sources of CO2 varied? Inquiring skeptical minds want to know, because they could make much more difference than human-produced CO2.

Xyzzy11
Reply to  Loren Wilson
May 18, 2016 4:46 pm

@Loren Wilson
Cue Ferdinand Englebeen ….

john harmsworth
Reply to  Loren Wilson
May 19, 2016 11:25 am

that’s a really interesting question. I have read several studies that indicate global humidity has actually fallen. I have a very difficult time believing that a planet that is 75% water on the surface can have any significant variability of humidity when averaged over the globe. If this is possible-and I would bet a large amount of money against it-, the only factor I could come up with is the increased CO2 levels reducing transpiration losses from plants. But I still say it’s impossible

Reply to  john harmsworth
May 19, 2016 11:27 am

JH,
Humidity declines when global T declines.

Alx
Reply to  Ira Glickstein, PhD
May 18, 2016 7:48 pm

I take it “most likely” means your best guess. “majority” means greater than 50%, lets say 51%. That makes 49% is unknown or best guesses. But the unknowns could actually account for 70% or 80%, since they are unknowns we do not know. We have no way knowing what we do not know, I suggest claiming to know the unknown is better left to religion.
Because fossil fuel is a factor does not mean we know how CO2 reached the level it is at now. The system has many factors, all working in chaotic and dynamic ways leading to an incredible complexity we do not understand near well enough.
A blind man touching a elephants tail may claim that the creature is “most likely” similar to a snake. It would be a good guess based on his limited knowledge, but of course is totally wrong.

Reply to  Ira Glickstein, PhD
May 19, 2016 11:14 am

chaam, Ira, Alx,
Climate alarmists always try to paint skeptics into a corner by asking the wrong question: where does the extra CO2 came from — when the critical question is: Who/what do we thank for the recent rise in such a beneficial, life-giving compound?
There are very few examples of a molecule that is so completely harmless, and at the same time so very beneficial to the biosphere: CO2 is every bit as essential to all life on earth as H2O. More is better at current, and probably at any projected concentrations. We wouldn’t even know that CO2 has risen without using very sensitive instruments that can measure a change of only one part in ten thousand; the total rise in CO2 over the past century.
Instead, they want to deflect into irrelevant questions in order to avoid the real question, which should be: Why does the alarmist crowd exhibit such fear and animosity toward a completely natural and harmless compound that is extremely beneficial to all plants and animals?
Quoting Akbar: It’s a trap!

May 18, 2016 11:33 am

“Greenhouse” gasses act by impeding the loss of infrared radiation to space and should warm the atmosphere regardless of what happens to deep ocean water or surface ice. They cannot significantly heat oceans directly as infrared radiation cannot penetrate water more than a few millimetres. The heralded GCMS’ specifically predict atmospheric warming especially in the tropical mid troposphere. The absence of this warming is evidence that the models are either wrong or lack significant understanding of important climate processes. Melting surface ice and hidden heat in deep oceans don’t explain away this empiric evidence.

john harmsworth
Reply to  Andy Pattullo
May 19, 2016 11:37 am

Bang on! Also, no significant additional heat was ever noted in N Pacific surface waters. If the heat input is to be blamed on upwelling, there is no way that there was sufficient time for any recent warming to cycle through the oceans depths. Such heat must have preceded the recent warming by centuries and have nothing to do with man made changes to the atmosphere. The models are known to be wrong and this “admission/cover-up attempt” is proof that they lack significant understanding of important climate processes.

SC
May 18, 2016 11:34 am

Net loss 213 Gt? I thought NASA had reversed it’s assessments pertaining to ice loss in the antarctic.
“Zwally’s team calculated that the mass gain from the thickening of East Antarctica remained steady from 1992 to 2008 at 200 billion tons per year, while the ice losses from the coastal regions of West Antarctica and the Antarctic Peninsula increased by 65 billion tons per year.”
http://www.nasa.gov/feature/goddard/nasa-study-mass-gains-of-antarctic-ice-sheet-greater-than-losses

Reply to  SC
May 19, 2016 9:10 am

How do we differentiate wheather the “loss” is from melting or lack of replacement?

John
May 18, 2016 11:43 am

“East Antarctica: gained 14 ± 43 gigatons”
Wow, sure got that one narrowed down.

Kaiser Derden
May 18, 2016 11:45 am

so if I fill a bathtub with water at 33 degrees and dump in some ice then try to “heat” the tub up with an external heat source the AVERAGE temperature of the water in the tub will remain at 32 degrees until ALL the ice has melted ? really ? or would you expect wide temperature differential in the water depending on how close to the ice or the heated tub walls you measured … yes a tub with JUST water at 33 degrees would heat up faster than the tub that also had ice in it … but I’d bet you’d be hard pressed to measure the lag …

RWturner
Reply to  Kaiser Derden
May 18, 2016 11:54 am

No, the tub with ice would warm substantially slower, not rising in temperature until the ice melted. Heat tends to conduct through water quite easily. This is basically physics 001

MarkW
Reply to  RWturner
May 18, 2016 12:10 pm

I’ve always preferred physics 007.

Reply to  RWturner
May 19, 2016 9:17 am

So, you are willing to set in a tub of water, lukewarm, with 50lbs of ice cubes floating on top, and then turn on only the 120 degree F hot water, without mixing the water in any way?

Reply to  Kaiser Derden
May 18, 2016 12:47 pm

Not quite, the average should be slightly warmer than the melting point, because the water isn’t a perfect conductor of heat, (but it is a very good conductor), and the ice will not take the water below 32F, but some portions of the water will be higher than 32F. I’m not sure if we’re talking about hundreths of a degree or a degree, but the average will be some temperature above freezing.

george e. smith
Reply to  Paul Jackson
May 18, 2016 4:06 pm

Well water is an excellent conductor of heat and I even use it in my radiator to cool my car’s engine.
It is also a fluid so it can also convey heat by convection.
It is the ICE which won’t rise above zero deg. C UNTIL it is melted and becomes water.
You’ve got the horse after the cart.
G

Sparky
Reply to  Paul Jackson
May 22, 2016 8:59 am

Actually water is a very poor conductor of heat. Normally you have to physically mix the water in a bath before you get in, and as an experienced sea swimmer, I can assure you that swimming from a warm current to a cold pool, the boundary is quite sharp.

Reply to  Ira Glickstein, PhD
May 19, 2016 9:31 am

It has been my experience when setting in a bath of water that has gotten cold that when I add very hot water that the hot water draws in the area under the faucet getting hot enough to even burn my feet while the rest of my body remains cold. This can only be remedied by mixing the water. This phenomenon is confirmed while calibrating RTDs in a rotating Watergate calibrator. Without rotating and mixing the temperature closest to the heat source can be 10 – 20 degrees F higher.

john harmsworth
Reply to  Kaiser Derden
May 19, 2016 11:44 am

The size of the tub and the mixing rate is what determines the temperature differentials. Water in contact with ice will be at approximately 32F until all the ice is gone. The ice will also be at 32F until it is all melted. water at a distance from the ice can be at a different temperature ( even lower for salt water) but open water mixes pretty well and heat transfer is quite fast. Surface evaporation, solar heating and heating/cooling from surface air will also have their say.

May 18, 2016 11:47 am

@’confident questioner’
“Seriously,” he continued, “we should watch the alarming melting of glaciers and polar sea ice rather than the ‘Pause’ in Global Warming according to thermometer readings.”
It is hard for me to understand how these various climate models can quantify the predicted rise in sea-levels due to melting ice without being able (using these same models) to quantify the effect of melting ice on global temps. The conundrum is: if the models already knew about ‘heat of fusion’ then why didn’t these models predict the “hiatus”, before it happened? So, if they can’t predict the hiatus correctly, then why trust their predictions on sea-levels rising?
Sounds like an excuse, not a reason.

gnomish
May 18, 2016 11:51 am

“Yes Carbon Dioxide (CO2) is a key “Greenhouse” gas, second only to Water Vapor (H2O).”
nope.
all the rest doesn’t matter after that boner.

Thomas Homer
Reply to  gnomish
May 18, 2016 1:19 pm

I thought the same thing, and that was after this from the article:
‘•Yes, the Atmospheric “Greenhouse” effect is real.’
Once again, the claim is that this is something that can be measured. So why aren’t we measuring it now? If it can be measured, we would be measuring it! And we could set up an equation to show how much warming the Mars’ atmosphere (95% CO2) causes. If it exists at all, then my claim is that it will be a fraction of a micro-degree for the duration of a fraction of a microsecond. On the same order of magnitude as the duration of how long a room full of mirrors stays lit after turning out the lights.

Reply to  Thomas Homer
May 18, 2016 6:24 pm

+ MANY!

gnomish
Reply to  Thomas Homer
May 19, 2016 2:37 pm

ira- the statement under discussion asserted that water vapor was second to co2
[added 20 May by Ira – Please Read it again, what I wrote was “Yes Carbon Dioxide (CO2) is a key “Greenhouse” gas, second only to Water Vapor (H2O).” That means CO2 is SECOND and H2O is FIRST. Of course it is well established that Atmospheric H2O is way, way more effective as a “Greenhouse” gas than CO2.]
now, you do the math.
using the values of 500ppm for co2 and 4% for h2o in a volume of atmosphere, what do you find?
i’ll tell you what- you find that the water vapor, from phase change alone (which makes it the lightest major gas in the atmosphere, requiring no convection at all for it to rise to the stratosphere, carries 50,000 times the heat that co2 can.
so no, water vapor is not second only to co2. that’s idiotic and that’s what we pointed out
no harm done, but your non-sequitur didn’t address the topic at all.

Reply to  gnomish
May 18, 2016 1:20 pm

Please explain. Thanks.

FTOP_T
Reply to  gnomish
May 18, 2016 3:06 pm

When you start out with a false premise:
“Yes, the Atmospheric “Greenhouse” effect is real. It is responsible for the Earth being about 33⁰C (60⁰F) warmer than it would be absent “Greenhouse” gasses in the Atmosphere.”
It goes downhill fast.
Maybe Ira can explain why the moon, without an atmosphere, yet with the same amount of insolation based on distance from the sun reaches 123C whereas the highest temperature ever recorded on earth is 57C. Clearly the presence of an atmosphere with CO2 results in lower temperature.
Further, the ocean cannot be 33C warmer because of CO2. Oceans reach temperatures of 30C+ in the tropics while staying frozen in the arctic. Every warmest when presented with the physics of water argues that CO2 slows cooling of the ocean. Physics does not allow “slowed cooling” to increase a temperature beyond its highest level reached from the outside source. It has to be the sun heating the oceans to 30C, no room left for 33C from CO2.

David Chappell
Reply to  FTOP_T
May 19, 2016 7:52 am

“…the Day side of the Moon does reach 123C, but the Night side of the Moon reaches – 153C, so the average temperature of the Moon is about MINUS 30C. ”
Which, for me, neatly illustrates the nonsense of the concept of a global average temperature. Using that logic, the current (1400 UTC 19 May) average temperature of the earth is -12C calculated from the max recorded temperature of 51.5C (in Pakistan) and the min temperature of -75.4C (in Antarctica)

Sleepalot
Reply to  FTOP_T
May 19, 2016 1:38 pm

“the moon, (…) reaches 123C whereas the highest temperature ever recorded on earth is 57C.”
One is ground temp, one is air temp – those things are not comparable. Earth Max ground temp (Death Valley)
is near +80C (iirc).

Reply to  FTOP_T
May 19, 2016 1:53 pm

Regarding the temperature of the moon, y’all might enjoy my post entitled The Moon Is A Cold Mistress.
w.

FTOP_T
Reply to  FTOP_T
May 19, 2016 7:44 pm

Willis,
Interesting thought experiment. Although I would argue that with equal radiative planetary emissions at TOA, it is emission height that is the determining factor vs. atmospheric composition. Radiative gases lower temp max and raise temp minimum, but do not “heat” anything.
If the AGW argument is CO2 is going to further moderate the earth’s temperature. Exxon should get a humanitarian of the century award.

gnomish
Reply to  gnomish
May 24, 2016 12:43 pm

sorry, Ira- my mistake.
poor reading comprehension on my part.

Tom Halla
May 18, 2016 12:05 pm

+100

RickC PE
May 18, 2016 12:07 pm

If the pause is the result of melting ice (sea ice, glaciers, or whatever), does that mean the pause should last until all that ice has melted? If so, seems we’re in for a very long pause.

MarkW
Reply to  RickC PE
May 18, 2016 12:12 pm

Presumably the ice was melting well prior to 1997 when the pause started.
Does this mean that for some reason, the amount of warming dropped off substantially in 1997?
Or does it mean that the amount of melting doubled or tripled during that year?

Reply to  MarkW
May 18, 2016 12:59 pm

Obviously if the temperature stays the same due to warming now, the previously in the 1930-40’s the temperature went up due to cooling unlike real recently after the 1950’s when the temperature went up due to warming; just remember that the temperatures always go up, the Witch Doctors will tell us while and just keep writing them checks to appease the Gods.

Bryan A
Reply to  RickC PE
May 18, 2016 12:30 pm

The current pause will only last (be acknowledged) until their tongues reach that all important Tripping Point

chris y
May 18, 2016 12:17 pm

This science was settled, unsettled, and resettled by Hansen between 2003 and 2013-
“As we shall see, the small forces that drove millennial climate changes are now overwhelmed by human forcings.”– Hansen et al., 2003
“The longevity of the recent protracted solar minimum, at least two years longer than prior minima of the satellite era, makes that solar minimum potentially a potent force for cooling,” Hansen and his co-authors said.”– Hansen et al., 2011.
“The 5-year mean global temperature has been flat for a decade, which we interpret as a combination of natural variability and a slowdown in the growth rate of the net climate forcing.”– Hansen et al., 2013

Johann Wundersamer
May 18, 2016 12:19 pm

Fascinating presentation; Thanks – Hans

Latitude
May 18, 2016 12:21 pm

Yes, temperatures have gone up by about 0.8⁰C (1.5⁰F) over the past century
====
How much of that is due to “adjustments” and “algorithms”?…….

Davids
May 18, 2016 12:38 pm

A very illuminating and comprehensive debunking of a theory. Really well explained.

May 18, 2016 12:40 pm

Heh.

Joel Snider
May 18, 2016 12:42 pm

This was actually something I tossed out when the excuses for the Pause were somewhere in the mid-teens – the part where the heat was hiding in the bottom of the ocean – why would we be seeing the ocean temperatures rise at the same time all this ice is supposedly melting into the oceans – at the same time they’ are using all that snow melt to explain concurrent record snowfall?

May 18, 2016 12:42 pm

For me it looks like that skeptics admit that the AGW theory has already failed. The error of the IPCC’s model is now about 50 % mainly because of the pause. At the same time the majority of skeptics do not know, what is wrong with the IPCC’s theory. TCS according to IPCC is 0.5 K/(W/m2) * 3.71 W/m2 = 1.85 K. Is this the correct scientific basis or is there something wrong? The skeptics should find the answer. Another option is that IPCC’s formula is correct but there are other forces, which are unknown to IPCC and they are so strong that they can eliminate CO2 forcing and even to cause cooling like during the period from 1945 to 1975 (roughly). So far I have seen only the sun theory as explanation. It is a very good option for the long term fluctuations from hundred to thousand years but is it capable to explain the quicker changes of the temperature?

Editor
Reply to  aveollila
May 18, 2016 4:20 pm

You can know what is wrong with IPCC theory, yet still not know how the climate works. IOW, it is sufficient to show that IPCC theory has fatal errors. For some of the things that we and the IPCC don’t know, see:
https://wattsupwiththat.com/2015/09/17/how-reliable-are-the-climate-models/
In this article I explain one of the major errors in the IPCC report, and touch on some of the climate factors that are not understood.
(The footnotes in this article are actually pretty important)

afonzarelli
Reply to  Mike Jonas
May 18, 2016 5:32 pm

Mike, very nice piece… needs to be circulated as it’s rare to find all this info in one place. (as an example, F. E. once mentioned that they got clouds wrong; i’ve never happened across an explanation till now)

Reply to  Mike Jonas
May 18, 2016 9:04 pm

Mike,
After reading your article, I understood that you accept that the CO2 forcing according to IPCC (RF = 5.35*ln(C/280) is correct. You propose that the problem is that many other things are missing or wrongly combined. Did I get it right?

MRW
Reply to  Mike Jonas
May 19, 2016 8:09 am

Jonas,
I was in the hospital when your article was published last September and missed it. It’s great.

MarkW
Reply to  aveollila
May 19, 2016 9:34 am

“is it capable to explain the quicker changes of the temperature?”
Yes

Reply to  aveollila
May 19, 2016 9:52 am

“At the same time the majority of skeptics do not know, what is wrong with the IPCC’s theory”
This is because there is so much wrong with IPCC theory that if you throw a dart at it the chance of hitting something wrong is nearly 100%. This contributes to the perception that skeptics are not all on the same page, while all of consensus climate science is, albeit the wrong page.
What most have not come to grips with, including many skeptics, is the foundation error that has led to the many other errors that followed. I can say what this is with 100% certainty, and surprise, surprise, Hansen was at the nexus of this error, which got canonized in AR1 and has never been corrected. It has to do with misapplying Bode’s control theory to the climate in order to support the possibility of impossibly large positive feedback.

Reply to  co2isnotevil
May 23, 2016 9:20 am

co2isnotevil wrote: “To calculate the actual net warming of the planet, you need to start with a planet with no ice and clouds which will receive more solar power and be intrinsically warmer.”
To follow up with another observation about your entangling of feedbacks with discussion about the net greenhouse effect. You assert the total greenhouse effect is less in practice because of total cloud effect on albedo. But if you are going to open that can of worms you have to also include ice albedo. Yes, dropping all GHGs out of the atmosphere presumably means *no* clouds and thus less reflectivity which would mean some conceptual warming (how you argue the 33K should be smaller), but the dramatic drop in temperature will mean ice covering most of the world (compare to the last glacial at just ~4.5°C cooler) which will radically ramp up albedo, leading to cooling feedback and permanent snowball earth. You don’t need clouds for reflectivity when you have ice.
The larger point is that the GHE “absent other effects” is purely a theoretical concept, but 33°C is the right figure for that conceptual effect looked at theoretically and in isolation. You have not successfully identified any flaws in how feedbacks are handled in climate science.

Reply to  geoffmprice
May 24, 2016 10:13 am

Geoff,
How can you justify separating the GHG effect of evaporation from the other effects of water? Is it because the GHG effect is the only influence that causes warming, while all other effects result in cooling?
Yes, ice albedo has an effect on the sensitivity, but only along the thin line of latitude where the average temperature is close to 0C. This is such a small part of the surface that the effect is largely negligible planet wide, although consensus climate science doesn’t recognize this as they try and extrapolate this minor influence as a planet wide phenomenon.
You have not paid attention if you think I have not identified serious flaws in how feedback is characterized. This was first done by Hansen (who confused feedback and gain), ‘corrected’ by Schlesinger and committed to doctrine in AR1. IN addition to numerous other errors, neither accounted for the fact that Bode’s amplifier measures input and feedback to determine how much to deliver from an implicit, infinite source of power. The climate system is passive and has no such implicit power supply and instead consumes input and feedback to generate its output. This COE constraint has never been accommodated by consensus climate science because to do so undermines the narrative.
How else do you think the consensus can justify amplifying 3.7 W/m^2 of incremental input into the more than 16 W/m^2 of surface emissions required to sustain a 3C average temperature increase? Every time some ignorant fool says ‘its the feedback’, it reminds me of the line in the film Idiocracy, ‘It’s the electrolytes’.

Reply to  co2isnotevil
May 24, 2016 12:30 pm

“How can you justify separating the GHG effect of evaporation from the other effects of water? Is it because the GHG effect is the only influence that causes warming, while all other effects result in cooling?”
While they are all loosely related to water (this is earth, after all), the radiative effects of water vapor in the atmosphere, cloud feedback, and ice albedo feedback are all “separated” and discussed separately because they are actually different things and it is more useful to talk about them separately. They are not “unfairly” separated where only the warming influences are counted, as you repeatedly assert, but cannot support.
Ice albedo does not result in cooling, it is absolutely a positive feedback, as already discussed – per paleoclimate, per modern observations.
The prevailing evidence on clouds also happens to indicate that it is “likely” positive, i.e. amplifies some warming, again as already discussed. However, there is low confidence in this result and it could be negative.
You are neither supporting your points, nor accurately reflecting (much less addressing) mine.
“Yes, ice albedo has an effect on the sensitivity, but only along the thin line of latitude where the average temperature is close to 0C”
Ice albedo has an effect wherever ice cover changes, which is not a trivial process to summarize and doesn’t really have to do with any “thin line of latitude where the average temperature is close to 0C”. You can’t just say things.
“This is such a small part of the surface that the effect is largely negligible planet wide”
During the last glacial maximum ice sheets advanced quite far in North America, Europe and Asia and the ice caps were much larger. This is an enormous change in terms of albedo and the amount of solar energy reflected by the planet. That you acknowledge the role of cloud feedback but reject ice cover as “negligible”(!) is not remotely logical.
“How else do you think the consensus can justify amplifying 3.7 W/m^2 of incremental input into the more than 16 W/m^2 of surface emissions required to sustain a 3C average temperature increase? Every time some ignorant fool says ‘its the feedback’, it reminds me of the line in the film Idiocracy, ‘It’s the electrolytes’.”
Well I’m not an expert, I see an article that says ECS of 3 translates to ~0.75 ºC/(W/m2). You saying “16 W/m^2 of surface emissions required to sustain a 3C” then seems to imply you are assuming ECS is closer to 0.75(!), far below even the broad consensus uncertainty range of 1.5-4.5 for this value, which comes from looking at the totality of paleoclimate, modern instrumental and model-based ECS estimates which universally find a range for ECS that is far from this value (generally entirely in the positive (>~1.1) feedback range):
http://www.climatechange2013.org/images/figures/WGI_AR5_FigTS_TFE.6-1.jpg
So this is all just another way of saying you are assuming a negative feedback, super stable climate (based on nothing solid that you are able to present – just what appears to be some vague misapplication of electrical engineering analogies.)
Basically, your view flies in the face of a lot of empirical evidence:
* we directly observe things like the strong positive WV radiative feedback
* we do not directly observe dramatic negative feedbacks that would overwhelm this in the other direction. Even cloud feedback (while very uncertain) appears likely positive based on available evidence.
* when we look at paleoclimate we do not see evidence that the climate is extremely resistant to change as you assert. The climate is able to shift into glacial / ice age states and back out based on what appear to be non-dramatic changes in energy input (e.g. orbital shifts in insolation).

Reply to  geoffmprice
May 24, 2016 1:53 pm

Geoff,
Much of what you say are tired old talking points that have been beat to death elsewhere and I see no point to address all of them, but I will address the major points of misinformation.
You fail to see how ice ages are the combined effect of several different orbital factors reinforcing or cancelling the effects of each other. There’s the roughly 20K year cycle as perihelion shifts through the seasons and the roughly 40K cycle as the Earth tilts from minimum to maximum and the 100K year cycle as the eccentricity of out orbit varies over a relatively wide range. A simple Fourier analysis of the temperature clearly shows these cycles. You are also misinformed about the magnitude of these effects. While the yearly average solar input does not change over a very wide range, the effects are amplified by the asymmetry between hemispheres (S has a larger fraction of water and land at the pole) combined with the precession of perihelion which is what Milankovitch discovered a long time ago. There is also a lot of uncertainty about just how constant the output of the Sun really is. After all, most of the stars we observe in the Universe are variable over a very wide range of periods and magnitudes. And of course, the paleo data is absolutely clear that CO2 concentrations respond to temperature changes and not the other way around.
You claim that I am assuming negative feedback and data suggests positive feedback, but that is incorrect. I assume nothing and go by what the data tells me. The data strongly infers something that looks like negative feedback, but as I keep saying, Bode’s analysis, which climate feedback theory is based on, does not apply to the climate system because the climate is a passive system (like an RC circuit), while Bode’s analysis applies only to systems with active gain like transistor amplifiers (actually Bode dates back to vacuum tubes) with an implicit power supply that provides all output. Why is it so hard to understand the difference between active gain and a passive system?
You also don’t seem to understand that the Stefan-Boltzmann LAW dictates that if the surface temperature rises by 3C, its emissions must increase by about 16 W/m^2. At 287K, the surface emits 384.7 W/m^2 and at 290K (3C warmer) it emits 401 W/m^2. This difference is 16.3 W/m^2. For the surface to emit 16.3 W/m^2 more, it must be receiving 16.3 W/m^2 more, otherwise it will cool until input == output. Each of the 239 W/m^2 of input from the Sun results in about 1.6 W/m^2 of emissions by the surface. If the next W/m^2 resulted in 1.6 W/m^2 more emissions by the surface, it would emit 386.3 W/m^2 which corresponds to a temperature of 297.3K or about 0.3C per W/m^2. The 0.8C +/- 0.4C per W/m^2 claimed by the IPCC is so far away from reality it’s absurd. For this to be true, the next W/m^2 of input needs to result in 4.3 W/m^2 more surface emissions. How is it possible that the next W/m^2 of input from the Sun is 2.7 times more powerful at warming the surface than any of the W/m^2 that preceded?
Are you a Bernie Sanders fan? He has trouble with simple arithmetic too.

Reply to  co2isnotevil
May 25, 2016 10:09 am

“You fail to see how ice ages are the combined effect of several different orbital factors reinforcing or cancelling the effects of each other.”
I’m well aware. Your comment is fairly non sequitur; I haven’t commented on this, nor is it particularly relevant to the discussion. Your arguments are a bit of a mess in general.
I see you are handwaving about solar and orbital in order to justify your basically blind assertions about the climate being inherently stable / negative-feedback based. Great. If what we are doing is making up stories, yes anyone can make up stories they like. I like yours fine – I agree it would be more convenient if we imagine a world where global warming isn’t a problem because the climate inherently resists change. On the objective, external earth the planet is warming in defiance of your beliefs about it, however.
“the paleo data is absolutely clear that CO2 concentrations respond to temperature changes and not the other way around.”
So striking how seemingly capable individuals continually repeat these elementary logical fallacies, year after year, despite endless corrections. The paleo data obviously does not tell you that temperature does not respond to CO2 increases. The fallacy is assuming causality can only flow in one direction (fairly basic). It does, in this case, tell you that CO2 does not appear to have been the trigger of the climate change being observed in cases like the most recent glacial.
“You claim that I am assuming negative feedback and data suggests positive feedback, but that is incorrect. I assume nothing and go by what the data tells me.”
Yet strikingly unable to cite data that supports your views or logically support your assertions about such data. For someone who claims to understand climate science better than the worldwide disciplines of climate scientists, geophysicists, atmospheric physicists, geologists and oceanographers, your tendency to argue by assertion and handwave paints a picture that I don’t think you are very self-aware about.
E.g. you may not be aware that people misapplying electrical engineering concepts to climate science is an eye-rolling internet cliche at this point. It is rather preposterous that you think there is a glaring fundamental flaw in the climate concept of feedbacks that over decades thousands of climate scientists and top scientists in every country reviewing climate science (national academies etc.) have overlooked. Perhaps if I have time I’ll try to understand the specifics so I can help you understand your error there. My understanding is that climate scientists use a widely and well understood concept of DC or static feedback per documents like:
http://earthweb.ess.washington.edu/roe/Publications/Roe_FeedbacksRev_08.pdf
“The 0.8C +/- 0.4C per W/m^2 claimed by the IPCC is so far away from reality it’s absurd.”
We are probably both somewhat wrong here – easy to confuse surface vs. TOA fluxes etc. If you would like to highlight exactly what IPCC claim you are talking about here (what WG1 section?) I am willing to try to help debug what you think you have identified here.

Reply to  co2isnotevil
May 25, 2016 10:57 am

It gets tedious reading the constant barrage of the ‘appeal to corrupted authorities’ logical fallacy that regularly appears here. For example, in these comments:
geoffmprice says that co2isnotevil is…
…strikingly unable to cite data that supports your views or logically support your assertions about such data. For someone who claims to understand climate science better than the worldwide disciplines of climate scientists, geophysicists, atmospheric physicists, geologists and oceanographers… &etc.
And:
…over decades thousands of climate scientists and top scientists in every country reviewing climate science (national academies etc.) &etc.
See? That logical fallacy is posted here regularly. But that’s another logical fallacy: the fallacy of endless repetition.
Constant repetition doesn’t make something true. But constant repetition sure is convincing to a certain small clique here. They believe it, because they want to believe it.
Next, geoffmprice says:
If what we are doing is making up stories, yes anyone can make up stories they like.
Regarding the fabricating of ‘stories’, if I may contribute a (true) story about how the IPCC does their ‘science’ behind closed doors:
Each of the statements here were originally ‘expert reviewed and approved’ for Chapter 8 of the 1996 IPCC Asessment Report. But the following committee statements were deleted prior to publication by the Chapter 8 lead author, Ben Santer (of Climategate infamy):
1. “None of the studies cited above has shown clear evidence that we can attribute the observed changes to the specific cause of increases in greenhouse gases.”
2. “While some of the pattern-based studies discussed here have claimed detection of a significant climate change, no study to date has positively attributed all or part (of the observed changes) to antropogenic causes. Nor has any study quantified the magnitude of a greenhouse gas effect or aerosol effect in the observed data – an issue of primary relevance to policy makers.”
3. ”Any claims of positive detection and attribution of significant climate change are likely to remain controversial until uncertainties in the total natural variability of the climate system are reduced.”
4. ”While none of these studies has specifically considered the attribution issue, they often draw some attribution conclusions, for which there is little justification.”
5. ”When will an anthropogenic effect on climate be identified? It is not surprising that the best answer to this question is, `We do not know.’ ”

But Santer deleted those comments, and replaced them with his own comment:
“The body of statistical evidence in chapter 8, when examined in the context of our physical understanding of the climate system, now points to a discernible human influence on the global climate.” [emphasis added]
Twenty years later Santer’s personal view has become a central talking point of the alarmist crowd. Anyone who still believes that the UN/IPCC is primarily interested in science either hasn’t followed years of discussions here, or they’re doing their part to cover for Santer’s wholesale rewriting of the IPCC scientists’ conclusions.

john harmsworth
Reply to  aveollila
May 19, 2016 12:00 pm

The skeptics know many things that are wrong with the AGW theory. The proponents do not seem to know what is wrong with it. They refuse to listen to any of the arguments of the skeptics and continue to patch on “adjustments” such as this latest one by Michael Mann, that completely lack any scientific rationale or justification. The Mann is a liar and is not a scientist!

Reply to  aveollila
May 19, 2016 10:35 pm

To: aveollila. For one skeptic’s calculations showing that climate sensitivity is about 0.54 K, including positive water vapor feedback, but not including net cloud feedbacks which are negative, see https://wattsupwiththat.com/2016/05/12/negative-climate-feedbacks-are-real-and-large/comment-page-1/#comment-2218029 .

Michael 2
May 18, 2016 12:48 pm

Warmth is measured by thermometer. Warmth is measured in degrees. The claim was global warming; not global ice melting.

Reply to  Michael 2
May 18, 2016 1:57 pm

Warmth is measured by thermometer. Warmth is measured in degrees.
The property of objects that is measured by a thermometer is called temperature, an “intensive” property. It’s not the same as internal heat energy (thermodynamic enthalpy), which is an “extensive” property, and is measured in joules or calories.
The claim was global warming; not global ice melting.
Both claims are made by the alarmists. To compute how much heat energy would be necessary to melt a certain amount of ice, you need to know the heat of fusion to compute heat energy required. But if you know the heat of fusion, then you also know that the introduction of heat energy for melting is not observable as a change in temperature. So if the models are “smart” enough to predict the amount of melting, they should also show the correct temperature response caused by melting.

Richard Cain
May 18, 2016 1:18 pm

“So, total Earth ice melt accounts for less 0.09% of the warming missing from the IPCC’s alarming projection”.
That is an interesting calculation. Let’s accept it’s correct, but within an error of ± 0.05%. That’s still an extraordinarily stable state within what most (other than politicians and mass media hacks) will agree is a chaotic system.
I’m therefore taking bets in which year some politician will be the first to promise “I will stop the next ice-age”. Send your betting slip and stake (gold only) in a tightly closed bottle into an ocean current near you addressed to Richard C or his heirs and assigns and subsequent heirs and assigns, EU zone 17 [previously known as Hampshire, England] .
I am offering odds of x10 the number of years between now and if/when you win.
Terms and Conditions Apply
You, your heirs or assigns must
bugger .. ran out of characters

Jack
May 18, 2016 1:36 pm

One must know that a lot of glaciers of medium altitude had disappeared by the end of the medieval optimum. The Little Ice Age beginning by the 14th century made them restored.
One of the farther south glaciers of the French Alps, the Marinet glacier stands between 2500 to 3000 meters in altitude. It is now in a shrinking phase, currently half the size it had by the mid 19th century. The lower end of the ice tongue is becoming a “rocky glacier”: The ice is buried under a thick heap of rocks and boulders that shields it from the sun’s heat.
So it is probable that a lot of glaciers currently are in a similar bad shape and will disappear again within one or two centuries unless the climate will become colder.
Nothing new under the sun except for the warmunists

Michael Jankowski
May 18, 2016 1:49 pm

Could have been watching ice cubes melt for 22,000+ years.

May 18, 2016 2:15 pm

When water changes phase either from liquid water to steam or vice versa or liquid water both directions from ice it does so at a constant temperature, but not constant enthalpy. When water precipitates out of the air onto an ice cap it take a staggering amount of heat with it, tons in fact, 12,000 Btu/h, 3,517 W. That’s why water is the 1,000 Btu/lb gorilla in the living room, orders of magnitude larger than CO2’s puny 2 W/m^2, 6.28 Btu/h / m^2. Even a minor change in the albedo, 100 +/- W/m^2, eclipses CO2’s trivial influence.

May 18, 2016 2:17 pm

How to quantify all that is dumb. good job! lol

dp
May 18, 2016 2:23 pm

Because global ice volume is rather stable the implication is the net melt is equivalent to the net freeze and so is energy neutral. Melt is not going to happen without freeze on an annual basis, and over longer periods it is, well, periodic, and we find ourselves now in a rather pleasant but temporary interglacial period. The natural forces that have maintained this short and long term stability are not understood. Lacking that understanding and with a pent up demand to do something god-like for the world, those who would be god, and they know who they are, invented thermogeddon as a consequence of humans being alive, inventive, and productive. That is why every solution for the unsupportable thermogeddon problem involves devolution of humanity except for the elites who need all the trappings so they can carry on doing the good work on behalf of Gaia.

Reply to  dp
May 19, 2016 9:18 am

Doesn’t the stability of the global ice volume, destroy the climate change meme?

May 18, 2016 2:32 pm

One major flaw in the CO2 argument is that it causes no pressure difference, adding CO2 to an atmosphere causes no pressure change, where as any idiot and their uncle has shown patterns and graphs that are caused by changes in temperature in an atmosphere, see my point?

Reply to  Sparks
May 19, 2016 8:16 am

@Sparks
“… adding CO2 … causes no pressure change …”
Actually, CO2 does contribute its partial pressure to the total atmospheric pressure on Earth. It’s not much though, much less than 1mb.
On Mars, the atmospheric pressure is 6mb, and almost all of it (95%) is CO2. But CO2 is about 28 times more plentiful (by volume) on Mars than on Earth. So the CO2 partial pressure on Earth will be a small fraction of 1mb at the surface. Probably not detectable in the presence of natural pressure variability.

Reply to  Johanus
May 19, 2016 11:23 am

@me
“But CO2 is about 28 times more plentiful (by volume) on Mars than on Earth. ”
I should clarify that statement: for each unit of surface area, the mass of CO2 in a column of air reaching to TOA is 28 times greater than the CO2 mass in a column of air on Earth also reaching to TOA
This is an appropriate measure for comparison, since partial air pressure is proportional to weight of such area-normalized columns.
Here’s the proof in Python2 code:
# Compute ratio of CO2 mass per km2-column for Mars and Earth
mass_earth_co2_kg=3.0e15 # i.e 3 trillion tonnes
mass_mars_co2_kg=2.5e16 * 0.95 # fudged: mass of atmos x 95%
area_earth_km2=5.1e8
area_mars_km2=1.4e8
co2_kg_per_mars_km2=mass_mars_co2_kg/area_mars_km2
co2_kg_per_earth_km2=mass_earth_co2_kg/area_earth_km2
print co2_kg_per_mars_km2/co2_kg_per_earth_km2
#ans=28.8

Sparks
Reply to  Johanus
May 22, 2016 7:43 am

Johanus,
You’re missing a valuable point, (correct me if I picked you up wrong) it is pressure difference that causes temperature, there is no major change in conditions that are caused by humans that can be attributed to atmospheric pressure, obviously atmospheric composition of a planet has a minimalistic effect on its climate.
I see comments all the time that suggest atmospheric composition is a cause of temperature, this is pure nonsense, when a gas is introduced into a planetary atmosphere it does not change the primary forces that cause pressure differences, and we’re talking about CO2, a property found here on earth (net change negligible) no matter how much we can produce of one gas and introduce it to a planetary atmosphere, it will never change the pressure of the planet and never change the pressure differences that fluctuate and change temperature on a planetary scale.
Get a grip of this one clear simple fact and have a look…

John Robertson
May 18, 2016 2:40 pm

Interesting.The Alarmed Ones seem oblivious to the joys of living on a world dominated by water.
A blessing for our kind of life.
Slightly off topic, if the total warming has been a whopping 0.8C, yet the conversion from mercury in glass to electronic sensors incorporated a 0.9C bias higher, does that mean we are actually cooling?
After all this is Climatology,where trends can be created from noise and blessed with world ending confidence.
So the trend is now cooling by 0.1C.
Sarc.

John Soldier
May 18, 2016 3:50 pm

On a related issue, how much energy would be really required to melt the Antarctic ice sheet?
The ice temperature varies with depth, but it is typically at –5 degrees C, that is 5 degrees below freezing point.
Any melting requires that portion of ice to be raised to zero (0 degrees C) then more heat applied (latent heat of
fusion) to convert it to melt water at 0 degrees C. This is what is required for the melting process.
For each 1 tonne (1,000kg) of ice: a. energy required to raise it to 0 degrees C is 17 MJ
b. latent heat of fusion to melt the ice is 334 MJ
Therefore the energy required to melt the ice cap from its present condition = 351 MJ/tonne
Total mass of the ice sheet = 30 million x 0.917 x 10E+9 = 27.5E+15 tonnes (the SG of ice is 0.917)
To melt the entire ice sheet would require 351 x 27.5E+15 = 965.3E+16 MJ heat energy.
But, the entire world’s proven energy reserves of oil, gas and coal in total have an energy content of only
2.87E+16 MJ and these reserves will last us for hundreds of years at current consumption rates.
What this means is that it would take 336 times the energy in all the planet’s fossil fuel reserves to melt the Antarctic ice sheet. (Check the data and do the maths yourself – it’s quite a revelation.)
Expressed another way, if we were silly enough to squander all our energy reserves right away and we focussed all that energy onto the Antarctic ice sheet it would melt only 1/336th portion of it (or 0.3%) and the sea levels would rise only 18cm (or 7 inches). And then we wouldn’t have any energy left to keep warm, or cook, or communicate, or travel, etc.
Those alarmist rising sea level claims are nonsense!

Reply to  John Soldier
May 18, 2016 5:43 pm

Gravity is a variable, the speed of light is the constant, are you confusing mass density to produce energy from a tiny amount, into a MOLE hill? (big joke in there somewhere) lol

May 18, 2016 3:53 pm

“Yes Carbon Dioxide (CO2) is a key “Greenhouse” gas, second only to Water Vapor (H2O)”
Yet more significant to climate, because it is the GHG whose concentration can change independently of temperature. Wouldn’t kill folks to reflect such basic facts. In fact, being able to do this lends credibility to other claims (credibility naturally a core challenge when going up against well-grounded science.)
“HOWEVER, warming is mostly natural and due to Earth’s recovery from the depths of the last ice age, some 18,000 years ago”
Except that global multi-proxy reconstructions of temp show a long slow cooling for the past ~6K years, prior to the sharp warming spike in modern times.
http://www.igbp.net/images/18.30566fc6142425d6c911a95/1384954096860/NL81-PAGES-fig2.gif
Not to mention the fact that this statement is plainly circular/meaningless. “Warming is natural – I can tell because it’s being caused by something since the last ice age.” In conventional science, we look for causes that are quite a bit more specific and rooted in physical rather than philosophical mechanisms, are testable claims, etc.
“No matter what we do, the Earth will warm for hundreds or thousands of years, then plunge into the next ice age”
Same criticism – this is circular. “It isn’t caused by humans because it is natural. It’s natural because it isn’t caused by humans.”
How can there be so much commentary that appears rather completely unilluminated by basic principles of logic?
A trope at this point, but naturally core statements about “all the models are wrong!” via similar rote repetition of assertion.
Total heat uptake by the climate system has been pretty much as predicted:comment image
Tamino recently walked through models vs. surface measurements:
https://tamino.wordpress.com/2016/05/17/models/
With the requisite (desperate) appeal to satellites/MSU. Using the de facto Monckton-driven standard of RSS one presumes, latest version? No? Not just any “satellite data” then.
Does it really not seem desperate to cling to ‘pause’? How many of these statistical ‘pauses’ have there been just since 1970, like half a dozen? Yet with consistent total uptake of heat by the total system / ocean throughout? Models don’t attempt to predict when the flatter La Nina-heavy type phases in surface warming will be (ENSO/weather driven), though they all predict them to exist. What’s relevant to theory and policy is rate of warming per decade, which continues to be right on. But I suppose that dives into subtleties that don’t serve a desired ideological tack.
Such a long article on completely invented controversies. With all due respect, this seems the definition of an alternate reality internet bubble. Makes one tired just looking at the effort that goes into it, having so much text based on profoundly counter-factual premises and wild assertions.

Editor
Reply to  geoffmprice
May 18, 2016 4:51 pm

[My comment in reply to “geoffmprice May 18, 2016 at 3:53 pm” has gone to the end instead of being in-line]

MarkW
Reply to  geoffmprice
May 19, 2016 9:38 am

First off, the claim that the current temperatures are warmer than any temperatures over the last few thousand years has been refuted.
Secondly you are adding proxies with low resolution to modern temperature records with high resolution. An invalid joining to begin with.
Please come back when you can give an honest presentation.

Reply to  MarkW
May 19, 2016 9:55 am

MarkW: “First off, the claim that the current temperatures are warmer than any temperatures over the last few thousand years has been refuted”
Well, Pages 2K is the largest yet done global, high-resolution, multi-proxy reconstruction and it clearly shows that modern temps are warmer than anything in the past couple thousand years, where we saw a slow overall cooling. That’s enough to highlight that Ira’s claims on this are silly. Your personal rejection of evidence and data for whatever reason is likely not relevant here.

Michael 2
Reply to  geoffmprice
May 20, 2016 1:00 pm

geoffmprice writes “Your personal rejection of evidence and data for whatever reason is likely not relevant here.”
Sigh; if only you were the blog owner you could enforce that claim.
But I wonder, if not relevant here, where then it is relevant?

Reply to  MarkW
May 19, 2016 10:50 am

geoffmprice May 19, 2016 at 9:55 am

“Well, Pages 2K is the largest yet done global, high-resolution, multi-proxy reconstruction and it clearly shows that modern temps are warmer than anything in the past couple thousand years, where we saw a slow overall cooling.”

Pages2K may be the “largest”, but that doesn’t stop it from also being one of the wrongest. There are links there to no less than twenty-seven articles here wherein Steve McIntyre points out a number of its many faults (click on “Older Posts” at the bottom for the rest of them).
If you believe Pages2K it is a sure sign you haven’t done your paleohomework.
w.

Reply to  MarkW
May 19, 2016 10:53 am

Heck, Geoff, I almost forgot my own analysis, called The Pages2K Goatrope. Short version? Pages2K is a joke.
w.

Reply to  Willis Eschenbach
May 19, 2016 11:31 am

Thanks for the link Willis. Probably needless to say, your article is not particularly convincing. Per the norm, you sound like a non-expert picking out issues and drawing broad conclusions of incompetence without addressing or even cultivating input from the actual experts into your assessments. As these things usually go, if the Pages 2K researchers actually took the time to argue with you on methodology in depth, the discussion would likely end up where they all generally do, right? Meaning, e.g., Goddard does a good job picking out some oddball adjustment in Paraguay and arguing that the temp record is all fraud etc. to wild applause in the anti-AGW bubble, but if you take the time you find out such arguments are (at least in my experience, uniformly) embarrassingly bunk. There is a lot of weakness to operating in a bubble divorced from the actual process the dedicated experts engage in.

Karl
Reply to  MarkW
May 19, 2016 11:00 am

Willis says: ” wherein Steve McIntyre points out a number of its many faults”
and Willis also provides a link to his own article.
..
Willis, when are you going to learn that science isn’t done on BLOGS ???
….
Ever hear of “peer review?”
[Note: This “Karl” is a fake; an identity thief. Disregard. -mod]

Karl
Reply to  MarkW
May 19, 2016 11:01 am

[Deleted. Site pest/impostor. -mod]

Reply to  MarkW
May 19, 2016 12:04 pm

geoffmprice May 19, 2016 at 11:31 am

Thanks for the link Willis. Probably needless to say, your article is not particularly convincing. Per the norm, you sound like a non-expert picking out issues and drawing broad conclusions of incompetence without addressing or even cultivating input from the actual experts into your assessments.

In other words, you can’t find anything scientific to object to in my post, so instead you attack how I “sound” to you. Who cares how I sound to you? The only useful question is whether my claims are right or not … a question you have not begun to address.

As these things usually go, if the Pages 2K researchers actually took the time to argue with you on methodology in depth, the discussion would likely end up where they all generally do, right? Meaning, e.g., Goddard does a good job picking out some oddball adjustment in Paraguay and arguing that the temp record is all fraud etc. to wild applause in the anti-AGW bubble, but if you take the time you find out such arguments are (at least in my experience, uniformly) embarrassingly bunk. There is a lot of weakness to operating in a bubble divorced from the actual process the dedicated experts engage in.

See above. If you had found a real error in my work, you’d have jumped on it and trumpeted it from the rooftops. Since you can’t find any scientific errors, you wave your hands about Steve Goddard. What does Goddard have to do with me? That’s a pathetic puerile attempt at guilt by association.
Sorry, amigo, but you’ve just given us a classic fail, complete with the usual BS ad hominem attempt to throw mud at me and hope some sticks.
And when a man starts throwing mud, to me that’s clear evidence that he’s out of real ammunition …
I gave you my post on the hilarious variations in the data of Pages2K, along with 27 posts by Steve McIntyre pointing out any number of scientific problems with it. If you want to discuss any of the science in those 28 posts I’m happy to do so.
But you whining about how I sound to you? Sorry, but that goes nowhere.
w.

Johann Wundersamer
Reply to  MarkW
May 19, 2016 12:21 pm

” Pages2K is a joke.”

Argument by assertion?
..
FAIL

[Note: this was not written by Mr Wundersamer. It is a fake comment posted by an identity thief/site pest. -mod]

Reply to  MarkW
May 19, 2016 12:53 pm

Johann Wundersamer May 19, 2016 at 12:21 pm Edit

” Pages2K is a joke.”

Argument by assertion?
..
FAIL

http://rationalwiki.org/wiki/Argument_by_assertion

Johann, I gave no less that 28 citations to back up my statement that Pages2K is a joke … on what planet is a statement with 28 references mere “assertion”?
Nice try, though, it looked all reasonable … but only if you weren’t following the story.
w.

Paul Courtney
Reply to  geoffmprice
May 19, 2016 2:35 pm

Geoff: CO2 is ghg whose concentration can change independent of temp? So increase in CO2 concentration does NOT mean increase in temp? You sound like a denier! Anyway, we see below your fealty to Pages 2k and multi proxy stuff, are those the “observations” shown on your 2d chart? If Pages 2k and the multis are not on your 1970-present chart, why not? What observations are those, anyway? ‘Cause your chart doesn’t say. On your first chart, is that Marcot shooting up off the chart, way above instrument and everything else on the blade end of your hockey stick? Does that lend it credibility? It’s hard to tell, your charts seem to obscure things. I don’t imagine these comments will get you wondering, since you know so much about the inside of bubbles.

Reply to  Paul Courtney
May 20, 2016 9:11 am

Paul,
“CO2 is ghg whose concentration can change independent of temp? So increase in CO2 concentration does NOT mean increase in temp?”
No, the specific comment here is just that you can’t really change the H2O content of the atmosphere by boiling water and emitting H2O – it will rain out. Consequence of chemistry, thermodynamics of phase change. Atmospheric CO2 on the other hand can be increased in this way, as we’ve shown (having increased the global atmospheric concentration by a full 40%).
“If Pages 2k and the multis are not on your 1970-present chart, why not?”
Pages 2K is a paleoclimate reconstruction. I touched on different topics in my comment, you seem to be confusing them. The paleoclimate chart was a response to Ira’s (obviously wrong/meaningless) claim that warming is just what happens coming out of a glacial/ice age. The 1970-present chart on ocean warming was directed at Ira’s separate but similarly meaningless “models are all failures” comments.
“What observations are those, anyway?”
Cheng et al. 2014, Cheng and Zhu, 2014a/b, Institute of Atmospheric Physics (IAP) historical upper OHC assessment. From the usual ocean temp data: Argo, expendable bathythermograph. (The usual anti-AGW objection is to complain that the ocean is big and we can’t possibly have enough samples so it is purely an accident that the data we have matches what physics and other observations say must be happening… in case you’re not sure what you’re supposed to say in response.)
“On your first chart, is that Marcot shooting up off the chart, way above instrument and everything else on the blade end of your hockey stick?”
No, that’s thermometers i.e. modern instrumental data, as labeled. (Choose your index, they’ll be the same at this resolution.)
” I don’t imagine these comments will get you wondering, since you know so much about the inside of bubbles.”
I am rather curious by nature but no, your comments/questions have not happened to get me wondering about anything.

Paul Courtney
Reply to  Paul Courtney
May 21, 2016 8:06 am

Geoff, May 20-Well, your post leaves ME curious. Your specific comment still up there, no need for me to quote it, so I’m still curious-if CO2 can go up or down “independent” of temp., what are you warmers so alarmed about? And if I can’t change H2O concentration (I’ll go even further, you can’t change it either), what does change it? Or do you say it’s rock-steady constant? See how curious I am? I’m not confused (and neither are you) by your intentionally misleading charts, we here have seen modern instrument data not merely attached to paleo but used to blot out the tails of paleo. We can’t tell WHAT that line is jumping off the chart, doesn’t it look the same as the Marcott line? Have you looked at what those proxies have done in last 50 (or 150) yrs to see if they follow the instrument data? Or you’re not curious about that, either? So you choose ocean temps as “observations”. I can choose? OK, I choose UAH. Oops, looks like it’s below 99.5% of models. OK, I can choose others, even adjusted surface temp indexes are significantly below model mean (statistics-wise, another area of non-curiousity for you?). Dr. Tisdale’s posts leave me curious about that, not you, though. Folks who read blogs don’t need to be scientists to deconstruct your propaganda, our curiousity is not as limited as yours, evidently.

Reply to  Paul Courtney
May 21, 2016 10:07 am

“I’m still curious-if CO2 can go up or down “independent” of temp., what are you warmers so alarmed about?”
Once CO2 increases in this way, as we’ve experienced in the real world, the CO2 in the atmosphere acts as a greenhouse gas and causes warming. Good for you to ask (though this is pretty basic.)
Do you understand how the greenhouse effect works and what the general scale of it is, or are you in the camp that rejects conventional physics, spectroscopy and thermodynamics on this? Just trying to clarify what it is you are having issues with.
CO2 and temp affect each other, they are actually mutually reinforcing in the longer run under current types of conditions, but they are not essentially immediately tied together in the way H2O is. Emitting lots of CO2 into the air over time drives up the greenhouse effect and temperatures. Emitting lots of H2O in the same way does not. Do you accept this point or have an issue with it? Good for you to keep asking, but for the conversation to be productive you need to be able to reflect the points back, and then to frame your counter-arguments (whatever they are) in a more specific way.
“And if I can’t change H2O concentration (I’ll go even further, you can’t change it either), what does change it?”
Maybe re-read the exchange so far? The temperature of the atmosphere is what changes the concentration of H2O, per Clausius–Clapeyron. Warmer air means more water vapor content (specific humidity). You have likely experienced this directly – notice how warm temperatures near the ocean in the tropics lead to more humid weather. This is why water vapor acts as a massive feedback.
“So you choose ocean temps as “observations”. I can choose? OK, I choose UAH. Oops, looks like it’s below 99.5% of models.”
Sure, you can “choose” according to what you see as politically desirable. But I am “choosing” based on physical logic and a desire to understand what is happening in the climate. Ocean temperatures are the most significant physical observation because some 93% of new heat trapped by the global warming (anthropogenic greenhouse) effect is expected to accumulate in the ocean. This is a consequence of the ocean’s dominating thermal mass in the system – it can just hold a *lot* more heat content. Looking at the ocean always immediately falsifies the claims that AGW isn’t happening as predicted – ocean heat accumulation appears to be generally right on target (actually a bit hot according to Cheng et al) relative to expectations, per the data we have.
Now, you are picking measurements of the troposphere (e.g. 1000m above the ground) presumably because there is wide disagreement among the different methods of measuring temperature there and that gives you more to work with in terms of cherry picking. I assume you mean the unpublished UAHv6, which appears to incorporate stronger use of what Spencer previously called “the old NOAA-15 satellite which has a decaying orbit” and gets cooler trends as a result, showing up at the bottom of this sampling of troposphere measurements (by far the coolest of the active data sets going forward, as RSS 3.3 is being revised to *correct* for that satellite):
https://drive.google.com/file/d/0B_dL1shkWewaYUdhcjdFOFJ3ZTA/view
So, yes you’ve “chosen” the coolest measurement you could find, not because the troposphere is important to measure (it’s not – it holds a small amount of the system energy and is very noisy), but because that’s what you want to see. What significance do you attach to this choice, beyond the utility as a talking point?
Meanwhile I doubt UAH *today* (either version) is below 99.5% of models (assuming you mean model means?) We’re still in El Nino and monthly temp readings have generally been above model means for Feb-April.
“OK, I can choose others, even adjusted surface temp indexes are significantly below model mean (statistics-wise, another area of non-curiousity for you?)”
Well again, not today they’re not; you need to update your talking points for 2016. Here is a recent deeper discussion of surface temp vs. models:
https://tamino.wordpress.com/2016/05/17/models/
In general it is multi-decade trend that is interesting; as the IPCC reported, observations match projections to within about 0.02 deg C / decade even through “pause”. If PDO is switched positive good chance temps will run warmer than projections for a bit, just as they have run cooler in PDO-. If PDO is still negative, temps may drift below model means again.
The combination of the facts that the ocean is warming on target and surface observations today are warmer than model means makes a strong case that global warming is generally on track. Arguing about the details further is a pretty meaningless distraction (though I’m sure we’ll happily do so for another few decades of warming.)
Ask Dr. Tisdale to explain why the El Nino in 2016 hits a temperature peak about 0.4 degrees higher than a similarly sized El Nino in 1998, if there has been “no warming in 18 years”.

Paul Courtney
Reply to  Paul Courtney
May 21, 2016 3:00 pm

Geoff: So you do need it quoted? “Yet more significant to climate because it [co2] is the GHG whose [sic] concentration can change independent of temperature.” You seem to be the one who doesn’t understand your own theory. My issue is that you contradict yourself. The conversation became unproductive, but much funnier, when you entered it with your hockey stick and cherry-picked graphs and smug condescension. What we’ve seen in the real world is co2 up on a straight line while all measurements of temp have bounced up and down, falsifying your theory (but, ironically, agreeing with the quote above, which quoted…you! You’re half as right as a stopped clock!). Is h2o feedback only? Maybe h2o is also a forcing, massive both ways I agree, overwhelming any co2 warming? You chose Cheng’s line as “observations”, accusing me of doing what you’re doing, very progressive. You ask Dr. Tisdale, and I’ll ask Cheng how he’s got 2014 .1 higher than ’98 el nino (he’s got EVERY year hotter than ’98!), and find out if he agrees with your notion that IR captured by co2 (or water vapor) downwells to heat the ocean. Gotta go, you can have the last word if you like. Try to be consistent.

Reply to  Paul Courtney
May 21, 2016 3:31 pm

“So you do need it quoted? ”
No. But if you made an effort to understand the points of others, it would give you a chance to raise your own points with some degree of lucidity.
Yes, there are relationships between CO2 and temperature, but that relationship is quite different than that between H2O and temperature. It is a distinction in time – CO2 will affect temperature over time (and vice versa), but the concentration of H2O cannot be driven up independently at all. Unfortunately, it seems a point you are unable or unwilling to grasp (or attempt to argue against).
“My issue is that you contradict yourself.”
I do not, but I understand your confusion.
“What we’ve seen in the real world is co2 up on a straight line while all measurements of temp have bounced up and down, falsifying your theory”
Mainstream theory does not predict a “straight line” of temperature in response to a spike in GHGs. Paul, you are super far from basic understanding of climate science here in imagining that it does. (Yes, I realize you believe that sounds “smug” but that’s a side effect of talking about science. While nowadays everyone wants every opinion to be “equally valid”, even if they are just making it up as they go, that just isn’t how science works. I apologize for the unpleasant existence of empirically verifiable objective reality.)
The surface is affected in particular by large-scale ocean weather patterns like ENSO, which effectively change the rate of heat mixing between ocean and surface, in a cyclical manner. If you want a straighter line of warming in response to GHG increase, you can do one of two things.
(a) look at the continual rise in ocean heat content. The ocean is the big dog, the surface is more of a tail. Nothing wags the ocean (well, major eruptions) so it tends to just consistently accumulate heat content in a greenhouse warming regime.comment image
(b) plot the temperatures of El Nino, ENSO Neutral and La Nina years separately, so you take out the largest complicating cyclical factor in surface temps. Then you see that each of these types of years is warming in much more of a straight line. (This is also why you can’t address the question why the same strength El Nino in 2016 generates such higher temperatures than 1998.)
http://blog.chron.com/climateabyss/2012/04/about-the-lack-of-warming/
Let’s go ahead and leave it there.
“find out if he agrees with your notion that IR captured by co2 (or water vapor) downwells to heat the ocean”
For the record, downwelling IR (from greenhouse) acts more like insulation – the net flow of IR is from the ocean upward but the downwelling IR effectively slows that energy flow. This allows the *sun* to add heat to the ocean (as the ocean is not cooling itself off as fast). A common error there to forget the (rather large) role of the sun.
All of which brings us back to the top of the discussion so a fine place to leave things. Cheers.

Reply to  Paul Courtney
May 25, 2016 11:12 am

Paul Courtney says:
The conversation became unproductive, but much funnier, when you entered it with your hockey stick and cherry-picked graphs and smug condescension. What we’ve seen in the real world is co2 up on a straight line while all measurements of temp have bounced up and down, falsifying your theory (but, ironically, agreeing with the quote above, which quoted…you! You’re half as right as a stopped clock!).
True dat.

Reply to  geoffmprice
May 22, 2016 7:53 am

The modern warming spike is artificial, there is no planetary change in atmospheric conditions caused by humans, just deceitful humans trying to control human behaviour, that’s the story of political science.

Reply to  Sparks
May 22, 2016 8:48 am

“The modern warming spike is artificial, there is no planetary change in atmospheric conditions caused by humans, just deceitful humans trying to control human behaviour, that’s the story of political science”
I’m familiar with such conspiracy theories (obviously). It’s just not a view grounded in empirical reality. There is in fact a planetary change in atmospheric conditions – an unambiguous 40% increase in the global concentration of CO2, unprecedented on geological timescales. The greenhouse effect is real and directly observed and not disputed by even the most extreme anti-AGW partisans in science.
Yes, I hear that you are a greenhouse denier. I know this is still an active internet subculture. Sounds like you are from the “it’s all about pressure” splinter of that particular fringe camp. A planet with a dense atmosphere floating in the void of space with no sun shining on it will be dead cold, despite its high-pressure atmosphere supposedly doing work in your ‘alternative’ thermodynamic model.

Reply to  Sparks
May 22, 2016 11:15 am

geoffmprice,
I’m a what??
Is that how your twisted ideas gets accepted? you’re in the business of false affirmation, no one accepts your idea so you seek to belittle anyone who disagrees with you, nice try dipstick but you’re talking to me, and your comment is vile and disgusting on levels you couldn’t imagine, so f*k off with your fake conspiratorial nonsense, you moronic fool.
You believe that temperature is rising on a planetary scale, and the cause is humans, because of Carbon dioxide, you’re an Imbecile of the highest order among some pretty tough competition, well done you.

Editor
May 18, 2016 4:22 pm

Ira, thanks for the interesting analysis. I followed you up to the end, where you say:

So how much does all that melting amount to in terms of delayed temperature increase? 80 x 8000 / 1,300,000,000 = 0.000492⁰C, which we may round up to 0.0005⁰C (0.0009⁰F) of the warming since 1979, and even less of the missing warming during the “Pause”.

I don’t see how you can calculate the effect of that amount of cooling on the surface temperature. You are completely correct that the amount of energy is small in global terms. But the effect on the climate system is far less clear.
Here’s how I’d express it:
There are 213 gigatonnes / year of ice melt.
It takes 330 megajoules / tonne to melt ice, which means there are
7.0E+13 megajoules / year required to melt ice.
Now, we have 5.1E+14 square metres earth surface, which means that melting the ice requires
0.14 megajoules / year / square metre
We can convert this using the fact that 1 megajoule / year / square metre = 0.032 watts / square metre on a constant basis.
This means that the effect of the melting of the ice is
0.004 watts / square metre
By comparison, we have a global 24/7 average of 500 watts / square metre downwelling radiation at the surface … so the ice makes a change of 0.0009% in the surface radiation balance. As you say, it’s too small to matter in regards to the global energy flow.
My best to you,
w.

Reply to  Willis Eschenbach
May 18, 2016 5:19 pm

So why do most balances show 340 W/m^2 ToA?
Mankind’s alleged atmospheric CO2 power flux (watt is power, energy over time) increase between 1750 and 2011, 260 years, was 2 W/m^2 of radiative forcing. (IPCC AR5 Fig SPM.5) Incoming solar RF is 340 W/m^2, albedo RF reflects 100 W/m^2 +/- 30 (can’t be part of the 333), 160 W/m^2 reaches the surface (can’t be part of the 333), latent heat RF from the water cycle’s evaporation is 88 W/m2 +/- 8. Mankind’s 2 W/m^2 contribution is obviously trivial, lost in the natural fluctuations.
One popular GHE theory power flux balance (“Atmospheric Moisture…. Trenberth et. al. 2011 Figure 10) has a spontaneous perpetual loop (333 W/m^2) flowing from cold to hot violating three fundamental thermodynamic laws. (1. Spontaneous energy out of nowhere, 2) perpetual loop w/o work, 3) cold to hot w/o work, 4) doesn’t matter because what’s in the system stays in the system) Physics must be optional for “climate” science. What really counts is the net RF balance at ToA which 7 out of 8 re-analyses considered by the above cited paper concluded the atmosphere was cooling, not warming. Of course Trenberth says they are wrong because their results are not confirmed by the predicted warming, which hasn’t happened for twenty years.

Reply to  Nicholas Schroeder
May 18, 2016 5:55 pm

“Temperature drives pressure theory” nice!!

Reply to  Nicholas Schroeder
May 18, 2016 6:08 pm

Pressure differences drive temperature.

Reply to  Nicholas Schroeder
May 18, 2016 6:14 pm

Nicholas Schroeder May 18, 2016 at 5:19 pm

So why do most balances show 340 W/m^2 ToA?

Good question, Nicholas. The 340 W/m2 is TOA solar radiation. However, only about 170 W/m2 of this sunshine reaches the surface. There is also about 330 W/m2 of downwelling longwave radiation.
This gives a total downwelling radiation at the surface of about half a kilowatt per square metre as a 24/7 global average, although obviously anhy given area gets more or less.
w.

Reply to  Nicholas Schroeder
May 18, 2016 6:43 pm

“There is also about 330 W/m2 of downwelling longwave radiation.”
Per my comments above I consider this a violation of thermodynamics. Since all of the ebbs & flows are accounted for, this 330 W/m^2 has to appear out of nothing (strike 1) and cycle in a perpetual loop without additional work (strike 2) from cold to hot without additional work (strike 3). Some of the power flux balances I pulled off the internet don’t show this loop, i.e. failed consensus.
If you apply S-B to that down welling radiation at -40 F or C with an emissivity for atmospheric gasses/CO2 of 0.3128 the W/m^2 result is negligible. (Nasif S. Nahle – “Determination of the Total Emissivity…”
I know this ground has been plowed, horse has been beaten, but I’m not satisfied with the answer.
Also as I mentioned above what happens in the system stays in the system and all that counts is the balance at ToA. If more leaves than enters, cooling results. That’s what 7/8 studies in Trenberth’s 2011 Figure 10 showed. And CO2’s 2 W/m^2 is trivial. That balance is controlled far more by albedo and other stores/releases than CO2.

Reply to  Nicholas Schroeder
May 18, 2016 8:49 pm

Nicholas Schroeder May 18, 2016 at 6:43 pm

“There is also about 330 W/m2 of downwelling longwave radiation.”

Per my comments above I consider this a violation of thermodynamics.

Fortunately, thermodynamics is not bound by your opinion. Scientists regularly use instruments to accurately measure the amount of downwelling radiation from locations all over the planet. You can explain that measured radiation in different ways, but waving your hands won’t make the scientists and their measurements suddenly disappear. The radiation is measurably there, regardless of your beliefs.
In other words, you get to have your own theories, but not your own facts.

Since all of the ebbs & flows are accounted for, this 330 W/m^2 has to appear out of nothing (strike 1) and cycle in a perpetual loop without additional work (strike 2) from cold to hot without additional work (strike 3). Some of the power flux balances I pulled off the internet don’t show this loop, i.e. failed consensus.

Here are two of my posts that show how the poorly-named “greenhouse effect” works. There is no violation of the second law, all heat flows from warmer to cooler. Turns out you don’t need CO2 or even an atmosphere. Check it out:
The Steel Greenhouse
People Living in Glass Planets
My best to you,
w.

Reply to  Nicholas Schroeder
May 19, 2016 8:08 am

IMHO this popular GHG perpetual motion heat loop contradicts hundreds of years of thermodynamic theory. Extraordinary claims require extraordinary evidence. If CO2 & GHGs have such impressive thermal properties, greenhouse operators would increase the CO2 concentrations to save on their heating bill (no evidence of that) as well as improve crop yield (plenty of evidence of that).
I’ve used IR thermometers and cameras in industrial settings. There is more to an IR instrument than point, read, and figure S-B. My impression is that the instruments are being improperly applied and interpreted.
The S-B equation including grey body emissivity results in negligible radiation returning from the inside of the shell. The energy available for radiation is what’s left over after T+A+R = 1, .e.g. for GHGs transmission 95% / absorption 5% / radiation 4%. Since most of the LWIR leaving the earth’s surfaces passes right through the GHGs there is little absorbed, little radiated and a resulting low emissivity. This shell concept is as flawed as the GHE & blanket. As I mentioned elsewhere removing this 330 W/m^2 loop changes nothing at the ToA.
I think a better analogy is the walls of a house. If I increase the insulation on the house the rate of heat loss decrease per Q = U * A * ΔT. The furnace firing rate also decreases as does my monthly bill. However, if the furnace fires at a fixed rate (ASR) and U goes down ΔT must increase to move that same Q through the thicker insulation. So what to do? Well, open a window or door, turn on the air conditioner, etc. There is not a thermostat on the sun, but there is certainly one on the atmosphere: water vapor. (See Jo Nova’s handbooks)
Disputing this GHG loop concept is not denying the greenhouse effect/principle/process. A greenhouse operator can increase thermal mass by installing boxes of rocks, trombe walls, black painted plastic tubes and barrels full of water or eutectic salts, aka the oceans.
If it gets too hot the operator can pull down reflective shades reducing the incoming heat, aka albedo which is more than just clouds. BTW IPCC AR5 credits clouds with -20 W/m^2 of RF and that’s a lot more cooling than CO2’s 2 W/m^2 of heating.
The operator can turn on misting water sprays and evaporative cooling to reduce the air temperature and raise relative humidity, aka storms, rain, snow, etc.
Both IPCC and Trenberth (same as IPCC) admit they really don’t understand the water vapor cycle, clouds, etc. very well, IPCC AR5 in TS.6 and Trenberth in the papers mentioned earlier.
1) The CO2 increase between 1750 and 2011 amounts to a 0.5% fluctuation in the overall global carbon balance. When the uncertainty in the origins of the carbon balance is +/- 850 Gt, 1,750 Gt band, it’s strains belief that 0.5% can be measured let alone attributed to any particular source. C13/C12 changes over 261 years have to be detected at ppm levels. Pretty unlikely.
2) At 2 W/m^2 RF CO2’s role in the greenhouse process is insignificant.

Reply to  Nicholas Schroeder
May 19, 2016 10:15 am

Nicholas Schroeder May 19, 2016 at 8:08 am

IMHO this popular GHG perpetual motion heat loop contradicts hundreds of years of thermodynamic theory.

In your humble opinion? Reminds me of Churchill, who allegedly said of someone:

He is a humble man … and he has much to be humble about.

I gave you two good clear explanations of how the so-called “greenhouse effect” works, including why it doesn’t break any laws of thermodynamics. Instead of coming back and telling me exactly where my explanation is incorrect, you’ve come back with your “humble opinion”.
Pass … you can lead a horse to water, but you can’t make him think.
w.

Reply to  Nicholas Schroeder
May 19, 2016 10:43 am

http://claesjohnson.blogspot.com/2011/08/who-invented-downwelling-longwave.html
“We find support to our claim that DLR and backradiation is fictional without physical reality, and thus that an essential component of the greenhouse effect is fiction and not physics.
DLR thus appears to be a recent man-made invention, and so the greenhouse effect of CO2 alarmism.
It is remarkable that several governmental agencies base their whole existence on measuring DLR/backradiation, which may not exist. This shows the strength of tax-financed science which can freely invent to measure anything, fiction or reality does not matter as long as numbers, tables and movies are produced according to the principle:
•the more fictional
•the more difficult to measure
•the bigger institute.”

FTOP_T
Reply to  Nicholas Schroeder
May 21, 2016 8:01 am

@Nicholas
I share your frustration with this fictitious 330 W/m2 of heat transfer from the atmosphere to the surface. This would require the earth’s surface to serve as a heat sink for a cold atmosphere.
How can space and the surface be a heat sink for the atmosphere which has no heat source?
How much colder is the atmosphere after it warms the planet below?
These posts by Roy Clark do a fantastic job of laying bare the fallacies of the Trenberth energy budget.
Climastrology is inventing the 330 DWLIR figure as a fudge against an average thereby avoiding the importance of time and latency.
https://tallbloke.wordpress.com/2012/03/21/roy-clark-where-it-all-went-wrong-with-climate-science/
http://www.venturaphotonics.com/ClimateFraud.html
Someday this GHG theory is going to be read its last rights.

FTOP_T
Reply to  Nicholas Schroeder
May 21, 2016 6:32 pm

@Willis
Over 1000+ Wm2 reaches the surface in the tropics, penetrates at depth, and it hits a substance that is extremely adept at retaining that heat. The TOA releases 240 Wm2 across the sphere, but 170 Wm2 for average reaching the surface simplifies a dynamic process in a non-physical way.

Reply to  Ira Glickstein, PhD
May 18, 2016 9:17 pm

Ira, many thanks for your kind comments about my writings. My target audience for my scientific work is the person I call the “literate layman”. To me this means someone who is educated but not necessarily in science, who may not be real comfortable with math, and who is curious about the world and its mysteries.
Climate science, by its very nature, requires a different writing style than other disciplines. This is because there are at least six complete and separate sub-systems to the climate: atmosphere, hydrosphere (ocean, lakes, and rivers), lithosphere, cryosphere, biosphere, and electrosphere. So it’s not like say some part of physics, where everyone involved is a physicist and speaks physics. In climate science, we have oceanographers and atmospheric chemists and tree-ring specialists and remote-sensing experts … so you can throw away all the specialty stuff, only one person in ten would understand it.
Next, I make up rules of thumb, they are valuable guidelines. One of my rules of thumb is that every number I put in my text costs me a reader. Seriously. Many people are quite math-averse. So I try to tell the story in such a way that the math is kind of secondary. Don’t get me wrong, the math is always central and critical. I just work at telling a story that is supported by the math, rather than the usual scientific method of having math tell the story.
Finally, to structure my documents, I think about the elevator speech and the matchbook speech. The elevator speech is the speech you’d give if you had the length of an elevator ride to convince the hearer of your claims. You get in on the first floor with someone, and you have until the 25th floor to explain your ideas. You need to boil your ideas down, over and over, until they fit into a few paragraphs.
The matchbook speech is the same, but you need to be able to write it on the cover of a matchbook. It’s tougher, instead of paragraphs you need to boil your ideas down to a few sentences.
Once I’ve done that, boiled my ideas down, the rest is pretty easy. The one caveat I’d put on it is that I’m incredibly fortunate in that I’m free of deadlines, or any “publish or perish” pressure. So I can apply another of my rules of thumb, which is to only write about something when I can’t stand not writing about it …
Regards,
w.

Reply to  Willis Eschenbach
May 20, 2016 10:31 am

To respond to your comments above Willis:
“In other words, you can’t find anything scientific to object to in my post”
Oh, the post is unimpressive right from the opening. Your main point seems to be to dispute Evans’ claim that “Today, the Earth is warming about 20 times faster than it cooled during the past 1,800 years”. You look for this in the older paleo data, which seems ludicrous: “The change from 1700 to 1900 is not anywhere near 20 times as steep as the drop from the start of the study to the present”. Evans is clearly comparing *modern* warming (last century) to the warming of the past 1800 years. The data unambiguously supports his comment. That you think he means the proxy data from 1700-1900 is your strange non sequitur. He said “today”, by what strange perspective is 1700-1900 “today”?
Your next point is that you think the data for the Indian ocean looks flat, and that this also contradicts the claim. Yet the oceans are obviously warming today, quite steep relative to that flat trend from previous millennia. These comments don’t seem to make sense, and as a result this doesn’t encourage me to dig a lot deeper into the post.
Understand the cultural gap here: I’m a skeptic, used to scientific method. “Blog auditing” as popularized by gentlemen like Mcintyre (http://www.shapingtomorrowsworld.org/news.php?p=5&t=209&&n=159#674) is not a normal part of the scientific process. For clear/valid reasons. It’s not easy to parachute into complex areas of science, pick some bits out and claim you’ve shown it’s all nonsense. Much, much easier to *convince* yourself you’ve done this. Indeed, the custom in blog auditing is to resist checking your criticisms against the actual experts, and to actually spin this as a virtue (of independent thinking)! This is fundamentally pretty alien to the normal scientific process.
If you are honest, you could concede that by *far* the most common result of such ‘blog auditing’ is that the author just flatly fails to understand something in published work. In rare cases, the blog auditor is successfully nitpicking something that is genuinely a weak point of some kind, but wildly exaggerating the significance of the issue. Now, is there a set of cases where the blog auditor really has proven that widely scrutinized and endorsed science like Pages 2K is “garbage”? I’ve dug deep into many such claims over the years, often doing the homework that you don’t seem to want to do yourself (i.e. contact the dedicated experts and ask them what they think about the criticisms) and the results have not been pretty, to put it mildly. I can’t say I can name a case where a blog auditor has truly shown that some major emperor of mainstream science “has no clothes”.
If blog auditors such as yourself were tearing giant holes in prevailing scientific knowledge as you claim, I still believe you would fare much better in publishing these criticisms. I know you might disagree, claiming there is suppression/conspiracy against contrary views. But you only have to look at the most celebrated cases like Wahl & Ammann 2007 (and the endless subsequent reproduction of MBH general results) for an example of how Mcintyre type auditing tends to fare on the normal playing field of peer review.
Cheers, again genuinely appreciate the link and that you’ve made an effort to engage with real data.

Michael 2
Reply to  geoffmprice
May 20, 2016 11:38 am

geoffmprice says “claiming there is suppression/conspiracy against contrary views.”
The RICO 20 and Shukla’s letters seem to fall squarely into that category. Obviously it is no longer a conspiracy since it is no longer secret.

Reply to  Michael 2
May 20, 2016 1:09 pm

Michael 2 writes “The RICO 20 and Shukla’s letters seem to fall squarely into that category”
No, that doesn’t fall squarely into the category of suppression of peer review research. RICO can only apply to an organization knowingly defrauding the public, customers or regulators in some way, which is clearly the intent of the letter you’re referring to.

Michael 2
Reply to  geoffmprice
May 20, 2016 1:52 pm

geoffmprice “No, that doesn’t fall squarely into the category of suppression of peer review research.”
Say what? When did this conversation suddenly become about suppression of peer review research (research into peer review?)
“RICO can only apply to an organization knowingly defrauding the public, customers or regulators in some way, which is clearly the intent of the letter you’re referring to.”
Yes, exactly. In this case the 20 signatories of the Shukla letter and more recently the cabal of state attorneys general. RICO may well be appropriate, but against the originators of this adventure.
Your earlier comment, since you seem to have forgotten it, was “I know you might disagree, claiming there is suppression/conspiracy against contrary views.”
Let me emphasize the object: contrary views. What might those be? Certainly not the 97 percent! How can 97 percent be contrary? Suppression of “contrary views” must necessarily therefore be any of several minority views, namely, that global warming is not necessarily dangerous, not entirely human caused, and/or maybe doesn’t exist outside of normal climate variation.
The list of attempts to silence skeptics is enormous. In the following, notice the frequent hatred and suggestions of violence that the TBW’s (True Believing Warmists) have for skeptics.
Some contrary views being suppressed by organizations meeting, sometimes secretly, for this purpose:
Dem AG Targets 90 Conservative Groups in Climate Change Racketeering Suit. [http]://freebeacon.com/issues/dem-ag-targets-90-conservative-groups-climate-change-racketeering-suit/
Quote: Walker’s subpoena is part of a coordinated effort by Democratic attorneys general spearheaded by New York’s Eric Schneiderman and undertaken in consultation and cooperation with leading environmental advocacy groups. The participants huddled at a Jan. 8 meeting at the headquarters of the Rockefeller Family Fund, a left-wing foundation. At the meeting, RFF, Greenpeace, other environmental groups discussed ways to “delegitimize [ExxonMobil] as a political actor,” “force officials to disassociate themselves from Exxon,” and “drive divestment from Exxon,” according to a copy of the meeting agenda obtained by the Washington Free Beacon last month.
Unquote.
Bill Nye: Throw ‘climate deniers’ in jail [http]://redalertpolitics.com/2016/04/14/bill-nye-throw-climate-deniers-jail-video/
October 15, 2015 In UK, Judges Propose Making It Illegal to Utter Any Statement Denying Global Warming
[http]://ace.mu.nu/archives/359537.php “‘The most important thing the courts could do,’ he said, was to hold a top-level finding of fact, to settle these scientific disputes once and for all: so that it could then be made illegal for any government, corporation (or presumably individual scientist) ever to question the agreed science again.”
Dem. Congressman on Witch Hunt Against Climate Scientists. “Roger Pielke Jr. … reveals that he was one of seven academics being being investigated by Rep. Raul Grijalva, the ranking Democrat on the House Committee on Natural Resources.”
[http]://www.weeklystandard.com/article/rep-raul-grijalvas-shameful-witch-hunt-against-climate-scientists/866050
“Daily Kos Editor Says Skeptics Should Commit Suicide” [http]://canadafreepress.com/article/daily-kos-editor-says-skeptics-should-commit-suicide “Former Washington Post reporter David Weigel was recently fired from the paper for privately writing on the Journolist listserv that Matt Drudge should ‘… set himself on fire.’ “

Editor
May 18, 2016 4:41 pm

geoffmprice – your first graph is dodgy, because it splices higher-resolution data onto lower-resolution data. IOW, the pattern shown in the higher-resolution data may well have occurred in the past, but the past data is too low-resolution to be able to show it. Perhaps a better graph would be this one …
http://climate4you.com/images/GISP2%20TemperatureSince10700%20BP%20with%20CO2%20from%20EPICA%20DomeC.gif
… but you still can’t ‘legally’ splice the last few decades onto it.
PS. The “multi-proxy” reconstruction from Marcott et al, complete with “hockey-stick”, as shown in your graph – could be bogus:-
http://www.c3headlines.com/2013/03/the-marcott-science-hockey-stick-blade-fiasco-audit-mcintyre-bogus-data-manipulation.html
The Marcott-‘Science’ Hockey Stick Fiasco: Audit of New Study Proves Bogus Data Manipulation

Reply to  Mike Jonas
May 18, 2016 5:33 pm

Yes, that is a single ice core graph. It is clearly not a ‘better graph’ for purposes here, being a single type of proxy from a single location (topic is global warming), though it is clearly pretty consistent with the graph I linked. Yes, it is fine to highlight resolution differences in the older data but that’s not the topic, the topic is Ira’s claim.
No, it’s not very likely that planet-wide energy changes of the magnitude currently being seen are hiding in the ~300 year seams of the older Marcott reconstructions, but sure, you can cling to that. We know about Dryas type events that seem to be circulation related.
The Pages 2K portion (which reproduces both Marcott, original MBH and all the others) has higher resolution (~30 years?), and that segment is sufficient to reject Ira’s claim that current warming is ‘just what happens coming out of an ice age’. Why would you defend such claims? You appear to know better.
I expect the breathless blog expose of fraudulent data manipulation is about as reliable as all the other blog pages claiming climate data is a result of conspiracy, despite results like this continually getting reproduced by independent international efforts, and the fact that individual claims of “fraudulent data” have continually turned up nothing for a couple of decades, certainly including the original hockey stick controversy.
“Hockey stick” just comes from two facts clearly and redundantly shown in data/evidence: general flattish, slow cooling of Holocene past ~6K years followed by a sharp modern warming spike, which we’ve directly measured with thermometers and satellites. But again that’s not the topic here.

Paul Courtney
Reply to  geoffmprice
May 19, 2016 4:34 pm

Relying on Pages 2k, Marcott and MBHahaha instead of one ice core that has not been debunked; tying instrument record apples onto (debunked) multi proxy oranges; bad-mouthing WE’s one article while ignoring his 27 others. Smells like teen CliSci, fer sure.

Toneb
Reply to  Mike Jonas
May 19, 2016 12:17 am

Oh, now there’s a surprise.
Someone posting the Alley graph of GISP2 ice-core proxy temp data that shows the end date as present – when in fact it is 1855.
That this is so is stated on this very website …
https://wattsupwiththat.com/2013/04/13/crowdsourcing-the-wuwt-paleoclimate-reference-page-disputed-graphs-alley-2000/
Also:
The surrent atmospheric concentration of CO2 is ~400ppm.
That graph shows it at pre-industrial levels of ~280ppm
This is how those graphs (no attribution but they are from the climate4you website) would look if updated to present:
http://www.skepticalscience.com/pics/c4u-chart5.png

Christopher Hanley
Reply to  Toneb
May 19, 2016 1:41 am

Oh lord not again.
You can’t legitimately take a highly smoothed time-series graph “to present”.
If there was enough data for the GISP2 ice core proxy graph to be at the equivalent resolution of say HadCRUT3 it would look a lot busier with much higher peaks and lower troughs.
Adding the CO2 spike simply illustrates how ineffective CO2 has been as a temperature forcing factor.

Toneb
Reply to  Toneb
May 19, 2016 3:50 am

“Adding the CO2 spike simply illustrates how ineffective CO2 has been as a temperature forcing factor.”
No it (and it was meant to) simply illustrates that *sceptic* mythology continues to have a life of it’s own once created.
Also, no, one wouldn’t expect that that graph would “illustrate how ineffective …..”.
Anthro CO2 has only had a forcing effect since around 1960 (outweighed by aerosol until then) and this is a single site on a graph covering millennia with many local variations likely to have occurred in wind/ocean circulation.
Putting it another way:
So you’d expect to climatic swings in temperature ~3C to be reflected elsewhere on the globe? and for it to be representative of a global climatic shift even?
Precisely – and neither would anyone expect the last ~50 years of warming to be evident there, given the over-riding climatic variability evident in the extreme location.

Reply to  Toneb
May 19, 2016 11:34 am

Toneb says:
Someone posting the Alley graph of GISP2 ice-core proxy temp data that shows the end date as present – when in fact it is 1855.
OK then, here’s a chart with a very recent splice:
http://oi53.tinypic.com/sg2wav.jpg
As we see, current temperatures are still far below those earlier global warming events — which all occurred when CO2 was much lower, thus debunking the alarmnist crowd’s CO2=cAGW nonsense.

Toneb
Reply to  Toneb
May 20, 2016 12:16 am

“As we see, current temperatures are still far below those earlier global warming events — which all occurred when CO2 was much lower, thus debunking the alarmnist crowd’s CO2=cAGW nonsense.”
If you stay so db…
Then it must be true.
Never mind the worlds experts eh?
So, you would accept that there have been variations in temperature throughout the the Earth of the order of 3C over a few hundred years?
Where is the evidence of this other than at this one (immensely extreme location).
Who climate has a as THE major influence the ocean currents and wind regime.
Think AMOC and NAO.
But then that would be science eh db?
And god forbid that the experts know more than you.
After all you get yours from *sceptic* blogs don’t you?
The font of all scientific truth. (sarc)

Reply to  Mike Jonas
May 19, 2016 6:00 am

your first graph is dodgy, because it splices higher-resolution data onto lower-resolution data. IOW, the pattern shown in the higher-resolution data may well have occurred in the past, but the past data is too low-resolution to be able to show it. Perhaps a better graph would be this one …
… but you still can’t ‘legally’ splice the last few decades onto it.

Well the last 16 decades anyway since the last ~160 years are missing. In the vicinity of that ice core the temperature has increased by ~1.5ºC during that time. The most recent GISP2 data has a time resolution of ~6 years.

Reply to  Mike Jonas
May 19, 2016 10:22 am

Mike Jonas,
Personally, all I need is to see this bogus chart to know that Marcott et. al is trying to sell us a pig in a poke with this splice:
http://www.realclimate.org/images//Marcott.png
That’s not even a hokey stick. That’s a 3-stage atomic powered skyrocket!

Reply to  dbstealey
May 19, 2016 10:44 am

Also, splicing on recent temperatures (as recently as 2009, per the graph) the current global warming still doesn’t come close to natural global warming events during the Holocene:
http://oi53.tinypic.com/sg2wav.jpg
Since past global warming events happened before human CO2 emissions could have had any effect, what is the explanation for those obviously natural warming episodes?

john harmsworth
Reply to  dbstealey
May 19, 2016 12:29 pm

8000 years of exactitude around a variance that hit .4 degrees at the most convenient time to make the point! A degree of certainty that could land a lesser person in the loony bin for delusions of grandeur! I’ve got an idea! Let’s wreck the world’s economy based on this piece of artwork. The hubris is stunning!

Robert of Texas
May 18, 2016 5:09 pm

LOL, they got to get this guy and Bill Nye together! Now the earth is like a glass of cold water! Maybe if they bubble heated CO2 through the water under a heat lamp they can prove it warms faster and make a video of it for school children.
There are times when the argument becomes so STUPID you just have to laugh it off.

Reply to  Robert of Texas
May 18, 2016 6:18 pm

lmao

NW sage
May 18, 2016 5:26 pm

Ira – I appreciate the analysis. You express concern over the fact that so many people seem to accept without question the premise that global warming is/must be human caused. I believe the following to be true: The human ego is strong and large and subject to being self inflated. Many many otherwise very intelligent citizens are told often, and therefore believe, that they (we) can do ANYTHING! This seems to include materially changing the earth/solar system/universe. Our popular entertainment and media constantly reinforce this theme (sci-fi is really good at this). Very very few outside the engineering field have any appreciation have any idea just how much ENERGY substantial changes need and how difficult it is to obtain and control that energy.
Until that fact is understood at a fundamental level the phrase “we can do ANYTHING” will continue to be widely believed.
NO ONE lies to hear ‘NO YOU CAN’T but it is true. We really cannot change the weather or the climate.

Reply to  NW sage
May 18, 2016 6:54 pm

People have been changing the weather for hundreds of thousands of years, it’s true, people have been around throughout Ice ages and whatever else they observed and survived through, they changed the weather, they burned witches alive and committed genocide, people have ripped out and held freshly beating hearts to appease one thing or another, why not do a rain dance? the pattern of utter ignorance and stupidity has left a trail throughout history, why on earth would anyone suspect that cats ‘n dogs are causing man made something or other?
They don’t, people are liars, criminals and opportunistic to the point of murder, there is nothing more frightening on earth than a person. (except spiders, wriggly things, the unknown and death) Always remember that!

Asp
May 18, 2016 6:17 pm

If we agree that looking at the ice caps melt is more indicative of global warming than measuring temperature, are we not then saying that the climate models some of us worship are a total waste of time?

john harmsworth
Reply to  Asp
May 19, 2016 12:32 pm

Send half the climate scientists to the Arctic to measure that ice. Send the other half to the Antarctic to measure that ice. Pick them up in ten years and compare. then swap pole for pole and try again.

philincalifornia
May 18, 2016 7:22 pm

“HOWEVER, warming is mostly natural and due to Earth’s recovery from the depths of the last ice age, some 18,000 years ago.”
This isn’t correct Ira. We’re in an overall cooling phase since the Holocene Climate Optimum, prior to plunging into something that will likely be enormously disruptive for humanity.
I think you meant “the little ice age”.

JohnWho
May 18, 2016 7:23 pm

“Ingenious or Misleading Rationale for the “Pause”?”
How about a third choice – “absurd”?

Evan Jones
Editor
May 18, 2016 7:44 pm

There has been ice all along. So why hasn’t there been a pause all along? Seems like a fundamental flaw to me.

May 18, 2016 8:36 pm

“Yes, the Atmospheric “Greenhouse” effect is real. It is responsible for the Earth being about 33⁰C (60⁰F) warmer than it would be absent “Greenhouse” gasses in the Atmosphere.”
Not exactly. This is the case between the LTE EQUIVALENT 255K emission temperature of the planet and its 285K average surface temperature. However, the same GHG’s that cause this warming are also causing cooling via reflection from ice and clouds. As I consistently try to point out, the effects of water vapor extend far beyond its nature as a GHG.
The planets intrinsic albedo (minus ice and clouds) is closer to 0.1, thus the incident energy and emissions would be 307 W/m^2 corresponding to a temperature of about 271K. So while the GHG’s increase 255K up to 285K, the influence of ice and clouds consequential to these same GHG’s reduces 271K to 255K. The NET warming from GHG’s (which must include water vapor) is only 14C and less than half of what is consensually stated.

Michael D Smith
Reply to  co2isnotevil
May 19, 2016 10:22 am

YES! You’re the 2nd person I’ve seen (besides me) point this out. Most of the 0.31 albedo is due to white clouds, which are a reaction to incoming solar and the presence of water – part of the feedback. I used 0.15 as the base, to include some haze, which makes the GHE about 20°C, not 33°C. A very important, but conveniently ignored concept! Conventional climate science is backwards. Albedo is not fixed, it rises with temperature, causing cooling, and declines with cooling, causing warming. Its level is determined by the properties of water, which aren´t going to change anytime soon. More warmth below (especially oceans), more clouds. Less warmth, fewer clouds. This is the main driver of the fact that actual sensitivity as measured by ocean warming is only about 0.25 * the expected 1.2°C per doubling.

Reply to  Michael D Smith
May 19, 2016 10:51 am

Michael,
“More warmth below (especially oceans), more cloud.”
It’s not this simple. More warmth means more water vapor which means more clouds, but what’s increasing from 0C to 30C is the cloud volume, not the amount of surface covered by clouds. It’s not until evaporation starts to increase exponentially above about 30C that cloud area starts to increase again. At the 0C isotherm where snow/ice is ebbing and flowing, the sensitivity is significantly larger and often incorrectly extrapolated to the rest of the planet.
I say increases again, because below 0C, the surface area covered by clouds also increases as the temperature increase. The reason for the inflection point at 0C is that clouds reflect the same as surface ice and snow, so the cooling (often incorrectly referred to as negative feedback) from increased reflection is absent. Interestingly enough, the inflection point at 30C is also a consequence of a negative feedback like effect. This time it’s caused when incremental evaporative cooling completely offsets the heat arising from incremental solar input and contributes to establishing a maximum temperature for the surface of the oceans before Hurricanes can form owing to massive amounts of atmospheric water vapor.
This plot show this behavior in no uncertain terms.
http://www.palisad.com/co2/sens/st_ca.png
Many more plots demonstrating this, along with an explanation of what’s being plotted and plots of the sensitivity of many climate attributes to various others can be found here:
http://www.palisad.com/co2/sens

Greg K
May 18, 2016 8:40 pm

We’ve been bouncing around in a cooling trend for about 35 million years, since Antarctica got stuck down at the South Pole. The current warm phase has lasted about 12,000 years, with warmer and cooler periods, and I think it is very pleasant but it is unlikely to last.
Geological and glaciological history suggests we should know where we have stored our coats

May 18, 2016 9:04 pm

Another fine contribution by Ira Glickstein, PhD. Many thanks!

May 18, 2016 10:35 pm

Ira says, “The most likely explanation is that their climate theory is either incomplete or totally wrong, so their models failed.”
This is consistent with Ray Bätes’s claim in a recent paper.
Quote, “A major current issue in climate science is that, despite the considerable progress that has been made in the development of global climate models (GCMs), it has not proven possible to narrow the uncertainty limits of 2 × CO2 equilibrium climate sensitivity since the Charney Report [1979]. Recently, it has been seen necessary to widen these limits, with Intergovernmental Panel on Climate Change (IPCC) [2013] giving a probable range of (1.5, 4.5)°C as compared with the IPCC [2007] range of (2, 4.5)°C.

The central conclusion of this study is that to disregard the low values of effective climate sensitivity (˜1°C) given by observations on the grounds that they do not agree with the larger values of equilibrium, or effective, climate sensitivity given by GCMs, while the GCMs themselves do not properly represent the observed value of the tropical radiative response coefficient, is a standpoint that needs to be reconsidered.”
Source: J. Ray Bätes, Estimating Climate Sensitivity Using Two-zone Energy Balance Models, 2015.
http://onlinelibrary.wiley.com/doi/10.1002/2015EA000154/pdf
Comment: As I understand it, the reason for the lack of convergence among models is that modelers cannot agree on the role of clouds, specifically the feedbacks.
However, the role of clouds in modulating albedo (reflectance) may also be underestimated by GCMs. (I cannot locate the reference, but I recall a critical claim that the GCMs all use the same value for Earth’s albedo, fixed around 0.30.)
Pallé et al, in a series of studies have shown that this assumption is not tenable.
Source: Pallé, E., et al. “Earth’s albedo variations 1998–2014 as measured from ground‐based earthshine observations.” Geophysical Research Letters (2016).
http://arxiv.org/pdf/1604.05880.pdf
While I maintain a skeptical view of the claims of cosmo-climatologists, their focus upon the cloud-albedo relationship seems to me to be so much to the point that we should keep up to date with developments in this countervailing theory of climate change.

May 19, 2016 1:10 am

If earth emitted as much as it absorbed, there would be flipping life on the planet

Reply to  Mark - Helsinki
May 19, 2016 8:30 am

Mark,
There would be NO life on the surface since photosynthesis converts some of the incident energy into the chemical energy in sugar, upon which all other life depends (excluding life near deep ocean vents).

charles nelson
May 19, 2016 1:17 am

I have repeatedly pointed out that the ‘warming’ of the Arctic this February was in fact a massive ‘cooling event’. Given that the area was in darkness at the time, the only way heat could have been transported there was via water vapour (from El Nino).
What happens when you introduce water vapour into an environment where the temperature is minus 30˚C…?
The rather cold, horrid summer they’re currently having in Europe is undoubtedly the result of all that ‘warming’ they had back in Feb.
Enthalpy dear boy…enthalpy.

Steve R
May 19, 2016 4:55 am

If this is a new point to be made, it leaves me a bit confused. Why else then has everyone been so worked up about the loss, or not, of polar ice?

Reply to  Steve R
May 19, 2016 5:37 am

Who’s everyone?

Reply to  Steve R
May 19, 2016 9:58 am

Loss of polar ice is concerning for a few reasons. It’s a positive albedo feedback for one. It is also a major change to the nature of the arctic that has wide-ranging effects on weather and ocean patterns (e.g. weakening of the AMOC appears to have begun, though that is more due to the accelerating melt of ice off the land.)

Reply to  Ira Glickstein, PhD
May 20, 2016 6:19 am

Have you seen this Ira?
..
http://edition.cnn.com/2016/05/20/asia/india-record-temperature/
[added by Ira 21 May. Yes, Betty. Apparently a new record temperature in India of 51 C. The previous India record was 50.6 C, and that was in 1956. So, since 1956 the peak India temperature has increased a whopping 0.4 C (assuming the thermometers, measuring conditions, urban heat islands, time of measurement, etc. are EXACTLY identical. If they differ, as they most likely do, by tenths of a degree, the actual warming might be 0.2 C or 0.6 C.)
However, since 1979, according to satellite data, the Global average has increased by at least 0.3 C and possibly as much as 0.6 C. Since 1956 the Global increase may be larger still.
Thus, the new India record is totally in accord with expectations, or a bit less than expected, based on satellite data. If the average of the 102 IPCC models were correct, we’d expect an increase of at least 0.6 C and possibly as much as 1.4 C since 1979, and even more since 1956.
Therefore, the new India record temperatures are somewhat disappointing, and do not support alarming predictions of a comping climate catastrophe.]

Reply to  Ira Glickstein, PhD
May 20, 2016 12:47 pm

I suppose this is an enjoyable thing to believe from an ideological perspective. In reality, shrinking/removing the planet’s arctic ice cap over time is a pretty significant alteration to the planet. There’s no getting around the fact that it is a fairly remarkable thing for humans to be doing, as prevailing understanding and evidence seems to indicate (acknowledging that there are significant weather/circulation components here as well.)
[added by Ira 21 May. geoffmprice: Based on the NSDIC data I cited, we have lost about 1/20000 th of the total Global Ice since 1979. I do NOT consider this to be what you call “a pretty significant alteration to the planet”. And, most of that ice loss is likely due to natural rather than human-causation.]

Michael 2
Reply to  geoffmprice
May 20, 2016 1:08 pm

geoffmprice wrote “There’s no getting around the fact that it is a fairly remarkable thing for humans to be doing”
Indeed. I am delighted with myself. Any time I need a boost in my ego I need only think of this vast power I wield: Melting icecaps and making new farmland. I am my finger of heat and drought at my enemies!
Doubters will exist questioning that I had anything to do with icecaps melting but who can prove otherwise?

Matthijs
May 19, 2016 7:07 am

“Yes, the Atmospheric “Greenhouse” effect is real. It is responsible for the Earth being about 33⁰C (60⁰F) warmer than it would be absent “Greenhouse” gasses in the Atmosphere.”
If this theory is correct, the models are correct.

Reply to  Ira Glickstein, PhD
May 19, 2016 12:47 pm

Ira,
You said,
‘2) the absorbed energy causes these gasses to warm up (and by conduction warm the oxygen and nitrogen that make up nearly all of the Atmosphere)’
This is not technically correct, as there are no known mechanisms to convert significant vibrational/rotational state energy (which is really just a form of EM energy stored as resonant periodic motion of electron clouds), into the kinetic energy stored as a molecule in motion. Vibration is not like a bat hitting a ball, as the vibrational frequency is very, very fast relative to the amount of time that electron clouds are interacting with each other during a collision.
When an energized GHG molecule collides with N2/.O2, the most likely result is the emission of a photon by the energized GHG molecule, which from a temperature point of view is equivalent to increasing the speed of the molecules since photons and molecules in motion affect most temperature sensors equally based only on the energies involved. Note that the velocity of a GHG molecule in motion is unaffected by absorbing a photon. Moreover; N2/O2 neither absorbs or emits photons in the LWIR, thus its temperature is irrelevant to the radiation leaving the planet while photons emitted by GHG molecules are most relevant.
The exception is collisional broadening, where small amounts of energy are converted between linear kinetic energy and EM energy, however, this has an equal probability of decreasing the velocity emitting a higher frequency photon as it does to increase the velocity decreasing the frequency of the emitted photon, thus only under very special circumstances (for example, laser cooling), can there be any kind of net conversion.
The photons re-emitted by GHG molecules can be absorbed by the water and ice in clouds, which unlike narrow band absorbers/emitters like GHG’s are broad band absorbers/emitters. The water can then indirectly heat the N2/O2, but for the water in the atmosphere to be in LTE, it must be absorbing the same amount of energy its emitting (otherwise, the atmosphere would warm or cool without bound) and as pointed out earlier, the temperature of atmospheric N2/O2 is irrelevant to the radiiant balance. Trenberth conflates energy transported by photons (the radiant balance) and energy transported by matter (latent heat, convection, etc) and this is one of many significant errors found throughout consensus climate science that provides the necessary wiggle room to support an otherwise impossibly high sensitivity.

Reply to  Ira Glickstein, PhD
May 20, 2016 7:41 am

co2isnotevil May 19, 2016 at 12:47 pm
Ira,
You said,
‘2) the absorbed energy causes these gasses to warm up (and by conduction warm the oxygen and nitrogen that make up nearly all of the Atmosphere)’
This is not technically correct, as there are no known mechanisms to convert significant vibrational/rotational state energy (which is really just a form of EM energy stored as resonant periodic motion of electron clouds), into the kinetic energy stored as a molecule in motion. Vibration is not like a bat hitting a ball, as the vibrational frequency is very, very fast relative to the amount of time that electron clouds are interacting with each other during a collision.

Not true, collisional quenching of rovibrational states is commonplace and much studied (it is the bane of those of us doing laser studies of such states around atmospheric pressure, most of my laser excitation ended up being quenched).
See here:
http://www.ncbi.nlm.nih.gov/pubmed/25358152
Another example is the CO2 laser which operates using the vibrational energy levels of the CO2 molecule. For example the vibrationally excited 001 level emits at the 9.6 micron wavelength by a transition to the 020 excited state, to maintain lasing that level must be quenched to the ground state. This is done by collisional deactivation with He which is added to the laser tube for that purpose. As I’m sure you’re aware He only has translational energy.

Reply to  Ira Glickstein, PhD
May 20, 2016 9:20 am

Phil,
As I said, it takes very special circumstances and these circumstances are not common in the atmosphere. For example, shine a laser at a molecule at a frequency just below the peak of an absorption line and subsequent emissions will remove linear kinetic energy when returning to the ground state in order to make up the difference. The distribution of photon energies in the atmosphere are not this constrained.
In the case you suggested where molecules need to return to the ground state, an energized molecule colliding with another molecule will most likely emit a photon and return to the ground state, but as an outside observer, photons and molecular velocities have the same effect on temperature sensors and you will not be able to tell the difference.
Moreover; its a moot point as the kinetic energy of gas molecules in motion has no bearing on the radiative balance, sensitivity or any other important climate attribute. We can detect a hot gas cloud in space only by its emission spectra and a cold gas only by its absorption spectra. N2/O2 has no relevant absorption or emission lines in the LWIR.
The evidence that collisions cause photon emissions is in the emitted spectra of the planet, where despite saturated lines where all surface emissions at a particular frequency are absorbed, there’s significant energy leaving the TOA in those same bands. The only possible origin of these photons in the clear sky is from GHG’s returning to the ground state and emitting photons.

Roger Taguchi
Reply to  Ira Glickstein, PhD
May 24, 2016 3:19 pm

Hi Ira (re posting May 19 at 11:16 am), co2isnotevil (re posting May 19 at 12:47 pm), and Phil (re posting May 20 at 7:41 am)!
1. Phil is right that “collisional quenching of rovibrational states is commonplace and much studied”. This is a major problem when trying to detect the initial products formed at high vibrational and rotational energy levels during elementary chemical reactions such as H + Cl2 = HCl’ + Cl, where H is atomic hydrogen and HCl’ represents the initial product molecules in high (v,J) states, where v & J are the vibrational and rotational quantum numbers. The infrared (IR) photons given off by these initial product molecules were first detected by John Polanyi, who deservedly shared the 1986 Nobel Prize for Chemistry. It takes only a few collisions for high J (non-equilibrium) states to lose energy until a Boltzmann (equilibrium) distribution characterized by a single temperature is achieved. Initial vibrational (v) state distributions were also highly non-Boltzmann (non-equilibrium); population inversions at high v numbers meant the possibility of chemical lasers, first suggested by Polanyi. The possibility of huge chemical lasers in orbit using the reaction H2 + F2 = 2HF’ was the theoretical (though not practical) basis for Ronald Reagan’s “Star Wars” (SDI) program, which nonetheless succeeded in bankrupting the Soviet Union (Evil Empire) and led to its demise. Compared to rotations, it takes many more collisions for vibrational states to relax to an ambient Boltzmann distribution. Part of the energy transferred during deactivating collisions ends up as increased translational energy of the departing molecules, i.e. as an increase in temperature. This is the basis for the greenhouse effect, since the ambient molecules are N2 and O2 which cannot and do not re-emit any significant IR, black body or otherwise.
2.(a) co2isnotevil is correct when he wrote “Note that the velocity of a GHG molecule in motion is unaffected by absorbing a photon [assumed to be involved in a transition between two different quantized (v,J) states]”.
(b) His statement “2) is not technically correct as there are no known mechanisms to convert significant vibration rotation state energy (which is…energy stored as resonant periodic motion of electron clouds) into the kinetic energy stored as a molecule in motion” is wrong [see Point 1 above]. The reason is that the energy is not stored in electron clouds [this IS possible, but then the higher states would be excited electronic states, and for most simple molecules like N2, O2 and CO2, these would involve high-energy ultraviolet (UV) photons, not infrared (IR) photons]. Higher vibrational states involve more extreme vibrational motions of the atoms (with most of the mass in the nuclei); imagine stretching the bonds as springs holding atoms together in molecules. Higher rotational states involve higher rotational speeds of the nuclei of the atoms (imagine twirling a baton faster). Because a gas molecule at atmospheric pressure collides with others about 10^10 times per second (and only reduced by a factor of 4 at 10 km altitude), the rotational energy distributions are at equilibrium at ambient temperatures in both the v=0 ground vibrational state and the v=1 first vibrationally excited state. For CO2, the first excited bond-bending vibrational state lies 667 cm^-1 higher in energy [by hcf = 6.63 x 10^-34 J.s x 3.00 x 10^10 cm/s x 667 cm^-1 J], and at 288.2 K the equilibrium (Boltzmann) distribution means that exp(-hcf/kT) = 0.0355 = 3.55% is the ratio of v=1 to v=0 molecules. Net absorption of 667 cm^-1 photons emitted by the Earth’s surface will increase this ratio. This would be a stress applied to a system at equilibrium, and by Le Chatelier’s Principle the system will shift in such a direction as to partially relieve that stress. The relief comes by having most of the increase in excited molecules lose their energy in radiationless collisions with N2 and O2 molecules, which at 400 ppmv CO2 outnumber CO2 by a factor of 2500 [linear molecules like N2, O2 and CO2 all have heat capacities at constant pressure of 7k/2 per molecule, where k is the Boltzmann constant, so the heat content (enthalpy) is directly proportional to the number of molecules. See https://en.wikipedia.org/wiki/Heat_capacity .] But Le Chatelier’s Principle says that the relief is only partial. At the new equilibrium, there will remain a slightly higher ratio of v=1 to v=0 molecules, and this would correspond to a slightly higher temperature. I.e. the troposphere will have warmed up (the greenhouse effect).
(c) The truth of the statement “When an energized GHG molecule collides with N2/O2, the most likely result is the emission of a photon” depends on the probabilities of spontaneous emission and of deactivating collisions. If the radiation lifetime of the excited state molecule is 10^-3 seconds, at atmospheric pressure there will be about 10^7 collisions in a lifetime. If it takes only several hundred or thousand collisions to deactivate the excited state, then most excited state molecules will be quenched in radiationless collisions.
Most, but not all. Therefore the reverse process can occur, especially at higher altitudes. Collisions between a ground state (v=0) molecule and ambient molecules (mainly N2 and O2) will occasionally result in the formation of an excited (v=1) molecule; at 288.2 K, the ratio of v=1 to v=0 molecules is 3.55%. At 20-40 km altitude, the total pressure is smaller than that at the Earth’s surface by a factor of 20 to several hundred. So at these altitudes, there is a significant probability that an excited molecule will emit a 667 cm^-1 photon that will escape to outer space and not be absorbed by an intervening molecule in the v=0 ground state.
3.(a) Ira has done a good job of researching the literature and evaluating climate sensitivity. However, there are some mistakes in the literature explanation of the physics, for which Ira cannot be faulted (he is not a physicist). However, I believe any intelligent layperson (and Ira has earned a Ph.D., so he is not stupid) can understand the following arguments:
(b) There are 4 points he raises about water vapor and carbon dioxide in the atmosphere:
(1)” they absorb LWR (Long Wavelength Radiation) from the warm Earth surface.” This is correct. The absorption is not even across the Planck black body spectrum emitted from the surface, however. A sample IR spectrum of the TOA outgoing photons is available at http://climateaudit.org/?p=2572 . This is closely modelled by computer calculations in the MODTRAN spectrum available at
https://en.wikipedia.org/wiki/Radiative_forcing .
(2) “this absorbed energy causes these gases to warm up (and by conduction warm the O2 and N2)”.
co2isnotevil correctly pointed out that absorption between (v,J) vibration-rotation states does not initially affect the translational motion. However, collision between the excited (v,J) states and surrounding N2 and O2 molecules does involve transfer to translational modes, i.e. warming (see my Point 1 above), including that of CO2 molecules. So it might be more accurate to say that “this absorbed energy is transferred during radiationless collisions to O2 and N2 molecules which by conduction (further collisions) to CO2 as well, all molecules ending up warmed”.
(3) “the warmed atmosphere radiates LWR in all directions”. This is true for the main greenhouse gases CO2, water vapor and ozone (O3), but not for the main gases of the troposphere N2, O2 and Ar. In particular, the warmed atmosphere does not radiate Planck black body radiation across the spectrum. For energy balance, the TOA flux must be 240 W/m^2, and this corresponds to the total area under the actual spectrum. 240 W/m^2 is also the area under a perfect 255 K Planck black body spectrum (assuming emissivity 1). Equating the two is valid in saying that the TOA flux is equivalent to that of a perfect 255 K Planck black body. But such a perfect 255 K Planck emitting surface does not actually exist, at 4.9 km or elsewhere. Therefore the mechanism for the greenhouse effect is all wrong in the literature. For one thing, everyone agrees that CO2 is such a powerful absorber/emitter at 667 cm^-1 that complete absorption/emission occurs within metres of the Earth’s surface. At 4.9 km altitude, the density is only about half that at the surface. Reducing density by a factor of 2 cannot be expected to allow for “sudden escape of IR photons to outer space”. No, continued absorption/emission must occur until the density is really small, maybe 100 times smaller than that at the Earth’s surface (i.e. in the stratosphere at 20-40 km altitude).
The truncation of the downward CO2 absorption ditch at “220 K” tells us that the emission occurs from the stratosphere at 10-20 km, not at 4.9 km. But there are then two problems with the literature explanation.
First, a perfect 220 K black body emits only 133 W/m^2, nowhere near the 240 W/m^2 required for energy balance. Second, a 215 K CO2 emission from the stratosphere is clearly seen by itself poking above the 210 K Thunderstorm Anvil spectrum in Grant W. Petty’s Fig. 8.3(c) in his excellent book “A First Course in Atmospheric Radiation, Second Edition”. This means that the 220 K CO2 emission is not powered from the Earth’s surface below, but by the absorption of incoming Solar UV and visible radiation by ozone in the stratosphere (which explains why there is a temperature inversion – the temperature actually rises with increasing altitude in the stratosphere). The actual area of the 220 K CO2 stratospheric emission peak corresponds to only 16 W/m^2, way smaller than the 240 W/m^2 required for energy balance.
Yet the most egregious mistake in the literature is calling the actual net absorption spectrum recorded by satellites looking downward on the Earth an “emission spectrum”, and talking about LWR emissions. Yes, there are LWR emissions, but except for the 19 W/m^2 emission from the stratosphere (which includes 3 W/m^2 from ozone in the stratosphere), we ought to consider only the NET ABSORPTION of IR photons. To understand the difference between emission and absorption spectra, see
https://en.wikipedia.org/wiki/Emission_spectrum and
https://en.wikipedia.org/wiki/Fraunhofer_lines and
https://en.wikipedia.org/wiki/Spectral_line .
(4) “some of this [LWR] radiation reaches the Earth’s surface and causes additional warming”. This is wrong, except when there is a temperature inversion near the Earth’s surface [for example, within several hundred metres during Polar winters (nighttime)].
To understand this, imagine a passive perfect 288.2 K black body shell (with no internal energy source) 1 km above and enclosing the Earth’s 288.2 K surface (emissivity 1). The Earth’s surface, and both sides of the shell would emit 391 W/m^2. What would escape the outer surface of the shell? Since it is a perfect black body, it is opaque, and none of the Earth’s surface emission would escape. So the emission is 391 W/m^2, exactly the same as if the shell were not there. But a spectrometer looking up from the Earth’s surface would see 391 W/m^2 “back radiation”, which on the right side of energy budget diagrams in the literature (e.g. Kiehl & Trenberth’s) is shown “warming the surface”. But both the inner surface of the shell and the Earth’s surface are at 288.2 K, the same temperature, so there cannot be a net flow of “downwelling” energy (this would violate the Second Law of Thermodynamics). The obvious explanation is that the Earth’s surface emits 391 W/m^2 which is totally absorbed by the perfect black body shell, so the 391 W/m^2 back radiation can be considered to be made up of photons originally emitted from the Earth’s surface and reflected, after a delay, back to the Earth’s surface. The net change in energy & temperature for the Earth’s surface would then be exactly zero.
There is no additional warming due to back radiation.
In reality, the Earth’s temperature usually decreases linearly with altitude (the lapse rate is -6.8 K/km, which simply says that the temperature changes from 288 K to 220 K over 10 km). So at steady state the “back radiation” is less than the emission from the Earth’s surface (unless there is a temperature inversion in the lower troposphere). The photons in this back radiation can again be considered to be the reflection of photons originally emitted from the Earth’s surface, with no net change in energy or temperature for the Earth’s surface.
4. Because there is a temperature difference between the Earth’s surface and higher altitudes, some have argued that there is no equilibrium, and that this difference is caused by the presence of greenhouse gases (supposedly the loss of IR photons emitted from greenhouse gas molecules at the TOA explains the low temperatures there, and therefore the lapse rate).
5.(a) We will first derive the dry adiabatic lapse rate from first principles.
(b) The gravitational potential energy of a molecule of mass m at height h is U = mgh, where the acceleration due to gravity is g = 9.81 m/s^2.
(c) Gas molecules are also in motion, and therefore contain energy of motion. The heat content (enthalpy) for diatomic molecules like N2 and O2 that make up 99% of dry air is H = 7kT/2 where k = 1.38 x 10^-23 J/K is Boltzmann’s constant [see Cp for diatomic gases at https://en.wikipedia.org/wiki/Heat_capacity , where Cp = dH/dT. ].
(d) If there is no heat transfer into or out of each layer of the troposphere (i.e. if conditions are adiabatic), then as molecules move up against gravity, their potential energy increases, but at the expense of a decrease in heat content (enthalpy). I.e. dU/dh = -dH/dh
(e) Using the Chain Rule for derivatives on the right side, dU/dh = – (dH/dT)(dT/dh)
(f) Therefore d(mgh)/dh = – [d(7kT/2)/dT][dT/dh]
(g) mg = -(7k/2).dT/dh
(h) dT/dh = – 2mg/(7k)
(I) For dry air which is 78.1% by volume nitrogen (N2, molar mass = 28.0 g), 21.0% oxygen (O2, 32.0 g) and 0.9% argon (Ar, 39.95 g), the molar mass = 0.781(28.0) + 0.210(32.0) + 0.009(39.95) = 28.95 g = 0.02895 kg.
(m) Dividing by Avogadro’s Number, 6.022 x 10^23 molecules/mol gives m = 4.81 x 10^-26 kg.
(n) Substituting for m, g and h in 4(h) gives dT/dh = -9.8 x 10^-3 K/m = -9.8 K/km . This is the dry adiabatic lapse rate, which says that temperature decreases linearly with altitude, by 9.8 K per km.
(o) Compare this derivation with the more confusing one at https://en.wikipedia.org/wiki/Lapse_rate .
(p) Note that this derivation does not require, or even include, trace amounts of greenhouse gases. The fundamental principle is that at equilibrium, temperature is not the constant with altitude, but it is the total of U + H which is assumed constant (so that dU/dh = -dH/dh ).
6.(a) The actual lapse rate of -6.8 K/km is only 69% of the dry adiabatic lapse rate in magnitude. This is roughly constant over a wide range of surface temperatures (the temperature profiles of the troposphere are parallel).
(b) The difference in lapse rates is due to injection of heat into the layers of the troposphere due to absorption of outgoing IR photons by greenhouse gases, absorption of outgoing IR by clouds (which emit less at colder cloud tops compared to the warm surface of the Earth), convection currents, and transfer of heat by condensation of liquid water or sublimation of ice crystals in clouds from water vapor produced from condensed states at the Earth’s surface.
(c) We can actually rationalize the magnitude of the actual lapse rate as follows: the MODTRAN spectrum available at https://en.wikipedia.org/wiki/Radiative_forcing shows that at 300 ppmv CO2, the TOA emission at 20 km is 260.12 W/m^2, from a 288.2 K surface which emits 383.34 W/m^2 (assuming emissivity = 0.98). This is a transmission factor of 260.12/383.34 = 0.679, or 68%. In other words, we expect the deviation due to heating of all layers of the troposphere should be 100 – 68 = 32% of the dry adiabatic lapse rate. I.e. the actual lapse rate should be 68% of -9.8 = -6.7 K/km, very close to the actual -6.8 K/km. Coincidence?
I think not. The heating due to NET greenhouse gas absorption (not emission) determines the equilibrium lapse rate. The approach to this equilibrium value is aided by the absorption of clouds (which absorb and emit at all frequencies, since condensed phases are Planck black bodies with emissivities close to 1) and by heat transfer through condensation/sublimation of cloud particles. Since radiative exchange processes take time, and can be too slow over rapidly heating surfaces (e.g. cloudless desert surfaces or equatorial oceanic waters during the daytime), convection currents can carry heat rapidly upward. All these different processes, however, end up driving the temperature profile to the equilibrium lapse rate of -6.8 K/km when there is 32% absorption of the Earth’s surface emission by greenhouse gases.
(d) This means that even though there is a temperature difference with altitude, and an imbalance in backradiation from clouds and greenhouse gases compared to upward emission from the Earth’s surface, there is no change from an average steady state profile, unless greenhouse gases change (for example, CO2 is doubled). Doubling CO2 results in shifting the steady state temperature profile by about 0.5 degrees at all altitudes, including the Earth’s surface. This is the climate sensitivity, which I have calculated separately at
https://wattsupwiththat.com/2016/05/12/negative-climate-feedbacks-are-real-and-large/comment-page-1/#comment-2218029 and following Comments.
I would like to add a small perturbation to the final result, getting 0.59 K for climate sensitivity, including increased stratospheric CO2 emission and 8% positive water vapor feedback, but not including cloud feedback, which is strongly negative, and could lower the total to 0.45 K or even 0.19 K. However, I have to go now, and will Comment further when I have time.

Reply to  Roger Taguchi
May 24, 2016 9:56 pm

Roger,
I agree with most of what you said, but there a few points that need clarification.
You said,
“Collisions between a ground state (v=0) molecule and ambient molecules (mainly N2 and O2) will occasionally result in the formation of an excited (v=1) molecule”
This is nearly impossible in the Earth’s atmosphere. The linear kinetic energy of a CO2 molecule at 500 m/sec is about 8E-21 joules and the energy of a 15u photon is 1.3E-20 joules. There’s simply not enough energy available to result in a transition to the v=1 state, but the fact that it’s close means that the probability of emission upon collision is rather high. Also, while there’s the possibility that the speed of a CO2 molecule will increase upon a collision and emit of a slightly lower frequency photon, there’s an equal probability that the speed will decrease and a slightly higher frequency photon will be emitted. This is the underlying physical mechanism of collisional broadening and is symmetric around the line’s resonant frequency. Note as well that if some energy was transferred in either direction and a photon was not emitted, the state energy will deviate from resonance increasing the probability of spontaneous emission. This can be tweaked to one side or the other under special circumstances, but such circumstances don’t exist in the atmosphere (i.e. laser cooling, etc.)
The example of a BB shell at some temperature surrounding the Earth has a significant difference from Earth, which is that the shell is in equilibrium with the Sun rather then the surface of the Earth whose equivalent ‘shell’ is semi transparent and closer to a gray body then a black body. If the region between the shell in equilibrium with the Sun and the surface was very dense, the temperature closer to the surface would be higher and potentially much higher (as in Venus, where the ‘shell’ is the cloud layer enclosing the planet) owing to a lapse rate imposed by gravity.

HankHenry
May 19, 2016 7:46 am

Great presentation. I do have a quibble with:
“Yes, the Atmospheric “Greenhouse” effect is real. It is responsible for the Earth being about 33⁰C (60⁰F) warmer than it would be absent “Greenhouse” gasses in the Atmosphere.”
If you take the coldness of the ocean abyss into account the surface temperature of the earth is less than the surface air temperature. Remembering that the weight of the atmosphere only amounts to the weight of 33 feet of ocean the cold of the abyss significantly lowers what the surface temperature of the earth is as measured in the air at the surface. This cold really does need to be accounted for when calculating the theoretical surface temp of the earth versus the measured surface temp. From afar the thickness of the atmosphere and the ocean compares to the thickness of the skin of an apple. I could be mistaken but I believe the number quoted in the textbooks for earth’s surface temperature overestimate it.

Michael D Smith
May 19, 2016 10:24 am

¨Gasses¨ is a verb. ¨Frank gasses up the boat¨. Gases is a noun.

Reply to  Michael D Smith
May 19, 2016 10:50 am

‘Gases’ is also plural.

Martin Hertzberg
May 19, 2016 10:46 am

“The greenhouse effect is responsible for the Earth being about 33 C warmer than it would be absent “Greenhouse” gases in the atmosphere”.
That statement is utter rubbish!
It is based on several erroneous assumptions. the most glaring one diminishes the input solar radiation to the Earth by about 37 % because of the Earth’s albedo, while at the same time it ignores the lessening of the IR radiation to free space by those same blocking clouds. In addition it assumes that all surface entities are black body emitters. This issue is considered in more detail in my paper in Energy and Environment, Vol. 20, No 1 2009 pp 83-93. The paper is entitled “Earth’s Radiative Equilibrium in the Solar Irradiance”. The erroneous calculation is referred to as “the Cold Earth Fallacy”.
The calculation is based on many “ifs”…… If, if, if…… As the old saying goes:
“If the Queen had balls she’d be King!”

Reply to  Martin Hertzberg
May 19, 2016 10:50 am

King Caitlin?
Or Queen Caitlin?
It’s all so confusing…

Editor
May 19, 2016 11:06 am

Nicholas Schroeder May 19, 2016 at 10:43 am

http://claesjohnson.blogspot.com/2011/08/who-invented-downwelling-longwave.html

“We find support to our claim that DLR and backradiation is fictional without physical reality, and thus that an essential component of the greenhouse effect is fiction and not physics.

DLR thus appears to be a recent man-made invention, and so the greenhouse effect of CO2 alarmism.

Say what? That’s dumb as a bag of ball bearings. Google “measurement downwelling radiation” and you’ll find reports from scientists all over the world who are measuring downwelling radiation. The idea that it is “fictional without physical reality” is a sick fantasy.
As I said above, you get your own theories, but not your own facts.
w.
PS—Here’s a partial list of the people measuring downwelling longwave radiation … you’ll have to visit each one in turn and inform them that they are so stupid that they haven’t realized that what they are measuring is “fictional without physical reality” …

Measurement : Longwave broadband downwelling irradiance
The total diffuse and direct radiant energy, at wavelengths longer than approximately 4 {mu}m, that is being emitted downwards.
Categories
Radiometric
Instruments
The above measurement is considered scientifically relevant for the following instruments. Refer to the datastream (netcdf) file headers of each instrument for a list of all available measurements, including those recorded for diagnostic or quality assurance purposes.
ARM Instruments
BSRN : Baseline Solar Radiation Network
BRS : Broadband Radiometer Station
CO2FLX : Carbon Dioxide Flux Measurement Systems
PRP : Portable Radiation Package
RAD : Radiation Measurements at AMF
SKYRAD : Sky Radiometers on Stand for Downwelling Radiation
SIRS : Solar and Infrared Radiation Station
MET : Surface Meteorological Instrumentation
External Instruments
ECMWFDIAG : European Centre for Medium Range Weather Forecasts Diagnostic Analyses
ECMWF : European Centre for Medium Range Weather Forecasts Model Data
FLUXNET : FluxNet: Network of Regional CO2 and Energy Flux Tower Networks
MOLTS : Model Output Location Time Series
NOAARAD : NOAA/ESRL/GMD Radiometers
NCEPGFS : National Centers for Environment Prediction Global Forecast System
Field Campaign Instruments
RAD-AIR : Airborne Radiometers
CESSNA-404-ARA : Airborne Research Australia Cessna 404
ARA-DIMONA : Airborne Research Australia Dimona Aircraft Laser Altimeter and Fluxes
SMART-RAD : Broadband Radiometers from NASA SMART Trailer
CO2FLX : Carbon Dioxide Flux Measurement Systems
VARANAL : Constrained Variational Analysis
QCRAD : Data Quality Assessment for ARM Radiation Data
ECMWFDIAG : European Centre for Medium Range Weather Forecasts Diagnostic Analyses
ECMWF : European Centre for Medium Range Weather Forecasts Model Data
FLUXNET : FluxNet: Network of Regional CO2 and Energy Flux Tower Networks
FTIR-RAOB : Fourier Transform Infrared Radiometer and Raobs
MIRAI : JAMSTEC Research Vessel Mirai
MERGED-COMMON : Merged data- common timestamp
RONBROWN : NOAA Research Vessel Ron Brown
ISLAND-GUEST-INSTRUMENTS : Nauru Island guest instruments
PARSL : PNNL’s Atmospheric Remote Sensing Laboratory
QCSFCRAD : Quality Controlled Surface Radiation Data (Long-Shi Experimental)
RAD : Radiation Measurements at AMF
RADFLUXANAL : Radiative Flux Analysis
RSR : Rotating Shading-arm Radiometer
SOAR : Shipboard Oceanographic and Atmospheric Radiation
SWFLUXANAL : Shortwave Flux Analysis
SOLARIRRADS : Solar and Infrared Radiometers
SFCFLUX : Surface Flux
MET : Surface Meteorological Instrumentation
OTTER : Twin Otter
UAV-ALTUS : UAV Altus
UAV-PROTEUS : UAV Proteus
UAV-EGRETT : UAV-Egrett
UAV-GNAT : UAV-General Atomics GNAT
UW-CONVAIR580 : University of Washington Convair 580 Aircraft

Michael 2
Reply to  Willis Eschenbach
May 20, 2016 12:55 pm

With regard to measuring downwelling infrared; it is trivially easy for essentially anyone willing to spend $60 (footnote 1) or so on a reasonably good quality infrared remote measuring thermometer. On a partly cloudy night, point it at a cloud, then at clear sky. The clear sky will go to the lowest possible reading (on my device, -60 F) but clouds will be more or less at their dew point temperature.
This measurement is possible because the device “sees” the infrared radiation from the cloud.
It can also read below ambient which is remarkable. The sensor is itself warm, and radiating infrared. The idea therefore is to insulate the sensor from ambient temperature (in a small capsule with nitrogen or some such gas) and it seeks equilibrium — its own outgoing radiation balances incoming radiation. If you aimed it at absolute zero, and it was adequately insulated, the sensor would reach absolute zero eventually.
1. [http]://www.amazon.com/dp/B00IT0ODPQ?psc=1

Reply to  Michael 2
May 20, 2016 2:36 pm

Regarding “suppression of peer review”, the full context of the quote you are responding to makes it clear I’m talking about peer review:
“If blog auditors such as yourself were tearing giant holes in prevailing scientific knowledge as you claim, I still believe you would fare much better in publishing these criticisms. I know you might disagree, claiming there is suppression/conspiracy against contrary views”
I’m saying, if anti-AGW blog auditors were truly finding gaping holes in science, I personally expect there would be more evidence of them having success publishing these views in journals around the world (across disciplines) and seeing that published research be successful, i.e. influential (referenced, endorsed). I was simply acknowledging that others are free to disagree, as you may put more faith in belief that there is conspiracy-like suppression of contrary views (i.e. science as an institution is broken / not trustworthy). Sounds like you indeed disagree in this manner – great. I don’t have any interest in discussing claims of persecution further; not my first rodeo. Have your last word.

May 19, 2016 11:53 pm

To Ira: Congratulations on a well-written summary of skeptical arguments which carry weight. There is, however, a better estimate for the magnitude of the greenhouse effect than the 33 degrees calculated by Hansen. Here’s my argument:
1. The Sun, with an equatorial radius of 6.963 x 10^5 km, radiates at a Planck black body temperature of 5778 K, at a mean distance of 1.496 x 10^8 km from the Earth [see https://en.wikipedia.org/wiki/Sun ].
2.(a) By the Stefan-Boltzmann law, the Sun radiates from its photosphere surface at
5.670 x 10^-8 x(5778)^4 = 6.320 x 10^7 W/m^2 . [see https://en.wikipedia.org/wiki/Stefan-Boltzmann_law ].
(b) By the time this radiation reaches the Earth, the inverse square law reduces the flux to
6.320 x 10^7 x [6.963 x 10^5/(1.496 x 10^8)]^2 = 1369 W/m^2 . This is the Solar insolation [a value of 1366 W/m^2 is quoted at https://en.wikipedia.org/wiki/Insolation ].
3.(a) If the Earth were a perfect Planck black body, the incoming 1369 W/m^2 would be totally absorbed, and then for energy balance totally re-emitted at longer wavelengths to outer space, which is at 3K for the background microwave radiation left over from the Big Bang.
(b) Because the insolation is applied to a circular cross-section of the Earth, but the outgoing radiation is from the surface of a sphere which has 4 times the area of the cross-section, the outgoing flux would be
1369/4 = 342.25 W/m^2.
(c) When this 342.25 W/m^2 is plugged into the Stefan-Boltzmann law, the temperature of the Earth’s surface would be 278.8 K (5.5 Celsius). Since the Earth’s mean temperature is 288.2 K (15.0 Celsius), warmer by 9.5 degrees than a perfect black body that absorbs all incoming Solar radiation, there must be a mechanism for reducing the amount of radiation lost to outer space. Thus the atmospheric greenhouse effect is real, and those who deny even the possibility are simply wrong. Thus one must honestly admit that there is a greenhouse effect, and CO2 is part of the cause.
(d) The “explanation” I have seen in the literature, however, is incomplete. Infrared (IR) photons emitted by the surface of the Earth are absorbed by CO2 at 667 cm^-1, forming excited state molecules in the bond-bending vibration mode. These excited state molecules can then re-emit IR photons, but because the energy levels are quantized, they have the same frequency & energy. So complete absorption followed by re-emission cannot by themselves result in warming of the troposphere, no matter how many times this occurs. The vibrationally excited CO2 molecules can also be quenched during radiationless collisions with N2, O2, and Ar molecules that make up 99.9% of dry air. The energy does not appear as an emitted photon, but is distributed among the translational and rotational motions of all molecules leaving the collision zone.
Because N2, O2 and Ar are non-polar molecules, they cannot and do not re-emit any significant IR photons, black body or otherwise. So they retain their extra energy of motion; i.e. the troposphere has warmed up (the greenhouse effect). Because N2, O2 and Ar outnumber CO2 by 2500:1 (for 400 ppmv CO2), and because the heat capacity at constant pressure, Cp, for each linear molecule N2, O2 or CO2 is the same, 7k/2, where k is the Boltzmann constant, almost all of the heat content (enthalpy) ends up in the non-radiating main molecules. Thus even a small concentration of CO2 can have a significant effect on the temperature of the troposphere. [Cp for the monatomic gas Ar is only 5k/2 per molecule because it cannot store energy in rotation. See https://en.wikipedia.org/wiki/Heat_capacity .]
4.(a) The measured outgoing IR flux at the TOA (Top Of the Atmosphere) is 240 W/m^2. This corresponds to a perfect black body (emissivity 1) at 255 K (calculated using the Stefan-Boltzmann law backwards).
(b) Hansen then said that the greenhouse effect is 288 – 255 = 33 K [see the original paper in Science, 28 Aug. 1981, Vol. 213, No. 4511, pp. 957-966, which may be available at
http://pubs.giss.nasa.gov/docs/1981/1981_Hansen_etal_1.pdf . Note that there was a typo in the very first equation that got by all the referees and proofreaders (the exponent 4 from the Stefan-Boltzmann law is missing)].
(c) However, this assumes that the albedo of the Earth remains the same, with or without greenhouse gases. This is unrealistic, since water vapor is the main greenhouse gas, and clouds dramatically affect the albedo.
5.(a) Thus we ought to consider an Earth totally devoid of clouds, which means no water vapor, no oceans, lakes, rivers, etc. There would also be no life forms, including trees. Then the albedo would be similar to that of the Moon, at 0.136 [see https://en.wikipedia.org/wiki/Moon ].
(b) Since 13.6% of the incoming Solar radiation would be reflected back to outer space, the flux absorbed would be reduced by that amount, and at energy balance the flux emitted as IR would be reduced to
1369(1-0.136)/4 = 295.7 W/m^2. This would correspond to a black body with emissivity 0.98 at 270.1 K. Therefore the greenhouse effect, still real, would be approximately 288.2 – 270.1 = 18 degrees.
(c) It is true that we have used a hypothetical Earth in this derivation, but so did Hansen, and his hypothetical Earth is IMO less defensible.

richardd verney
Reply to  rogertaguchi
May 20, 2016 4:41 am

All very interesting but we do not even know what the averagee temperature of this planet is. There areNASA papers putting it as low as 8degC, but that is an outlier, and 10 to 16 degress is probably more typical.
However, you overlook 2 important facts. First the storage capacity of the oceans which smooths the tempèrature extremes. Second the average temperature of the oceans is only around 4degC. It is only because we are living in an inrterglacial that the surface seems warm. In perhaps 8,000 years time the low average temperature of the oceans will come back to bite as we reenter the ideep throes of the ce age that we are currently in.
When considering the average temperayure of the Earth it is important to bear in mind that after some 4.5 billion years of solar plus DWLWIR, the average temperature of the oceans is only abot 4 degC
One question that should be answered is why are the oceans so cold?

Reply to  richardd verney
May 20, 2016 5:14 pm

Perhaps your average temperature of 4 Celsius (the temperature at which pure water has its highest density) includes the entire bulk of the oceans. But the emission comes from the opaque layer at the surface, which on average must surely be greater than 4 Celsius (maybe 15 Celsius?). Ditto for the land masses; for example, the thin opaque desert surface emits at a very high temperature in the daytime, but burrowing creatures are quite comfortable fractions of a metre down, due to the low thermal conductivity of the dry sand. The 10 km of the troposphere if compressed 1000 times to solid/liquid density would form a layer about 5.4 m (18 ft.) thick. Adjusting for the different heat capacities of 1000 atm. air molecules per unit volume compared to those for rocks and water, this still corresponds to several feet of water, and more for rocks/sand. The survival of desert animals means that most of the heat involved in daily or seasonal warming is stored as enthalpy in the troposphere, and not in the land and water skins. So using average temperatures for the bulk oceans or solid Earth is not valid (and due to heat stored in the molten core of the Earth, the temperature increases going down mine shafts a km or more deep, but these are not relevant for the Stefan-Boltzmann emission from the thin surface skin).

FTOP_T
Reply to  richardd verney
May 21, 2016 6:18 pm

@Richardd
It is certainly an open question regarding the temperature of the earth’s mean surface temperature. If that mean temp number is higher (highly likely) than the rest of the calculation below is meaningless.
“(c) When this 342.25 W/m^2 is plugged into the Stefan-Boltzmann law, the temperature of the Earth’s surface would be 278.8 K (5.5 Celsius). Since the Earth’s mean temperature is 288.2 K (15.0 Celsius), warmer by 9.5 degrees than a perfect black body that absorbs all incoming Solar radiation, there must be a mechanism for reducing the amount of radiation lost to outer space. Thus the atmospheric greenhouse effect is real, and those who deny even the possibility are simply wrong. Thus one must honestly admit that there is a greenhouse effect, and CO2 is part of the cause.”
The earth’s mean surface temperature as defined by the models is not actually measuring the physical surface. Land can reach a temperature significantly higher than the highest MSAT temperature used to calculate the average for the IPCC earth mean.
See https://tallbloke.wordpress.com/2012/03/21/roy-clark-where-it-all-went-wrong-with-climate-science/
“During the middle of the day, under full summer sun conditions, the peak solar flux is ~1000 W.m-2. The corresponding surface temperature is at least 50 C for dry ground. The increase in LWIR flux for a black body going from 288 to 338 K (+30 C) is about 227 W.m-2. Most of the solar heat is dissipated by convection. Heat is also stored below the surface and released later in the day. It is just impossible for a small change in LWIR flux from CO2 to have any effect on surface temperature when it added correctly to the surface flux balance. At night, convection more or less stops and the surface cools mainly by LWIR emission. The downward LWIR flux from the first 2 km layer of the atmosphere slows the night time surface cooling, but the atmospheric heating process is controlled by convection, not LWIR radiation. Furthermore, this is not an equilibrium process. The troposphere acts as two independent thermal reservoirs. The upper reservoir radiates to space all the time, mainly from the water bands near 5 km. The lower reservoir acts as a night time ‘thermal blanket’. The atmosphere is an open cycle convective heat engine with a radiatively cooled cold reservoir.
Manabe and Wetherald were quite honest about what they were doing. They simply produced an invalid hypothesis that should have been superseded. Later workers just allowed themselves to be seduced by the mathematics of the flux equations and never bothered to validate the models or investigate the real physics. The result is the global warming dogma that we still have today. And the associated corruption.
Now we get to the fraudulent part. Manabe and Wetherald were quite clear that they were calculating a surface temperature, however it was defined. This means the temperature on the ground that we feel with our bare feet. However, there is no long term record of the surface temperature. Instead, the meteorological surface air temperature (MSAT) was substituted for the surface temperature. This is the ‘weather temperature’ that is the air temperature measured in a ventilated enclosure placed at eye level, 1.5 to 2 m above the ground. It is simply impossible for there to be any observable change in MSAT caused by a small change in LWIR flux at the surface below the weather station enclosure.”

Reply to  Ira Glickstein, PhD
May 20, 2016 8:59 am

Ira,
GHG’s may cause 33C of warming, but they also cause 15-18C of cooling owing to reflection from ice and clouds. You can’t get the warming without the corresponding cooling. This is a significant problem with the way the science is presented where only the warming part is acknowledged (positive feedbackl like) and the cooling is ignored (negative feedback like).

Reply to  co2isnotevil
May 20, 2016 12:33 pm

“GHG’s may cause 33C of warming, but they also cause 15-18C of cooling owing to reflection from ice and clouds”
Eh? Are you saying that GHG warming also inevitably *increases* ice (albedo) somehow? Is this a cloud feedback argument?
Your subsequent statements seem to make it clear you are conflating feedback questions with this description of the net GHG effect. The 33C figure concerns idealized equilibrium states and illustrates the order of significance of the GHG effect itself; it’s not trivial to calculate the total feedbacks in a hypothetical planet where all GHGs are spontaneously stripped away. In general I don’t see how it is helpful to muddle these concepts.
The mainstream view does not only acknowledge positive feedbacks and ignore negative (e.g., negative lapse rate feedback). That just sounds like a relatively shallow and unsupportable accusation of academic incompetence/bias/fraud. In general albedo feedback is positive (over time, warming reduces total ice and therefore total ice reflectivity, as so far observed.) Cloud feedback is also not clearly negative because some types of clouds have a heat-trapping effect; it is about the type and locations of cloud changes in response to warming – as IPCC concludes, prevailing trend in research and analysis is that net cloud feedback is perhaps smaller but positive, but with significant uncertainty about that (contributing to the general wide range of ECS estimates).

Reply to  geoffmprice
May 20, 2016 10:56 pm

geoff,
I’m saying that most of the GHG surface warming (I agree its not properly called feedback, at least per Bode, but conventionally it seems to be), comes from water vapor. Water vapor also results in clouds and ice and you can’t get the surface warming from water vapor without the corresponding surface cooling. from ice and clouds. Clouds, ice, latent heat, weather and water vapor absorption/emission are all co-dependent, so you must always consider the end to end effect of the hydro cycle and not just one of its influences.
To calculate the actual net warming of the planet, you need to start with a planet with no ice and clouds which will receive more solar power and be intrinsically warmer.

Reply to  co2isnotevil
May 20, 2016 11:37 pm

“Water vapor also results in clouds and ice and you can’t get the surface warming from water vapor without the corresponding surface cooling”
Certainly no guarantee increased water vapor drives increased ice, as you are also necessarily talking about increasing temperature which causes net ice melt as we’re seeing. In practice, cooling in a global sense has to be an adjustment of radiative equilibrium – a shift in balance between incoming and outgoing radiation from/to space. (Meaning, for example, just piling some more snow on East Antarctica isn’t likely to change earth’s average surface temp – there was already snow there). The main way ice can do this is by reflecting more sunlight back to space, i.e. albedo. But some increase in precipitation on a warming world does not guarantee more total ice cover (as we are seeing).
Anyway, conceptually the remainder of your comment seems not unreasonable. But the logic doesn’t extend to the current predicament the way you implied previously. The thought experiment of removing all GHGs allows you to sort of sidestep the strong positive feedback of water vapor absorption/emission, but this feedback is currently operating (directly observed increases in specific humidity, associated changes in longwave gaps, etc.)

Michael 2
Reply to  geoffmprice
May 21, 2016 4:08 pm

Geoffmprice says “increased water vapor drives increased ice, as you are also necessarily talking about increasing temperature which causes net ice melt”
I believe the idea is that the evaporation takes place in warm climates. The atmosphere then takes the vapor to a cold climate where vapor precipitates as snow. Most of Greenland is permanently below freezing (as is most or all of Antarctica); both places depend on vapor transported from warmer climates.
Should it happen that Greenland becomes the new Temperate Zone well then I’ll just go there.
He also writes “I’m saying, if anti-AGW blog auditors were truly finding gaping holes in science, I personally expect there would be more evidence of them having success publishing these views in journals around the world (across disciplines) and seeing that published research be successful, i.e. influential (referenced, endorsed).”
Your mileage obviously varies. WUWT seems to be highly influential with large numbers of people, but not perhaps influential with the relatively tiny number of people that cluster around each other and are called “peers”, deciding what is true and what is not, but also deciding what gets published.
For reasons too complicated (and redundant) to delve into here, these peers have decided to allow only one opinion to be published. Climategate assures all interested readers that this is indeed the case. What that means is that there *might* be meaningful contrary opinion that on a different planet might well constitute the peer opinion but isn’t permitted here. Then again, it might be as you believe, that there isn’t because there cannot be a contrary opinion.
I don’t need no steenkin peer to tell me what to believe. Just lay it out and let the jury (that’s me) decide. But I need to know that the whole story is on the table. It is like a court of law; if only the prosecutor (or only the defense) was permitted to speak, it is not a proper trial. Also like a court of law, if 20 witnesses all agree that a thing is so, but these witnesses have decided in advance what to say, that is collusion and they are at best worth no more than one witness, but likely none at all. Collusion is bad; I do not trust claims that originate from such arrangements. But neither do I assume they are wrong solely for that reason.
So what opinion might be contrary? Variable opinion seems to exist primarily in consequence and mitigation. It is easy enough to drill an ice core, measure oxygen isotopes (no diffusion problem), and decide it corresponds to temperature somewhere. That’s reasonable. Not proof; but good enough for me. What is not proven, and in my opinion cannot be proven (and therefore isn’t science) is what the temperature is going to be in New York City in 80 years and what the sea level is going to be and whether this is a good thing (for some) or a bad thing (for others) and whether the United States should commit suicide right now versus wait for possible doom. However if the rest of the world can be persuaded to de-industrialize and de-populate, well, that’s how you win at the game of “Risk”.
I also have come to the opinion that GAT (Global Average Temperature) is absolutely meaningless. It might mean something if all measurements were made in exactly the same places, in the same way, without encroachments that alter the micro climates. It still would not be a temperature, it is just an index, an average, a number. It would be like taking the average between a genius and an idiot and getting an I.Q. of “100” which is of course pretty much how you get the “100” in the first place. What does the 100 mean? Nothing! You can decrease the genius, increase the idiot, and still have “100”.
Too many people, (maybe including you ) treat the Earth as if it had the same temperature over its entire surface and could instantly equalize temperature such that a block of ice in the arctic can actually stop global warming until it is all melted OR that, as you suggested above, if the Earth warmed to make more ice it would also melt more ice. There’s little question that the hydrologic cycle would increase its activity with more energy in the system, and decrease its activity (and essentially stop altogether) the colder the Earth becomes. That being the case, increasing this activity seems “good” as that is the principle mechanism for producing fresh water out of salt water.

Reply to  Michael 2
May 22, 2016 7:50 am

” The atmosphere then takes the vapor to a cold climate where vapor precipitates as snow.”
This isn’t a cooling relative to the energy budget, unless you claim albedo increase (ice coverage spreads despite warming temps). Per prior comment, more snow on an already snowy ice sheet doesn’t cause some kind of additional cooling.
“Should it happen that Greenland becomes the new Temperate Zone well then I’ll just go there.”
Yep, you and a few billion more. Isn’t it strange how alarmists don’t recognize how awesome that plan is?
“I don’t need no steenkin peer to tell me what to believe. Just lay it out and let the jury (that’s me) decide.”
Turns out there can be some weaknesses to that approach that unfortunately are inherently invisible to you. 🙂
“if 20 witnesses all agree that a thing is so, but these witnesses have decided in advance what to say, that is collusion and they are at best worth no more than one witness”
This sounds like a flowery way of pushing improbable anti-science conspiracy theories. This is not remotely what peer review is.
“What is not proven, and in my opinion cannot be proven (and therefore isn’t science)”
Common misconceptions about “scientific proof”
http://www.psychologytoday.com/blog/the-scientific-fundamentalist/200811/common-misconceptions-about-science-i-scientific-proof
“is what the temperature is going to be in New York City in 80 years and what the sea level is going to be and whether this is a good thing (for some) or a bad thing (for others) and whether the United States should commit suicide right now versus wait for possible doom.”
There’s lots of uncertainty on all of these things. At the same time, if you put the heat on the water in the pot will warm. It’s not a coincidence that the planet has warmed as generally predicted by Hansen in 1981. The laws of physics govern the overall system. The preponderance of evidence puts some constraints around the uncertainty (AR5 WG1). And the preponderance of evidence points to net negative impact (AR5 WG2).
But you are right this is not the domain of “proof”. This is the domain of likelihood, costs and benefits, and perhaps arch-skeptic Bertrand Russell’s quote that “I am prepared to admit any well-established result of science, not as certainly true, but as sufficiently probable to afford a basis for rational action.”
“However if the rest of the world can be persuaded to de-industrialize and de-populate, well, that’s how you win at the game of “Risk”.”
This is just not sounding very grounded. Who first came up with this diabolical scheme, Arrhenius 1896? US Office of Naval Research in 1945? Revelle 1957? Manabe and Wetherald 1967?
“I also have come to the opinion that GAT (Global Average Temperature) is absolutely meaningless.”
It’s a metric measured in an overall consistent way. Why is it going up?
In general the kinds of questions you raise are addressable in the data. I.e. you can use math and statistics to ask questions and get at answers (or uncertainties) in the data. For example, claims like “well it could be urban heat increases” have been floated for decades, but you can test that by doing things like comparing urban trends to rural trends, and if you can show it necessary, you can build an improved metric which quantifies those problems and adjusts for them.
Again, it is not “divine truth” of global temperature (which would require exactly measuring every vibrating molecule across the surface of the planet) but it is a physically consistent and logically sound (known inaccuracies adjusted) and yet it is still showing warming. Is it *possible* there is some as-yet unknown bias that changes this? Well people have been looking hard for well over a decade, and the process is not *that* complex for an infinite number of massive sampling biases to hide, but many folks here are still looking. Still as yet there is no *compelling* reason to bet the future on this sort of wishful hope.
“Too many people, (maybe including you ) treat the Earth as if it had the same temperature over its entire surface”
This is not remotely the underlying assumption or belief. There are many ways to build a GMST metric to watch (surface air temp, trop. air temp, ocean surface temp) – any reasonable building of that metric shows multi-decade global warming.
“and could instantly equalize temperature such that a block of ice in the arctic can actually stop global warming until it is all melted OR that, as you suggested above, if the Earth warmed to make more ice it would also melt more ice.”
Don’t really follow this part. Acceleration of ice melt at the edges of the sheets (esp. Greenland and West Antarctica) is utterly uncontroversial. Ice mass balance measurements are not very precise, so there aren’t highly reproducible answers to total ice mass balance (the real yardstick of science). Paleoclimate makes it very clear that the sea rises with temperature.
“increasing this activity seems “good” as that is the principle mechanism for producing fresh water out of salt water.”
There are many, many effects of warming and climate change. Unfortunately there is some confidence that “The contrast in precipitation between wet and dry regions and between wet and dry seasons will increase”, and extreme precipitation events in already wet regions doesn’t turn out to be good.
meanwhile…
http://www.nytimes.com/2016/04/10/world/asia/climate-related-death-of-coral-around-world-alarms-scientists.html
etc. (and no, Steele is not correct, heat bleaching of already stressed coral is not a positive event, and evidence is overwhelming that the world’s coral reefs are declining not thriving.)

Michael 2
Reply to  geoffmprice
May 22, 2016 7:11 pm

geoffmprice wrote: (re: The atmosphere then takes the vapor to a cold climate where vapor precipitates as snow.) “This isn’t a cooling relative to the energy budget.”
That is correct. It is energy transport involving phase changes of water (among other things), but that is not why I responded as I did. It seemed you were arguing that a warmer Earth won’t have snow at all as though the entire Earth must be the same temperature; if it is warm enough to evaporate water it is warm enough not to create snow.
Since you admire reality so much I will offer some. This increased transport of energy captured by water vapor, when condensed in the arctic, releases that heat of vaporization into the atmosphere and thus warms the arctic disproportionally as compared to the tropics. “We” have observed this to be so.
It is thus established that water evaporation has increased, but so has precipitation since such things need to balance.
(Re Should it happen that Greenland becomes the new Temperate Zone well then I’ll just go there.)
“You and a few billion more. Isn’t it strange how alarmists don’t recognize how awesome that plan is?”
Yes, it is not strange that alarmists are not like me. The alarmist plan is to implement global socialism immediately; a thing they’ve wished since the days of Plato. Whether doing so has any impact on climate is uncertain, unlikely and for many, irrelevant.
I make no prediction how many human beings will still be alive to migrate to Greenland but as it was Vikings (well, Icelanders to be precise) that found it, it will likely be Vikings that migrate there and it will be thousands, not billions.
I notice that you have not offered your plan what to do with Greenland should it become the new temperate zone. Northern Canada and Siberia will also then be in the Temperate Zone; perhaps one of those locations would be more to your liking.
Re: (I don’t need no steenkin peer to tell me what to believe. Just lay it out and let the jury (that’s me) decide.) “Turns out there can be some weaknesses to that approach that unfortunately are inherently invisible to you. :)”
I appreciate your lack of more substantial response. It confirms my sense that you do not subscribe to personal decision making or democracy. I suspect you feel contempt for libertarians and are very likely a left-wing authoritarian willing and preferring to tell other people what to think and believe rather than try to persuade. If only you had the power…
Re: (if 20 witnesses all agree that a thing is so, but these witnesses have decided in advance what to say, that is collusion and they are at best worth no more than one witness) “This sounds like a flowery way of pushing improbable anti-science conspiracy theories. This is not remotely what peer review is.”
Strange you did not detect the significance of “20”. That’s the RICO 20 headed by Shukla.
Re (I also have come to the opinion that GAT (Global Average Temperature) is absolutely meaningless.) “It’s a metric measured in an overall consistent way. Why is it going up?”
I do not accept its consistency. As to why it is going up, well, here I am studying science so that I can develop my own answers despite the inconsistencies. The hockey stick goes up because its inventor wishes it to go up, more or less.
“and yet it is still showing warming.”
I have not argued otherwise. Few here argue otherwise.
Unfortunately there is some confidence that “The contrast in precipitation between wet and dry regions and between wet and dry seasons will increase”, and extreme precipitation events in already wet regions doesn’t turn out to be good.
That seems reasonable to me. More energy is contained in the atmosphere. However, the predicted increase in storms has failed to materialize, in my opinion, because you need a sink for the heat. Inasmuch as the arctic has warmed faster than the tropics, this “sink” has become filled and is no longer much of a sink, hence fewer storms.
http://www.nytimes.com
The New York Times — famous for announcing that rockets cannot fly because there’s no air in space to push against.
http://www.pddnet.com/blog/2015/01/today-engineering-history-ny-times-claims-rockets-cant-fly
“That Professor Goddard, with his ‘chair’ in Clark College and the countenancing of the Smithsonian Institution, does not know the relation of action and reaction, and of the need to have something better than a vacuum against which to react — to say that would be absurd. Of course he only seems to lack the knowledge ladled out daily in high schools.”
Sounds a lot like the insults handed out to skeptics on a daily basis.

Reply to  Michael 2
May 23, 2016 10:48 am

“It seemed you were arguing that a warmer Earth won’t have snow at all as though the entire Earth must be the same temperature”
I haven’t argued anything remotely like this. Seriously, this is a major disconnect, which might concern you. How are you going to address arguments if you are so polarized on a subject that you have difficulty accurately reflecting what others say?
“Since you admire reality so much I will offer some. This increased transport of energy captured by water vapor, when condensed in the arctic, releases that heat of vaporization into the atmosphere and thus warms the arctic disproportionally as compared to the tropics. “We” have observed this to be so.”
There are a number of proposed or studied reasons for arctic warming, but yes this seems a part of it. It is non-trivial to directly observe/measure this, however, so it is a stretch to say “we have observed this to be so” without supporting justification. The studies I have seen supporting this were model based (if you have stronger citations, I’m happy to see them.)
Btw, in general, you can blanch at the use of “we” all you like, but it doesn’t change the fact that (the world is consistently and verifiably interpreted as a place where) we share a common objective reality. Whether this philosophically horrifies one or not is up to individual preference. Part of me agrees it would be nice if we each could control our own objective reality. We would probably both choose to make spectroscopic greenhouse energy absorption go away in that scenario, given that it is basically a pain.
“Yes, it is not strange that alarmists are not like me. The alarmist plan is to implement global socialism immediately; a thing they’ve wished since the days of Plato.”
I don’t think I need to bother commenting that this is pretty far out there. Most conspiracy theorists put the origins of the great “scientific socialist hoax” back to Fourier in 1824 at the earliest, but you go to Plato! In what part of Plato’s writings did he lay the seeds for the “global warming scientific hoax” and associated modern (secret) conspiracy for “global socialism”?
What an epic arc this plan has had! You would think this is one where they all had to work together to pull it off at this scale and over such a long arc of history: Rosicrucians, Illuminati, Knights Templar, freemasons – all of them, across the board.
“I appreciate your lack of more substantial response. It confirms my sense that you do not subscribe to personal decision making or democracy.”
Well, it’s fair to say I am not a supporter of the idea that democracy means that all opinions are equal and valid, if that is what you mean. There are such things as objectively verifiable facts. Asimov put it rather bluntly: “There is a cult of ignorance in the United States, and there has always been. The strain of anti-intellectualism has been a constant thread winding its way through our political and cultural life, nurtured by the false notion that democracy means that ‘my ignorance is just as good as your knowledge.”
“I suspect you feel contempt for libertarians”
Your judgement is not serving you well. I am a libertarian.
“preferring to tell other people what to think and believe rather than try to persuade.”
Pointing out factual errors is not “telling people what to think”. You are both free to believe things in contradiction of evidence, and you are free to try to present counter evidence.
RE: global average temperature.
“I do not accept its consistency.”
It’s not enough to just assert things. I am talking about consistency in time – measurements are made, and calculations from that, consistently over time. What specifically is inconsistent?
Yes, the context acknowledges that you can have issues like siting changes over time etc. This is why homogenization is done, and this is the sort of thing that I pointed out is subject to statistical analysis and validation. But pointing out the existence of such problems (long studied and addressed) does not prove that the metrics are meaningless as you assert. It remains the case that no longer how you choose to construct a metric reflecting global mean surface temperature, that metric is going up over time. It is not an accident that this is so (and in and of itself this demonstrates that these metrics are not physically meaningless.)
“The hockey stick goes up because its inventor wishes it to go up, more or less.”
You seem to be missing the absolutely crucial role of independent replication in science. The ‘hockey stick’ shape is absolutely in the data no matter how you come at the problem, as at this point many different efforts (using many different kinds of proxies) have shown. Per above if you are talking about the blade (sharp, modern warming uptick) it is the best measured part of the hockey stick shape – it goes up because the planet’s temperature is going up.
“More energy is contained in the atmosphere. However, the predicted increase in storms has failed to materialize, in my opinion, because you need a sink for the heat. Inasmuch as the arctic has warmed faster than the tropics, this “sink” has become filled and is no longer much of a sink, hence fewer storms.”
The interactions are complex, IPCC tends to report low confidence in predictions of this sort. What you say may have an element of truth if the AMOC is slowing due to arctic icemelt and this is slowing hurricane development out of the Caribbean, e.g., which is possible:
http://www.realclimate.org/index.php/archives/2015/03/whats-going-on-in-the-north-atlantic/
with an update just a few days ago from oceanographer Rahmstorf:
http://www.realclimate.org/index.php/archives/2016/05/amoc-slowdown-connecting-the-dots/
Imagining that the world’s coral reef crisis isn’t happening because the NY Times reported it (and you don’t like the NY Times) is formally called an ad hominem logical fallacy.

Michael 2
Reply to  geoffmprice
May 25, 2016 10:15 am

geoffmprice wrote: “I haven’t argued anything remotely like this.”
I suppose it’s time to scroll back and locate the item that aroused my interest.
geoffmprice May 20, 2016 at 11:37 pm “Certainly no guarantee increased water vapor drives increased ice, as you are also necessarily talking about increasing temperature which causes net ice melt as we’re seeing.”
Thus, increased temperature increases water vapor which increases ice formation but that same increased temperature increases “net ice melt”. Those were your words.
“Seriously, this is a major disconnect, which might concern you.”
Hence my continued participation.
“How are you going to address arguments if you are so polarized on a subject that you have difficulty accurately reflecting what others say?”
By avatar (mental model). It’s a bit simple minded as it necessarily only gets a bit of the left brain. Curiously, it can do things my mind cannot or is unwilling even though it is a product of my mind.
It is a common saying that “you are not entitled to your own facts.” But that misrepresents the truth; you choose which facts to put in the library of your mind. Well, I choose my facts; I presume you choose yours. Perhaps sheep do not choose their facts but someone does the choosing for them. That’s the intended purpose of a public education.
“Btw, in general, you can blanch at the use of ‘we’ all you like, but it doesn’t change the fact that we share a common objective reality.”
Unless you sit where I sit, sleep in my bed, eat my food, have my experiences in life, we do not share a common objective reality.
Your comment is astonishing but also interesting. How can anyone imagine that we share a common reality? We would be ants for whom “everything not forbidden is compulsory” (*).
* (re: T.H. White, “Once and Future King” [https]://en.wikiquote.org/wiki/The_Once_and_Future_King)
“Part of me agrees it would be nice if we each could control our own objective reality.”
Indeed, but that would be boring; kinda like making your own first person video game and then playing it.
“We would probably both choose to make spectroscopic greenhouse energy absorption go away in that scenario, given that it is basically a pain.”
I wouldn’t change a thing. I exist, either by luck or by God (both). I exist because my body is adapted to the way things are, not the way I might wish them to be.
“I don’t think I need to bother commenting that this is pretty far out there.”
Yes; you would be wandering into my turf. Besides, why discuss common things already in agreement?
“Most conspiracy theorists put the origins of the great ‘scientific socialist hoax’ back to Fourier in 1824 at the earliest”
By now you’ve noticed that I am not strongly influenced by what “most of” anything does or does not. I have also said nothing about a hoax. Sokal’s hoax was marvelous, but a hoax is no fun if you don’t reveal it at some point.
“but you go to Plato!”
Yes; Plato’s Republic specifically. I’ll admit that I don’t quite comprehend it and may be interpreting it somewhat differently as compared to persons more academically involved in it.
It’s about governance; who should do it, what is the aim of government, that sort of thing.
There’s plenty about U.N., UNFCC, IPCC that is not secret and is decidedly socialist. Simply google “global warming” “social justice” (quoted that way to reduce spurious responses) to see how they are tied together.
“Global Warming and Social Justice. Eric A. Posner. University of Chicago – Law School”
[http]://papers.ssrn.com/sol3/papers.cfm?abstract_id=1121781
“Social justice is the only solution to global warming”
http://climateandcapitalism.com/2015/01/15/social-justice-solution-global-warming/
“What an epic arc this plan has had!”
Yes. The herd instinct never gives up and is numerically superior but relatively unimaginative. So it isn’t really so much an “arc” as the same old ideas born fresh in the minds of most college students who want Free Stuff and have been getting it all their short lives (*). The Force is strong on weak minds. (**)
* [http]://www.socialistinternational.org/ (section on representation at COP21)
** [http]://scifiquotes.net/quotes/154_Weak-minded
“You would think this is one where they all had to work together to pull it off at this scale and over such a long arc of history”
Had they actually worked together (and had the same goal) then global socialism would have happened with King Hammurabi and the Sumerians. The main problems are deciding who is going to lead the herd and where is the herd going?
In the past it was warfare that decided this question. In the future it will be warfare that decides this question. Right now we seem to be between leaders. Make the most of it while you can.
“Well, it’s fair to say I am not a supporter of the idea that democracy means that all opinions are equal and valid, if that is what you mean.”
That is exactly what I mean. On its face I agree with you. The opinion or decision of an intelligent, educated person is inescapably superior. But that superior opinion and a dollar will get you a cup of coffee at Mickey D’s (maybe) and maybe a poke in the eye.
The problem is that the herd rejects differently abled persons; either superior or inferior. The herd wants Conan the Barbarian, and in the modern age of instant communication, wealth and rudeness is taken for leadership, hence Trump and Putin.
The best one can do is hope that Conan the Barbarian chooses wisely his counselors and for you (or me) to be that counselor.
“There are such things as objectively verifiable facts.”
I look forward to the occasional objectively verifiable fact. For now, I accept Descarte’s judgment that the only absolutely verifiable fact is “cogito ergo sum”. Every other fact is perceived through one’s senses and might not actually be there. On the other hand, what seems to be real might as well be treated as real since the alternatives are not productive.
“Your judgement is not serving you well. I am a libertarian.”
Congratulations. It would be interesting to see how the ATTP regulars treat you. They seem to think all libertarians are stupid clones and not very pleasant.
“Pointing out factual errors is not ‘telling people what to think’. You are both free to believe things in contradiction of evidence, and you are free to try to present counter evidence.”
You have expended quite a bit of time “pointing out factual errors” presumably to some purpose. It is an indirect method of telling other people what to think by leveraging their wish to be right; you need only define right and wrong when it is more correct to understand that I define right and wrong.
“This is why homogenization is done, and this is the sort of thing that I pointed out is subject to statistical analysis and validation.”
I comprehend the purpose of homogenization but in the court of public opinion the data has been tampered. Thus the untampered data must always be available for comparison (and so far as I know, this is indeed the case). Where tampered data seems to (nearly) always endorse the Consensus it seems a bit suspicious.
“But pointing out the existence of such problems (long studied and addressed) does not prove that the metrics are meaningless as you assert.”
You conflate my comments to your own sinister purposes. I accept that metrics and indexes are what they are; but an average of temperatures is not itself a temperature because the concept has no meaning. Temperature is a description of the intensity of vibrational energy of atoms and molecules, but not the energy thereof (measured in ergs or joules for instance).
Averaging energy can make sense, averaging temperature does not. Global warming proxies are entirely or for the most part representing temperatures, not energy. Temperature can easily be changed with no change in energy in the case of a gas. So temperature, all by itself, already means little BUT it is what has been measured.
“It remains the case that no longer how you choose to construct a metric reflecting global mean surface temperature, that metric is going up over time.”
So it seems, except for the past 18 years or so, depending on who you ask.
“You seem to be missing the absolutely crucial role of independent replication in science.”
That is so. Everyone is waiting for the independent replication of climate. Perhaps Alpha Centauri has a planet where we can perform this independent replication.
“The ‘hockey stick’ shape is absolutely in the data no matter how you come at the problem”
It is in some data, not in other data. The entire purpose of PCA (Principle Components Analysis) is to choose which proxies to use. For instance, Briffa’s treemometers are highly variable in terms of utility to advancing the hockey stick. Some of them, perhaps the majority, show a decline, hence the phrase “hide the decline”. A few show a nice hockey stick. PCA will detect the correlation and use those particular proxies, not because they are correct, but because they are strongly correlated with other proxies.
Is it correct, meaningful, and actionable? Maybe. This is where your “religion” kicks in. Your faith in the process probably exceeds mine.
“[http]://www.realclimate.org/index.php/archives/2015/03/whats-going-on-in-the-north-atlantic/”
There’s a few advocacy websites that I just don’t see much benefit in visiting. It’s a bit like going to pretty much any website of any religion and noticing that each pronounces itself correct and all others wrong.
Some quotes from it:
“That the real flow may be more unstable than previously thought would be bad news for the future.”
Doom! We’re all going to die. It’s worse than we (well, the models) thought!
“the AMOC has a well-known ‘tipping point’ (Lenton et al., 2008).” Well known to Lenton anyway.
“Hank Roberts says: Wili, we know we’re f*ed.”
“Slioch says: it would appear also to be the first step towards utter catastrophe.”
“richard pauli says: The ramifications are beyond all comprehension. Nothing in all history of civilization is this momentous.” (50 million dead Russians last century probably thought starvation was at least this momentous).
Gotta have anticapitalism: “Leif Knutsen says: Capitalism, unrestrained by the requirements of Planetary life support systems, is guaranteed mutually assured destruction.”
[End of quotes from realclimate]
“Imagining that the world’s coral reef crisis isn’t happening because the NY Times reported it is formally called an ad hominem logical fallacy.”
Your comment is called “straw man”. I have not argued that there is no world coral reef crisis (*).
I have argued that the New York Times is not an authority on climate change (or rockets); or in other words, I have challenged your own “appeal to authority” fallacy. I accept that authorities exist, and to cite an actual authority speaking within his expertise is not a fallacy. But the New York “rockets cannot fly in space” Times is not a science authority.
* I have no opinion on the topic of a coral crisis. I will go on record saying that I reserve the word “crisis” for things that seem a lot more threatening to my existence. Homo sapiens exists because everything that prevented Homo Sapiens experienced a crisis and lost their battles. I am here because dinosaurs are not here.

Reply to  geoffmprice
May 21, 2016 10:05 pm

Geoff,
On Earth, more water vapor does mean more ice and snow. The average summer temp at the S pole is still well below freezing, although the average N pole summer temp is close to freezing. Winter is guaranteed to bring snow to the poles and far below them, so the seasonal ebb and flow of ice and snow will continue no matter what. In fact, even today, the average winter snow line is below the max extent of the glaciers from the last ice age. We can use data from the winter to tell us what the effect on albedo was during the last ice ages.
What you are calling ‘positive’ feedback from water vapor is misleading and only includes its GHG effects. When you account for clouds, the apparent feedback is negative, But like I said, Bode’s feedback analysis has been misapplied to the climate system because the climate does not have powered gain (i.e. an implicit source of infinite power to supply the amplifiers output demands), so calling this feedback is somewhat misleading. There is a static bias which increases the average surface temperature, while feedback from clouds tends to moderate, which is another indication of something that is like negative feedback. Conventionally, its considered that all increase above 255K is due to feedback, but in fact most is due to gain. If you understand Bode’s analysis, you will see that feedback and gain are interchangeable from a mathematical perspective and this can lead to confusion.

Reply to  co2isnotevil
May 22, 2016 8:36 am

“On Earth, more water vapor does mean more ice and snow.”
Except, that’s not what we actually have been observing on earth. More water vapor only happens if the atmosphere and ocean have warmed, which drives an (actually not even controversial) positive albedo feedback i.e. less ice and snow coverage – unless you are disputing the obvious acceleration of ice melt in Greenland and West Antarctica, the net sea ice trends (Despite Antarctic Gains, Global Sea Ice Is Shrinking http://earthobservatory.nasa.gov/IOTD/view.php?id=85246), *and* the paleoclimate record which clearly shows retreating ice and rising seas with temperature.
Some increased snow precipitation at the top of East Antarctica doesn’t provide an additional cooling feedback to the climate system.
“the average winter snow line is below the max extent of the glaciers from the last ice age”
Don’t know why you think comparisons like this are meaningful. Why would the winter snow line be expected to be above the max extent of the glaciers? Why even compare these things? What would this mean relative to the topic?
“What you are calling ‘positive’ feedback from water vapor is misleading and only includes its GHG effects.”
In climate science water vapor feedback refers to total water vapor feedback in two parts: (huge, positive) greenhouse and (smaller, negative) adiabatic lapse rate feedback. Clouds are not technically described under “water vapor” because they are not composed of water vapor. There is nothing “misleading” here, just a minor point of terminology confusion on your part.
“When you account for clouds, the apparent feedback is negative”
Yes, clouds are obviously related to water vapor content. No, they are not ignored by climate science. And no, it is not even in the ballpark of the factual to claim that WV + clouds is negative, this is *wildly* wishful. Clouds both warm (trap) and cool (reflect), and the WG1 summary of available evidence is that cloud feedback *alone* is uncertain but more likely positive (small), i.e. increasing WV generating trapping-type clouds a bit more rapidly than reflecting.
Summing this with the huge positive WV feedback and expecting negative takes you completely out of the world of reason. The entire (large) uncertainty range for ECS (1.5 to 4.5 degrees per doubling) is a net positive feedback range.
If clouds acted as a massive dampening feedback on warming and kept the climate system stable (a) we would have seen evidence of this happening already in the past 50 years and (b) it would become incredibly difficult to explain how the climate changes so significantly into and out of glacial/interglacial states (the cloud feedback would prevent this from happening, especially up against a wimpy long/slow forcing like orbital variation.)
“calling this feedback is somewhat misleading”
The definition in climate science is internally consistent, though the physical characteristics and complexity of the system varies from electrical engineering, yes. Subtle differences in terminology are not proof that electrical engineering principles are being misapplied here, though this is a common bias/error seen in the commentary of electrical engineers who imagine they are instant experts on climate.
“Conventionally, its considered that all increase above 255K is due to feedback, but in fact most is due to gain.”
I still think you are muddying vs. illuminating. The greenhouse effect itself is not “considered to be feedback”.

Michael 2
Reply to  rogertaguchi
May 20, 2016 12:21 pm

Rogertaguchi wrote “Since the Earth’s mean temperature is 288.2 K (15.0 Celsius)… Thus one must honestly admit that there is a greenhouse effect, and CO2 is part of the cause.”
Interesting argument. Earth is warm therefore CO2.

Reply to  Michael 2
May 20, 2016 5:20 pm

The infrared (IR) absorption spectra show that water vapor, CO2, and ozone are the three main greenhouse gases responsible for the temperature being greater than that of a perfect black body (emissivity 1) that absorbs 100% of the incoming Solar insolation and then re-emits all the energy as IR. Skeptics do harm to their cause if they deny reality. The CAGW cultists are of course in total denial, but we ought to be better.

Michael 2
Reply to  rogertaguchi
May 21, 2016 9:16 pm

rogertaguchi “Skeptics do harm to their cause if they deny reality.”
1. Skeptics do not have a shared cause. They simply doubt the prevailing cause.
2. I suspect nearly everyone (a) denies some aspect of reality and (b) defines what is real as needed.
“Reality” is not captured by averages or means.
Suppose I have two blackbodies of identical size and shape. One is twice warmer than the other. S-B says the warmer one will be radiating 2^4 or 16 times more energy than the cooler. Together they radiate 17 times more than the cool one alone.
Now we average them together, each is 1.5 times as warm as the previous cool body. S-B says the cool one will now emit 5 times what it did before, and the warm one will emit the same 5, for a total of 10.
So you see, by averaging temperatures in a linear manner you have gone from 17 units of radiation to just 10. Willis E. had quite a bit to say on this a few years ago on “The Moon is a Cold Mistress”.
I suppose you could apply the T^4 term of the S-B formula to all measurements BEFORE averaging them, in this manner you aren’t actually calculating an average “temperature” but a total theoretical radiation of energy. Of course, emissivity would have to be factored at all measurement locations.

May 20, 2016 9:45 pm

Melting glaciers indicate a warming world. Let’s look at the big picture. We are still in an ice age since 2.58 million years ago. Today’s global temperature is one of the coldest in the last 540 million years. The world has been cooling for the past 3,400 years. Current warming began 250 years ago long before man started burning substantial amounts of fossil fuels. Despite this 250-year warming trend, the longer 3,400-year cooling trend has not been broken. It is intact and we are still heading to the next glacial period.
Even if we burn all our fossil fuel proven reserves at a constant present rate, we cannot stop the coming glacial period. To do that we have to somehow stop the natural CO2 sinks that sequester 14.6 gigatons every year. In the meantime, we can panic and argue about decadal warming and cooling trends.

Reply to  Dr. Strangelove
May 20, 2016 11:17 pm

“Despite this 250-year warming trend, the longer 3,400-year cooling trend has not been broken. It is intact and we are still heading to the next glacial period.”
argument by assertion fallacy. According to energy math, this isn’t remotely true – greenhouse forcing in watts per square meter is an order of magnitude larger than orbital forcing.
“Even if we burn all our fossil fuel proven reserves at a constant present rate, we cannot stop the coming glacial period. To do that we have to somehow stop the natural CO2 sinks that sequester 14.6 gigatons every year”
There are also large natural carbon sources, and sources/sinks had previously found a rough sort of equilibrium (i.e. relatively stable atmospheric concentration). The oceans have absorbed more as we drove the concentration up, absorbing a portion (quarter/third), but the ocean’s ability to do this is declining – as I understand it generally, the thermocline is getting saturated, at which point the rate limiting factor becomes transport from the thermocline to the deep ocean which is much slower. And as the ocean warms it can hold less CO2 generally – eventually it will shift to being a source of CO2 and not a sink, as seen coming out of the last glacial (i.e. the source of the misguided “CO2 only follows temperature” arguments that are common in these parts.)
The declining uptake rate of atmospheric CO2 by land and ocean sinks
http://www.biogeosciences.net/11/3453/2014/bg-11-3453-2014.pdf
No reason to think the atmospheric concentration is “self healing”, especially as we will be burning more sequestered carbon at a good clip at least through most of this century.

Michael 2
Reply to  geoffmprice
May 21, 2016 6:43 pm

“No reason to think the atmospheric concentration is “self healing”, ”
No reason at all. We are all going to die. I figure I’ve got 10 years or so, maybe more, maybe less. Turn off the light and heat and it will be rather definitely less.

Bill Hunter
Reply to  geoffmprice
May 23, 2016 2:48 am

““Despite this 250-year warming trend, the longer 3,400-year cooling trend has not been broken. It is intact and we are still heading to the next glacial period.”
argument by assertion fallacy. According to energy math, this isn’t remotely true – greenhouse forcing in watts per square meter is an order of magnitude larger than orbital forcing.”
Seems to me the first sentence plus the first clause of the 2nd sentence is probably correct and not in error by the assertion fallacy. Of course the 2nd clause of the 2nd sentence is an assertion, but 3,400 year trends are not typically broken by 250 warming periods and it seems that the cooling in the past 3,000 plus years from the Holocene optimum is real and is still intact. When and if we see something unprecedented in the Holocene perhaps that will change.

Reply to  Bill Hunter
May 23, 2016 8:51 am

“3,400 year trends are not typically broken by 250 warming periods and it seems that the cooling in the past 3,000 plus years from the Holocene optimum is real and is still intact.”
Well, you are just repeating the assertion, and seem to be making the same mistake.
The logic is circular: you are assuming that longer term trends must be driven by more powerful forces than shorter term trends, and you use this assumption to argue that the longer trend must be more powerful.
But what is happening physically? Where we’ve seen a sharp change in trend is in the last 150 years:
http://www.countercurrents.org/Marcott_PAGES2k.png
The best supported explanation for the glaciation / ice age cycles are that they are driven by long, very slow changes in solar insolation due to orbital wobble. How much influence do you think this factor has had in the past 150 years? Right – not very much. Orbital angle has barely changed in that geologic eyeblink. However, the greenhouse effect – which is quite powerful and raises the planet’s temperature on the order of 33°C – has been spiked by anthropogenic increases in multiple GHGs. This has caused a much more dramatic change to the energy that escapes into space, which we can directly observe. In climate science such energy influences are measured as forcings in watts per square meter and can be compared. Why do you assume that the slow, subtle, weaker orbital cooling effect must somehow be more powerful (because it is longer/slower) and win out over the new strong anthropogenic greenhouse forcing? Why hasn’t it so far? As the greenhouse effect is only getting jacked up higher, why won’t physics continue to play out how it must? The answer is that there is no reason to believe otherwise; it’s overwhelmingly likely that it will. Your argument is counter-physical. This is why climate scientists keep winning in their predictions of continuing warming.

Reply to  geoffmprice
May 23, 2016 11:15 am

geoffmprice,
By posting that spliced Marcott chart, you lost credibility. Do a search here for ‘Marcott’ and you will see Marcott et. al thoroughly demolished. Anyone who believes a chart like this reflects the real world is just avoiding reality:
http://www.realclimate.org/images//Marcott.png
All your links are to thinly-trafficked alarmist blogs like Tamino, realclimate (are they still in business?), Marcott, and even the NY Times, and Psychology Today.
You constantly deflect, like you did when the issue was rising snowfall. You changed the subject to polar ice.
The bottom line is this: there is nothing unusual happening with global temperatures. Nothing unprecedented is happening. Everything observed now has happened before, repeatedly, and to a much greater degree.
You’re just trying to sell a pig in a poke, and we’re not buying.

Reply to  dbstealey
May 23, 2016 11:43 am

Thanks for stopping by to ad hominem dbstealey. 🙂 Don’t expect responses to non-substantial points. I don’t know what you think I deflected regarding snowfall, or what point of significance you think that connects to. I am not selling anything; I’m a skeptic, and I couldn’t help pointing out a number of factual inaccuracies in the original article. It remains the case that Ira’s point about modern warming just being what happens coming out of a glacial is nonsense. You can (and I’m sure will) ad hominem any paper you like, but the slow cooling of the Holocene is reproduced across a great many global and/or multi-proxy reconstructions – as a result, claiming reality is radically different is counter-factual relative to *available* evidence. No hand-waving or name-calling changes this, sorry.

Reply to  geoffmprice
May 23, 2016 7:47 pm

geoffmprice says:
Thanks for stopping by to ad hominem dbstealey.
I copied your own style, but I wasn’t nearly as ad hominem as your comments to others were, so there’s no need to complain about it. You wrote lots of things about other folks, such as…
Sounds like you are from the “it’s all about pressure” splinter of that particular fringe camp. …completely out of the world of reason. …pushing improbable anti-science conspiracy theories. …I understand your confusion. …you are super far from basic understanding of climate science here …an alternate reality internet bubble. …profoundly counter-factual premises and wild assertions.
That’s more insulting to them than anything I wrote. So, to answer your complaints:
Regarding snowfall, you took exception to the statement:
“On Earth, more water vapor does mean more ice and snow.”
It appears that you believe there has been increased snowfall. Correct me if that’s not what you meant.
Next, you say:
…claims of “fraudulent data” have continually turned up nothing for a couple of decades, certainly including the original hockey stick controversy.
I disagree, and upon your request I’ll post plenty of evidence showing fraudulent data. Just say the word.
Next, re: Mann’s hockey stick. That was so thoroughly deconstructed by McIntyre and McKittrick that the journal Nature was forced to publish a Corregendum. If you’d like to read it, just say the word and I’ll post it (and BTW, a Corregendum is a pretty rare admission).
Next:
…climate scientists keep winning in their predictions of continuing warming.
That self-serving statement can easily be deconstructed. First, it has been known for longer than Michael Mann has been an earthling that the planet has been emerging from the LIA — one of the coldest episodes of the entire Holocene — in fits and starts, for the past few hundred years. No one who is around today can take any credit for “winning their predictions” of global warming. That’s just another baseless assertion.
The correct question is this: What is the primary cause of global warming?
No one has all the answers, but it’s pretty clear that the rise in CO2 is not the primary cause.
No one has ever produced verifiable, empirical, testable measurements quantifying AGW; measurements that are acceptable to both sides of the global warming debate. For one thing, if AGW had been empirically measured and verified, then the question of the climate sensitivity number would be answered, and we would know how much global warming to expect from a specific rise in CO2.
But no one knows the climate’s sensitivity to 2xCO2. There are plenty of guesstimates, ranging from a preposterous 6ºC, to ±3ºC, and on down through ≈2ºC, to ≈1ºC, to a fraction of a degree — to 0.00ºC (F. Miskolczi). Some scientists even say that CO2 causes cooling. So there is still wide disagreement over the effect of CO2 on global warming. Thus, there is no verifiable, testable measurement quantifying AGW, and without that, everything is guesswork. You write:
There is in fact a planetary change in atmospheric conditions – an unambiguous 40% increase in the global concentration of CO2, unprecedented on geological timescales.
You don’t know that, so your “in fact” is merely your opinion. The past century has seen an extremely flat global temperature change — only about ±0.7ºC. That is as flat a temperature change as anything seen in the geologic record. But as we go farther back in the geologic record, we lose the ability to discern decadal CO2 changes. Thus, we don’t know if, or how many times, CO2 has changed by 40% or more over several decades.
What we do know is that quite recently, temperatures have fluctuated by TENS of degrees — and that happened before human CO2 emissions were a factor. So now we see CO2 changing by ≈40%, but current global temperatures have been amazingly flat, while global T fluctuated much more in the past when CO2 was not changing. That pretty much falsifies the belief that CO2 is the ‘control knob’ of planetary temperatures.
You also say:
I’m a skeptic, used to scientific method.
For someone who posts Marcott’s alarming charts, and links to various alarmist blogs (while downplaying published, peer reviewed skeptics like Willis Eschenbach, and attempts to denigrate the sites of scientifc skeptics like McIntyre’s), I’m being polite when I say that your claim of being a skeptic is highly questionable, to say the least.
You’ve clearly taken sides against real scientific skeptics, by ignoring the fact that skeptics have nothing to prove. The onus is on the climate alarmist side to support their conjecture. Specifically, the unproven and measurement-free belief that there is a human ‘fingerprint of global warming’. That conjecture has failed due to the lack of measurments quantifying ‘man-made global warming’.
Your comments amount to merely personal opinions and beliefs, but little else. The rise in CO2 has not caused the endlessly predicted runaway global warming and climate catastrophes, thus debunking the “carbon” scare. Every scary and alarming prediction made by the climate alarmist crowd has failed. Not one of those predictions has ever come true; no exceptions.
When a conjecture like CO2=cAGW results in every alarming prediction based on that conjecture failing to come true, then that conjecture is falsified. As Prof Richard Feynman explains it, if your ‘theory’ is contradicted by observations, then your ‘theory’ is wrong. Feynman added, “That’s all there is to it.”

Reply to  dbstealey
May 23, 2016 8:26 pm

“You wrote lots of things about other folks, such as…”
You didn’t list any ad hominem arguments that I made. Look up what the term means. I didn’t write “about other folks”, but I did address their arguments. Yes, I wrote that “summing this with the huge positive WV feedback and expecting negative takes you completely out of the world of reason”, because it does. Pointing out when claims clearly contradict plentiful evidence is not being nasty. The WV feedback is well characterized and observed, and it is definitely not negative. I think your dispute is with reality.
“It appears that you believe there has been increased snowfall. Correct me if that’s not what you meant.”
I did not bring up the topic of increased snowfall. I don’t know that I’ve seen any reliable measurements of global snowfall. In specific locations like East Antarctica, I think the evidence tentatively supports saying increased snowfall. Globally, I wouldn’t know – there has probably been some increase in total precipitation, but the planet has also been warming. I am just disputing the claim (perhaps it was implied) that ice albedo increases in a warming world – it clearly doesn’t.
“I disagree, and upon your request I’ll post plenty of evidence showing fraudulent data. Just say the word.”
Note that “fraudulent” is a strong term. It has a specific meaning. It doesn’t mean “data I don’t believe”. Nor does it mean “data I think is botched because they applied the wrong statistical technique [even though there’s a good chance I don’t really understand the statistical technique.]” It means data that was made up, and passed off in peer review and the media as real measured data. If you have a case of true, fraudulent data that has corrupted significant findings of climate science – absolutely I’d like to see, this will be big news. Fire away. But don’t think the typical Goddard “look they adjusted Paraguay and I don’t know why!” type claim is going to shock me. I’ve been around the block on the conspiracy blogs.
“Next, re: Mann’s hockey stick. That was so thoroughly deconstructed by McIntyre and McKittrick”
I don’t want to talk about urban legends, even cherished ones. If you really care (I don’t), read one of the many internet summaries of that debate. Pay attention to papers like Wahl & Ammann 2007. This topic is also a good chance to learn about the important role of reproduction in science (critics tend to resist this, because they prefer the conversation to be about whether scientists are trustworthy, as that’s something more easily understood and spinnable via character attacks etc.)
“First, it has been known for more than Michael Mann has been an earthling that the planet has been emerging from the LIA”
This is a meaningless, unphysical statement. First, Pages 2K made it clear “there were no globally synchronous multi-decadal warm or cold intervals that define a worldwide Medieval Warm Period or Little Ice Age”. But let’s say there was a global LIA energy budget event. Giving a name to a low point doesn’t provide any information at all about why the temperature is warming. The implied argument is tautological: “the cause of modern warming is coming out of the LIA”. Well, what’s causing temperatures to rise since the LIA? Something mystical, apparently.
“For one thing, if AGW had been empirically measured and verified, then the question of the climate sensitivity number would be answered, and we would know how much global warming to expect from a specific rise in CO2.”
Meaningless arguments over definitions. AGW is empirically validated in the sense that every scientific national academy, society and agency in the world endorses it – i.e., significant (on same scale as shift into/out of glacial ‘ice age’ states) global warming from significant greenhouse gas increases.
This doesn’t imply every detail is known, and ECS is one particularly large area of important uncertainty about the total system response, but this has more to do with clouds than GHGs.
“You don’t know that [unambiguous 40% increase in the global concentration of CO2, unprecedented on geological timescales], so your “in fact” is merely your opinion”
Merely an opinion… here’s where I make you angry by pointing out that your comments are completely divorced from available evidence/reality. I take it you do not believe ice cores and the like can be used to measure CO2 concentration back in time, despite trapping literal air bubbles and the amount of study and validation that has gone into this process.
I think if we keep talking you’ll end up cycling through every endlessly-debunked climate myth that still circulates on the internet. I’m afraid I won’t be able to keep up.
“For someone who posts Marcott’s alarming charts, and links to various alarmist blogs (while downplaying published, peer reviewed skeptics like Willis Eschenbach, and attempts to denigrate the sites of scientifc skeptics like McIntyre’s, I’m being polite when I say that your claim of being a skeptic is highly questionable.”
We are using different definitions of skepticism. I am using the traditional one – skepticism is a method of critical thinking, not a position.

Reply to  geoffmprice
May 24, 2016 6:56 pm

geoffmprice says:
Pointing out when claims clearly contradict plentiful evidence is not being nasty.
Good thing, no? Because the claims of those who believe that CO2 is the control knob of the planet’s temperature have no credible supporting evidence — and they haven’t been able to credibly refute the solid arguments falsifying that belief.
All of your “evidence” for AGW comes down to the same old ‘Appeal to Corrupted Authorities’ logical fallacy:
AGW is empirically validated in the sense that every scientific national academy, society and agency in the world… &etc.
Thanx for admitting that AGW is nothing more than a belief. A conjecture. An opinion.
That belief is the total supporting “evidence” of the entire ‘CO2=AGW’ argument. Belief isn’t science, it’s really anti-science. It’s religion; in this case, it’s your eco-religion.
And:
I did not bring up the topic of increased snowfall.
Well, you replied to that comment by implying that snowfall has declined. It hasn’t.
Next, you say:
Note that “fraudulent” is a strong term. It has a specific meaning… It means data that was made up, and passed off in peer review and the media as real measured data.
There are dozens, if not hundreds of examples of “adjusted” so-called ‘data’, and in almost all cases the post-adjusted ‘data’ shows more (or scarier) warming. How many more examples would you like?
It’s like Bill Cosby saying, “I didn’t do it!” After dozens of unrelated women step up and say he did, his protests become unconvincing. Same-same.
Next, you talk about Mann’s debunked ‘hokey stick’ chart — and then mention the LIA?? Since you don’t seem to know: Mann attempted to erase the LIA. And also the MWP. They don’t appear on his chart:
http://www.coyoteblog.com/photos/uncategorized/hockeystick.gif
Since you’re not aware of why Mann was forced to write a Corrigendum that Nature was forced to publish, why go on? Your mind is made up and closed tight.
But for other readers: there is nothing unprecedented or unusual happening with global temperatures. Nothing. Despite the large rise in (harmless, beneficial) CO2, global temperatures did not go up as endlessly predicted.
That fact alone falsifies the CO2=CAGW conjecture. QED

May 21, 2016 8:43 am

About 340 W/m^2 strikes the ToA (top of atmosphere). About 100 W/m^2 reflects straight away by albedo which is clouds, ocean and vegetation. This leaves about 240 W/m^2 to be absorbed by the atmosphere, about 80 W/m^2, and reach the surface of the earth, about 160 W/m^2. All of these power fluxes are “abouts” because the uncertainties range from +/- 0.4%, +/- 1.5 W/m^2, to +/- 5%, +/- 17 W/m^2. (Trenberth et. al. 2011, Atmospheric Moisture Transports from Ocean to Land and Global Energy Flows in Reanalyses, Figure 10)
If something, anything, interferes in the amount of energy leaving ToA and reduces it by, say, CO2’s 2 W/m^2 then leaving ToA becomes 238 W/m^2. If the amount entering exceeds the amount entering, all thing being equal (which they aren’t) then the temperature will have to rise until the ToA is once again 240 W/m^2 per the heat transfer equation Q = U * A * dT. If U goes down from added insulation (blankets), then dT must go up. If the balanced earth hot side is 15 C, 288 K, then it will have to increase to make up for the decrease in U.
But suppose albedo increases from 100 W/m^2 to 102 W/m^2. Now only 238 W/m^2 reach atmosphere and earth, but at 15 C, 288 K, and per the equation, 240 W/m^2 are still leaving ToA. So to rebalance the heat flow net heat will be lost until the temperature drops to the point that 238 W/m^2 are leaving ToA.
Suppose the oceans absorb 2 W/m^2. Same scenario. Now there is only 238 W/m^2 leaving the atmosphere and earth’s surface and 240 W/m^2 leaving ToA. The atmosphere will have to cool until the dT equals 238 W/m^2.
This is why 7 of the 8 re-analyses discussed in the Trenberth paper and displayed on Figure 10 show – W/m^2, +/- 3.6% or +/ 12.3 W/m^2. As you can see even RCP 8.5 is lost in this 24.6 W/m^2 uncertainty band.
Whether the 340 W/m^2 GHG loop is real or not, it isn’t, is moot because al that matters is the ToA balance and, considering the magnitudes and uncertainties of the various power flux components, CO2/GHGs role in this balance is trivial.

Reply to  Nicholas Schroeder
May 22, 2016 11:34 am

I’ve been watching how some people are throwing about all this watts per square meter nonsense for some time, as some sort of justification in their understanding of a subject that is based on the idea of energy balance, it is laughable, seriously??
GIGO

Roger Taguchi
May 25, 2016 3:05 am

Reply to co2isnotevil re posting on May 24, 2016 at 9:56 pm, and to Ira Glickstein on May 24, 2016 at 11:08 pm:
1. Re: impossibility of translational energy forming v=1 excited state CO2 molecule. At any temperature, there is a Maxwell-Boltzmann distribution of molecular velocities proportional to v^2.exp[-mv^2/(2kT)]. This means that there are a few molecules moving at velocities higher than average, even 2 or 3 times greater, and also some moving more slowly than average. Therefore there are going to be collisions between molecules with components toward each other with relative translational kinetic energies easily greater than the energy gap between the v=0 and v=1 vibrational states.
2. Re: the black body shell is in equilibrium with the Sun, not the Earth. First, the Sun at 5778 K, the Earth’s surface at 288.2 K, and the cosmic microwave background radiation at 3 K are not at thermal equilibrium. There is a net flow of energy via photons (via radiation, since the vacuum of space is a lousy conductor of heat, and convection is irrelevant) from Sun to Earth to the cosmic microwave background. This is basically a steady state situation (where there is a flow, or flux of photons), not an equilibrium where flows are equal in both directions. The net flow of energy/power from the Sun to the Earth means that some of the incoming Solar radiation is reflected (the albedo) and the rest is absorbed by the dark Earth, which then warms up. However, solid and liquids with emissivities very close to 1 at 288.2 K emit LWR (IR photons) outward to space, trying to warm up the 3 K background. At steady state (energy balance), the LWR emission + the reflected radiation from the Earth must equal the incoming Solar radiation (usually expressed as a power per square metre, which is energy per second per square metre, so they are proportional, and may be used interchangeably without being used ignorantly, as some naïve critics might conclude). At such a steady state, the surface temperature of the Earth would be constant (or at least vary sinusoidally daily or seasonally). Is there a flow of photons from the 288.2 K Earth to the 5778 K Sun? Sure there is, but at
(288.2)^4 relative to (5778)^4 from the Sun to the Earth, so the NET flow is from the Sun (higher T) to the Earth (lower T), in keeping with the Second Law of Thermodynamics. Is there a flow of photons from outer space to the Earth? Sure there is, at 3^4 relative to (288.2)^4, but the NET flow is from the Earth to outer space. At thermal equilibrium, outer space, the Earth and the Sun would be at the SAME (surface) temperature, and the flux from outer space to the Earth to the Sun would equal that in the other direction.
But we aren’t there yet. As the stars, including the Sun, run through their nuclear fusion fuel at the centers, they will eventually all die, leaving slowly cooling white dwarf stars (for the Sun). And the Earth would eventually cool, as heat from the molten interior slowly moves outward by conduction to the surface and then radiate out to space. This would then approach the “heat death” of the universe, and since the distant clusters of galaxies appear to be accelerating outward from each other and us, thermal equilibrium via radiation may never be reached as everything cools toward an unattainable limit of absolute zero.
In our present steady state situation, I asked the reader to consider a thin black body shell around the Earth which would be in thermal equilibrium with the Earth’s surface, at 288.2 K, in order to see that although there is undeniable back radiation as seen from the Earth’s surface, it cannot warm the Earth. The reason is that the Earth’s outward IR radiation (without the shell) is powered by incoming visible radiation from the Sun. The thin 100% opaque shell (with the Sun taken out of the picture) would absorb outgoing IR radiation from the Earth’s surface until it is at the same 288.2 K, and then radiate both outward and inward. The inward flux would just balance the flux outward from the Earth’s surface, so we could then set them to net zero. The outer surface of the shell would emit at 288.2 K, but being thin, it would rapidly cool since it has no power source inside or outside to maintain the temperature. But the high heat capacity of the massive Earth would maintain its outward emission at 288.2 K, and THIS would be the power source that maintains the thin shell at 288.2 K. So basically a photon emitted from the outer shell is equivalent to a photon emitted from the Earth’s surface and travelling unimpeded through the shell (a perfect black body will absorb with 100% efficiency, but then re-emit with 100% efficiency). We can immediately see the equivalence if the thin shell were shrunk inward until it touched the Earth’s surface at all points. We now would have a single black body surface emitting outward from a 288.2 K surface. Both the back-radiation and the photons needed to balance the back-radiation instantly disappear when the gap disappears. Back-radiation does not lead to warming of the surface, so it ought to be taken out of energy balance diagrams.
In the case of the real Earth, the outgoing IR photons emitted from the 288.2 K surface experience net absorption by greenhouse gases water vapor, CO2 and ozone (with a smidgen methane, CH4). This absorbed energy warms the troposphere and the surface until the surface temperature is high enough that the increased Stefan-Boltzmann T^4 emission minus the net absorption equals 240 W/m^2, the radiance needed to exactly balance the incoming Solar radiation that is absorbed by the Earth’s surface, clouds and atmosphere. I have to say “net” absorption, because there is about 19 W/m^2 emission from CO2 and ozone in the stratosphere which is added to a greater CO2 absorption ditch [net absorption consists of about 38.1 W/m^2 for CO2, 80.4 W/m^2 for water vapor, and 4.7 W/m^2 for ozone, for a total of 123.2 W/m^2, although the accuracy may be only to the nearest W/m^2 for any gas].
I have expanded my explanation into great detail which I hope clarifies any confusion due to my shorter, less understandable one.
3. I have taken the time to write this out, because I respect the general knowledge and thought processes of co2isnotevil, Ira, rishrac, cba (on May 7 at 3:00 pm) and Phil who have raised valid points. There is no shame in being wrong, or partially ignorant, is one is willing to consider alternative arguments, admit mistakes, and make corrections when necessary. For example, there is a typo in Point 5(n) in my posting of May 24 at 3:19 pm: “m, g and h” should have read “m, g and k”, and I trust that this will not be used to condemn me for utter stupidity and/or incompetence. The CAGW cultists, however, I don’t trust.

Michael 2
Reply to  Roger Taguchi
May 25, 2016 10:40 am

I appreciate your detailed explanation.